Final Renal, Shock, Burn, Liver, Reproductive***

Ace your homework & exams now with Quizwiz!

The nurse is collecting data from a client. Which symptom described by the client is characteristic of an early symptom of benign prostatic hyperplasia? 1. nocturia 2. scrotal edema 3. occasional constipation 4. decreased force in the stream of urine

4. decreased force in the stream of urine Rationale: decreased force in the stream of urine is an early symptom of benign prostatic hyperplasia. The stream later becomes weak and dribbling. The client then may develop hematuria, frequency, urgency, urge incontinence, and nocturia. If untreated, complete obstruction and urinary retention can occur. Constipation or scrotal edema is not associated with benign prostatic hyperplasia

Which activities are most important for the nurse to teach a client with esophageal varices to prevent harm from bleeding or hemorrhage? Select all that apply. A. Avoid alcoholic beverages. B. Eat soft foods and cool liquids. C. Do not engage in strenuous exercise or heavy lifting. D. Try to eat six smaller meals daily instead of three larger ones. E. Be sure to keep your mouth open when sneezing or coughing. F. Cross your legs only at the ankles when sitting, rather than the knees.

B. Eat soft foods and cool liquids. C. Do not engage in strenuous exercise or heavy lifting.

Which instruction will the nurse give the client immediately before a pelvic exam and Pap test? A. Clean your genitals with warm water. B. Empty your bladder in the bathroom. C. Relax and the test will be less painful. D. Lie flat on the table until the test is over.

B. Empty your bladder in the bathroom.

A clients urinary catheter is removed on the 3rd post-operative day following his transurethral resection of the prostate. Afterward, he is upset because he is dribbling urine. Which is the most appropriate nursing intervention in this situation? Notify the physician Instruct the client to limit fluid intake to 1000 ml/day to prevent dribbling Obtain urine for culture, because he may have a urinary infection Explain to him that dribbling may take some time to clear up and assist him to keep dry.

Explain to him that dribbling may take some time to clear up and assist him to keep dry.

Which statement expresses a correct understanding regarding the treatment of acute pancreatitis? "Always avoid calcium and magnesium replacement therapy" "The main focus is to increase pancreatic stimulation" "Withhold foods and fluids (NPO) during the acute period" Do not rely on the passage of flatus or bowel movement as indicator of peristalsis return"

"Withhold foods and fluids (NPO) during the acute period"

1. A nurse is completing the admission assessment of a client who has renal calculi. Which of the following findings should the nurse expect? A. Bradycardia B. diaphoresis C. Nocturia D. Bradypnea

1. A. Tachycardia is a manifestation associated with a client who has renal calculi. B. CORRECT: diaphoresis is a manifestation associated with a client who has renal calculi. C. Oliguria is a manifestation associated with a client who has renal calculi. d. Tachypnea is a manifestation associated with a client who has renal calculi. NCLEX® Connection: Physiological Adaptation, Pathophysiology

A client at risk for shock secondary to pneumonia develops restlessness and is agitated and confused. Urinary output has decreased, and the blood pres- sure is 92/68 mm Hg. The nurse suspects which stage of shock based on this data? 1. Stage 1 2. Stage 2 3. Stage 3 4. Stage 4

2. Stage 2

A client with myocardial infarction is develop- ing cardiogenic shock. Which potential condition would the nurse anticipate and monitor the client for to detect cardiogenic shock? 1. Pulsus paradoxus 2. Ventricular dysrhythmias 3. Rising diastolic blood pressure 4. Falling central venous pressure

2. Ventricular dysrhythmias

20. Which specific client symptom indicates to the nurse that septic shock (Sepsis-3) may be present? A. Hypotension B. Pale, clammy skin C. Anxiety and confusion D. Oozing of blood at the IV site

20. D. Oozing of blood at the IV site With septic shock, compared to other types of shock, inappropriate clotting occurs with the formation of microthrombi that use available clotting factors and platelets. When these substances are depleted by excessive microclotting, bleeding can occur from any orifice or site of tissue injury. The other symptoms, although present in septic shock, are also present in every other type of shock and are not specific for septic shock.

For which client will the nurse expect extracorporeal shock wave lithotripsy (ESWL) as treat' ment for gallstones to be contraindicated?

30-year-old who is 70 inches (1.75 m) tall and weighs 325 lb (147.2 kg)

Which clients will the nurse recognize as having a higher risk for development of acute pancreatitis?

34-yearold man with Stage n HIV disease 40-'Year-old woman who has had cholelithiasis for 3 years 56-old man who drinks alcohol heavily and is underweight

Which of the following lab profiles is consistent with a diagnosis of acute pancreatitis? WBC = 12,500 mm3 Creatinine = 1.7 mg/dL Platelets = 140,000 C-Reactive Protein = 2.1 mg/dL

WBC = 12,500 mm3

Which of the following lab results would indicate to the nurse that the patient who is in the acute recovery phase from a burn injury is at risk of developing the most serious complication which occurs at this phase? Creatinine = 1.7 mg/dL C-Reactive Protein = 2.1 mg/dL WBC = 12,500 mm3 Platelets = 140,000

WBC = 12,500 mm3

A client in shock develops a central venous pres- sure (CVP) of 2 mm Hg and mean arterial pressure (MAP) of 60 mm Hg. Which prescribed interven- tion would the nurse implement rst? 1. Increase the rate of O2 ow. 2. Obtain arterial blood gas results. 3. Insert an indwelling urinary catheter. 4. Increase the rate of intravenous (IV) uids.

4. Increase the rate of intravenous (IV) uids.

A client complains of fever, perineal pain, and urinary urgency, frequency, and dysuria. To assess whether the client's problem is related to bacterial prostatitis, the nurse reviews the results of the prostate examination for which characteristic of this disorder? 1. soft and swollen prostate gland 2. swollen and boggy prostate gland 3. tender and edematous prostate gland 4. tender indurated prostate gland that is warm to the touch

4. tender indurated prostate gland that is warm to the touch Rationale: the client with bacterial prostatitis has a swollen and tender prostate gland that is also warm to the touch, firm, and indurated. Systemic symptoms include fever with chills, perineal and low back pain, and signs of UTI, which often accompany the disorder

Which client will the nurse recognize as having most risk factors for cholelithiasis?

50-yearold Mexican-American female who has three children and takes hormone replacement therapy

Which client will the nurse recognize as having highest risk for pancreatic cancer?

60-year old man who smokes two packs of cigarettes daily and has liver cirrhosis

Which client will the nurse recognize as having the greatest risk for nonacoholic fatty liver disease (NAFLD)? A. 45-year-old Latino man who is 30 lb (13.9 kg) overweight and has type 2 diabetes B. 50-year-old white woman who drinks one glass of wine daily and has breast cancer C. 60-year-old black woman who is hypertensive and takes a diuretic daily D. 70-year-old Asian man who has gastroesophageal reflux disease (GERD)

A. 45-year-old Latino man who is 30 lb (13.9 kg) overweight and has type 2 diabetes

Which precaution is most important for the nurse to instruct clients with hepatitis C (HCV) who are receiving drug therapy with any second-generation protease inhibitor? A. Avoid crowds and people who are ill. B. Do not touch these drugs with your bare hands. C. Alternate periods of activity with periods of rest. D. Be sure to take vitamin K supplements with this drug.

A. Avoid crowds and people who are ill.

About which factors will the nurse ask a client who is suspected of having uterine leiomyomas? Select all that apply. A. Hypertension B. Type 2 diabetes C. Early menarche D. Alcohol use E. Postmenopausal F. Heavy consumption of red meat

A. Hypertension C. Early menarche D. Alcohol use F. Heavy consumption of red meat

What will the nurse recognize as the cause of splenomegaly in a client who has cirrhosis? A. Increased pressure in the portal vein causing backflow of blood into the spleen B. The loss of cellular regulation in the liver spreading to the spleen and causing extensive scarring C. Chronic inflammation and infection increasing the spleen's maturation and release of white blood cells D. Direct destruction of spleen cells from alcohol or other toxins causing replacement with scar tissue formation

A. Increased pressure in the portal vein causing backflow of blood into the spleen

5. In addition to the pelvic examination, for which diagnostic test will the nurse most likely prepare a client suspected to have uterine leiomyomas? A. Transvaginal ultrasound B. Laparoscopy C. Hysteroscopy D. Endometrial biopsy

A. Transvaginal ultrasound

19. Which common nonsexually transmitted causes will the nurse ask a client about to determine possible causes of vulvovaginitis? Select all that apply. A. Yeast infection B. Herpes simplex C. Feminine hygiene sprays D. Pediculosis pubis E. Tight-fitting clothes F. Vaginal sponges

A. Yeast infection C. Feminine hygiene sprays E. Tight-fitting clothes F. Vaginal sponges

1. The nurse is conducting a history on a male client to determine the severity of symptoms associated with prostate enlargement. Which finding is cause for prompt action by the nurse? a.Cloudy urine b.Urinary hesitancy c.Post-void dribbling d.Weak urinary stream

ANS: A Cloudy urine could indicate infection due to possible urine retention and should be a priority action. Common symptoms of benign prostatic hyperplasia are urinary hesitancy, post-void dribbling, and a weak urinary stream due to the enlarged prostate causing bladder outlet obstruction.

A nurse assesses a client who is recovering from a Whipple procedure. Which assessment finding alerts the nurse to urgently contact the health care provider? a. Drainage from a fistula b. Absent bowel sounds c. Pain at the incision site d. Nasogastric (NG) tube drainage

ANS: A Complications of a Whipple procedure include secretions that drain from a fistula and peritonitis. Absent bowel sounds, pain at the incision site, and NG tube drainage are normal postoperative findings.

6. A nurse is caring for four postoperative clients who each had a total abdominal hysterectomy. Which client should the nurse assess first upon initial rounding? a. Client who has had two saturated perineal pads in the last 2 hours b. Client with a temperature of 99 F and blood pressure of 115/73 mm Hg c. Client who has pain of 4 on a scale of 0 to 10 d. Client with a urinary catheter output of 150 mL in the last 3 hours

ANS: A Normal vaginal bleeding should be less than one saturated perineal pad in 4 hours. Two saturated pads in such a short time could indicate hemorrhage, which is a priority. The other clients also have needs, but the client with excessive bleeding should be assessed first.

15. A 70-kg adult with chronic renal failure is on a 40-g protein diet. The client has a reduced glomerular filtration rate and is not undergoing dialysis. Which result would give the nurse the most concern? a. Albumin level of 2.5 g/dL b. Phosphorus level of 5 mg/dL c. Sodium level of 135 mmol/L d. Potassium level of 5.5 mmol/L

ANS: A Protein restriction is necessary with chronic renal failure due to the buildup of waste products from protein breakdown. The nurse would be concerned with the low albumin level since this indicates that the protein in the diet is not enough for the client's metabolic needs. The electrolyte values are not related to the protein-restricted diet.

3. A male client comes into the emergency department with a serum creatinine of 2.2 mg/dL and a blood urea nitrogen (BUN) of 24 mL/dL. What question should the nurse ask first when taking this client's history? a. "Have you been taking any aspirin, ibuprofen, or naproxen recently?" b. "Do you have anyone in your family with renal failure?" c. "Have you had a diet that is low in protein recently?" d. "Has a relative had a kidney transplant lately?"

ANS: A There are some medications that are nephrotoxic, such as the nonsteroidal anti-inflammatory drugs ibuprofen, aspirin, and naproxen. This would be a good question to initially ask the client since both the serum creatinine and BUN are elevated, indicating some renal problems. A family history of renal failure and kidney transplantation would not be part of the questioning and could cause anxiety in the client. A diet high in protein could be a factor in an increased BUN.

A nurse assesses a client who is recovering from an open Whipple procedure. Which action should the nurse perform first? a. Assess the client's endotracheal tube with 40% FiO2. b. Insert an indwelling Foley catheter to gravity drainage. c. Place the client's nasogastric tube to low intermittent suction. d. Start lactated Ringer's solution through an intravenous catheter.

ANS: A Using the ABCs, airway and oxygenation status should always be assessed first, so checking the endotracheal tube is the first action. Next, the nurse should start the IV line (circulation). After that, the Foley catheter can be inserted and the nasogastric tube can be set.

A nurse cares for a client who has cirrhosis of the liver. Which action should the nurse take to decrease the presence of ascites? a. Monitor intake and output. b. Provide a low-sodium diet. c. Increase oral fluid intake. d. Weigh the client daily.

ANS: B A low-sodium diet is one means of controlling abdominal fluid collection. Monitoring intake and output does not control fluid accumulation, nor does weighing the client. These interventions merely assess or monitor the situation. Increasing fluid intake would not be helpful.

9. The nurse is caring for four clients with chronic kidney disease. Which client should the nurse assess first upon initial rounding? a. Woman with a blood pressure of 158/90 mm Hg b. Client with Kussmaul respirations c. Man with skin itching from head to toe d. Client with halitosis and stomatitis

ANS: B Kussmaul respirations indicate a worsening of chronic kidney disease (CKD). The client is increasing the rate and depth of breathing to excrete carbon dioxide through the lungs. Hypertension is common in most clients with CKD, and skin itching increases with calcium-phosphate imbalances, another common finding in CKD. Uremia from CKD causes ammonia to be formed, resulting in the common findings of halitosis and stomatitis.

10. A client has recently been diagnosed with stage III endometrial cancer and asks the nurse for an explanation. What response by the nurse is correct about the staging of the cancer? a. The cancer has spread to the mucosa of the bowel and bladder. b. It has reached the vagina or lymph nodes. c. The cancer now involves the cervix. d. It is contained in the endometrium of the cervix.

ANS: B Stage III of endometrial cancer reaches the vagina or lymph nodes. Stage I is confined to the endometrium. Stage II involves the cervix, and stage IV spreads to the bowel or bladder mucosa and/or beyond the pelvis.

A nurse plans care for a client with acute pancreatitis. Which intervention should the nurse include in this client's plan of care to reduce discomfort? a. Administer morphine sulfate intravenously every 4 hours as needed. b. Maintain nothing by mouth (NPO) and administer intravenous fluids. c. Provide small, frequent feedings with no concentrated sweets. d. Place the client in semi-Fowler's position with the head of bed elevated.

ANS: B The client should be kept NPO to reduce GI activity and reduce pancreatic enzyme production. IV fluids should be used to prevent dehydration. The client may need a nasogastric tube. Pain medications should be given around the clock and more frequently than every 4 to 6 hours. A fetal position with legs drawn up to the chest will promote comfort.

An emergency room nurse assesses a client after a motor vehicle crash. The nurse notices a "steering wheel mark" across the client's chest. Which action should the nurse take? a. Ask the client where in the car he or she was sitting during the crash. b. Assess the client by gently palpating the abdomen for tenderness. c. Notify the laboratory to draw blood for blood type and crossmatch. d. Place the client on the stretcher in reverse Trendelenburg position.

ANS: B The liver is often injured by a steering wheel in a motor vehicle crash. Because the client's chest was marked by the steering wheel, the nurse should perform an abdominal assessment. Assessing the client's position in the crash is not needed because of the steering wheel imprint. The client may or may not need a blood transfusion. The client does not need to be in reverse Trendelenburg position.

A nurse cares for a client who is prescribed patient-controlled analgesia (PCA) after a cholecystectomy. The client states, "When I wake up I am in pain." Which action should the nurse take? a. Administer intravenous morphine while the client sleeps. b. Encourage the client to use the PCA pump upon awakening. c. Contact the provider and request a different analgesic. d. Ask a family member to initiate the PCA pump for the client

ANS: B The nurse should encourage the client to use the PCA pump prior to napping and upon awakening. Administering additional intravenous morphine while the client sleeps places the client at risk for respiratory depression. The nurse should also evaluate dosages received compared with dosages requested and contact the provider if the dose or frequency is not adequate. Only the client should push the pain button on a PCA pump.

After teaching a client who has been diagnosed with hepatitis A, the nurse assesses the client's understanding. Which statement by the client indicates a correct understanding of the teaching? a. "Some medications have been known to cause hepatitis A." b. "I may have been exposed when we ate shrimp last weekend." c. "I was infected with hepatitis A through a recent blood transfusion." d. "My infection with Epstein-Barr virus can co-infect me with hepatitis A."

ANS: B The route of acquisition of hepatitis A infection is through close personal contact or ingestion of contaminated water or shellfish. Hepatitis A is not transmitted through medications, blood transfusions, or Epstein-Barr virus. Toxic and drug-induced hepatitis is caused from exposure to hepatotoxins, but this is not a form of hepatitis A. Hepatitis B can be spread through blood transfusions. Epstein-Barr virus causes a secondary infection that is not associated with hepatitis A.

3. A client is admitted to the emergency department with toxic shock syndrome. Which action by the nurse is the most important? a. Administer IV fluids to maintain fluid and electrolyte balance. b. Remove the tampon as the source of infection. c. Collect a blood specimen for culture and sensitivity. d. Transfuse the client to manage low blood count.

ANS: B The source of infection should be removed first. All of the other answers are possible interventions depending on the clients symptoms and vital signs, but removing the tampon is the priority.

4. A client is admitted with acute kidney injury (AKI) and a urine output of 2000 mL/day. What is the major concern of the nurse regarding this client's care? a. Edema and pain b. Electrolyte and fluid imbalance c. Cardiac and respiratory status d. Mental health status

ANS: B This client may have an inflammatory cause of AKI with proteins entering the glomerulus and holding the fluid in the filtrate, causing polyuria. Electrolyte loss and fluid balance is essential. Edema and pain are not usually a problem with fluid loss. There could be changes in the client's cardiac, respiratory, and mental health status if the electrolyte imbalance is not treated.

An emergency room nurse assesses a client with potential liver trauma. Which clinical manifestations should alert the nurse to internal bleeding and hypovolemic shock? (Select all that apply.) a. Hypertension b. Tachycardia c. Flushed skin d. Confusion e. Shallow respirations

ANS: B, D Symptoms of hemorrhage and hypovolemic shock include hypotension, tachycardia, tachypnea, pallor, diaphoresis, cool and clammy skin, and confusion.

A nurse assesses a client who has liver disease. Which laboratory findings should the nurse recognize as potentially causing complications of this disorder? (Select all that apply.) a. Elevated aspartate transaminase b. Elevated international normalized ratio (INR) c. Decreased serum globulin levels d. Decreased serum alkaline phosphatase e. Elevated serum ammonia f. Elevated prothrombin time (PT)

ANS: B, E, F Elevated INR and PT are indications of clotting disturbances and alert the nurse to the increased possibility of hemorrhage. Elevated ammonia levels increase the client's confusion. The other values are abnormal and associated with liver disease but do not necessarily place the client at increased risk for complications.

8. A client has a recurrent Bartholin cyst. What is the nurses priority action? a. Apply an ice pack to the area. b. Administer a prophylactic antibiotic. c. Obtain a fluid sample for laboratory analysis. d. Suggest moist heat such as a sitz bath.

ANS: C A major cause of an obstructed duct forming a cyst is infection. The laboratory specimen is a priority since a culture is needed in order to prescribe sensitive antibiotics. Comfort measures can then be used, such as ice packs and moist heat.

A nurse assesses clients at a community health center. Which client is at highest risk for pancreatic cancer? a. A 32-year-old with hypothyroidism b. A 44-year-old with cholelithiasis c. A 50-year-old who has the BRCA2 gene mutation d. A 68-year-old who is of African-American ethnicity

ANS: C Mutations in both the BRCA2 and p16 genes increase the risk for developing pancreatic cancer in a small number of cases. The other factors do not appear to be linked to increased risk.

A nurse cares for a client with acute pancreatitis. The client states, "I am hungry." How should the nurse reply? a. "Is your stomach rumbling or do you have bowel sounds?" b. "I need to check your gag reflex before you can eat." c. "Have you passed any flatus or moved your bowels?" d. "You will not be able to eat until the pain subsides."

ANS: C Paralytic ileus is a common complication of acute pancreatitis. The client should not eat until this has resolved. Bowel sounds and decreased pain are not reliable indicators of peristalsis. Instead, the nurse should assess for passage of flatus or bowel movement.

17. A client is placed on fluid restrictions because of chronic kidney disease (CKD). Which assessment finding would alert the nurse that the client's fluid balance is stable at this time? a. Decreased calcium levels b. Increased phosphorus levels c. No adventitious sounds in the lungs d. Increased edema in the legs

ANS: C The absence of adventitious sounds upon auscultation of the lungs indicates a lack of fluid overload and fluid balance in the client's body. Decreased calcium levels and increased phosphorus levels are common findings with CKD. Edema would indicate a fluid imbalance.

28. A nurse reviews these laboratory values of a client who returned from kidney transplantation 12 hours ago: What initial intervention would the nurse anticipate? a. Start hemodialysis immediately. b. Discuss the need for peritoneal dialysis. c. Increase the dose of immunosuppression. d. Return the client to surgery for exploration.

ANS: C The client may need a higher dose of immunosuppressive medication as evidenced by the elevated BUN and serum creatinine levels. This increased dose may reverse the possible acute rejection of the transplanted kidney. The client does not need hemodialysis, peritoneal dialysis, or further surgery at this point.

A nurse assesses clients on the medical-surgical unit. Which client should the nurse identify as at high risk for pancreatic cancer? a. A 26-year-old with a body mass index of 21 b. A 33-year-old who frequently eats sushi c. A 48-year-old who often drinks wine d. A 66-year-old who smokes cigarettes

ANS: D Risk factors for pancreatic cancer include obesity, older age, high intake of red meat, and cigarette smoking. Sushi and wine intake are not risk factors for pancreatic cancer.

The human papilloma virus (HPV) test may be collected at the same time as the Papanicolaou (Pap) test for screening. Which finding indicates the highest risk for development of cervical cancer? Abnormal Pap results and positive HPV test Normal Pap results and no HPV infection Abnormal Pap results and no HPV infection Normal Pap results and positive HPV results

Abnormal Pap results and positive HPV test If not treated, women with an abnormal Pap results and a positive HPV test have the highest risk for developing cervical cancer.Women who have normal Pap test results and no HPV infection are at the lowest risk for developing cervical cancer.

The nurse is caring for a patient with a 36%Total Body Surface Area (TBSA) burn, who begins desatting into the low 80's on BiPap 15/8, fiO2 of 100%. The nurse suspects the client is experiencing which complication: Hospital Acquired Pneumonia (HAP) Loss of diaphramatic excursion Sleep Apnea Acute Respiratory Distress Syndrome (ARDS)

Acute Respiratory Distress Syndrome (ARDS)

A client is diagnosed with bladder cancer, and a cystectomy and an ileal conduit are scheduled. What should the nurse plan to do pre-operatively? Teach range-of-motion and Kegel exercises Administer cleansing enemas and laxatives as ordered Explain the procedure for irrigating the ileal conduit Maintain intravenous TPN at 100 mL/hour to improve nutritional status

Administer cleansing enemas and laxatives as ordered Preoperative cleansing of the bowel is necessary before surgical resection and formation of a urinary conduit. Fluids should not be restricted until after midnight or based on the specific prescriptions of the primary healthcare provider. Range-of-motion and Kegel exercises have no direct effect on this procedure. An ileal conduit is not irrigated.

A client is experiencing an attack of acute pancreatitis. Which nursing intervention is the highestpriority for this client? Assist the client to assume a position of comfort. Administer opioid analgesic medication. Do not administer food or fluids by mouth. Measure intake and output every shift.

Administer opioid analgesic medication. Pain relief is the highest priority for the client with acute pancreatitis.Although measuring intake and output, NPO status, and positioning for comfort are all important, they are not the highest priority.

Which serum laboratory values will the nurse expect to be elevated in a client who has acute pancreatitis?

Amylase Bilirubin Lipase Glucose

The student nurse studying shock understands that the common manifestations of this condition are directly related to which problems? Increased peripheral perfusion Anaerobic metabolism Renal perfusion impairment Hypotension Hyperglycemia

Anaerobic metabolism Hypotension

How will the nurse interpret a client's laboratory finding of the presence of immunoglobulin G antibodies directed against hepatitis A (HAV)? A. Active, infectious HAV is present. B. Permanent immunity to HAV is present. C. This is the client's first infection to HAV. D. The risk for infection if exposed to HAV is high.

B. Permanent immunity to HAV is present.

What is the nurse's best response to a client who fears he may have been exposed to hepatitis A while attending a banquet last week after which three restaurant workers were diagnosed with hepatitis A? A. "Which types of food did you eat at the banquet?" B. "If you have no symptoms at this time, you are probably safe." C. "You can receive an immunoglobulin injection to prevent the infection." D. "Contact your primary health care provider about receiving the hepatitis A vaccine."

C. "You can receive an immunoglobulin injection to prevent the infection."

What is the nurse's best first action when a client who just had a liver transplant develops oozing around two IV sites as well as has some new bruising? A. Applying pressure to the IV sites B. Checking the client's platelet levels C. Notifying the surgeon immediately D. Documenting the findings as the only action

C. Notifying the surgeon immediately

Which priority action will the nurse implement when an older male client reports diffi-culty starting the urine stream? A. Teach the importance of testicular self-exam (TSE). B. Discuss exercises to strengthen pelvic floor muscles. C. Teach signs of urethral obstruction and importance of prostate cancer screening. D. Provide information on testosterone functions and supplements

C. Teach signs of urethral obstruction and importance of prostate cancer screening.

A client with newly diagnosed gynecologic cancer is being discharged home. Which health care team member does the nurse contact to coordinate nursing care at home for this client? Case manager Hospice Social services Primary health care provider

Case manager If nursing care is needed at home, the hospital nurse or case manager makes referrals to a home health care agency.The primary health care provider is not the correct team member to coordinate home care. Hospice care is provided for clients who are at the end of life. This type of care is not necessary (or indicated) for this client. A referral to a social service agency is needed if the client is unable to meet the financial demands of treatment and long-term care follow-up.

61. The school nurse will teach in the high school health class that young women who have multiple sex partners are at high risk for which disease? Ovarian cancer Cervical cancer Amenorrhea Endometriosis

Cervical cancer

Which of the following lab profiles is consistent with a diagnosis of acute pancreatitis? WBC = 12,500 mm3 Platelets = 140,000 C-Reactive Protein = 2.1 mg/dL Creatinine = 1.7 mg/dL

Creatinine = 1.7 mg/dL

Which essential nutrient will the nurse expect to be deficient in a client who has liver cirrhosis and ascites? A. Sodium B. Potassium C. Vitamin C D. Vitamin K

D. Vitamin K

Of the following laboratory values, which of the following is most likely to be associated with leimyomas? Elevated CA-125 level Elevated White Blood Cell Count Decreased Hemoglobin Elevated HCG level

Decreased Hemoglobin

A young adult with testicular cancer is admitted for unilateral orchiectomy and retroperitoneal lymph node dissection. Which task will the nurse delegate to assistive personnel (AP)? Discuss reproductive options with the client and significant other. Evaluate the client's understanding of chemotherapy and radiation treatment. Encourage the client to cough and deep-breathe after surgery. Teach about the availability of a gel-filled silicone testicular prosthesis.

Encourage the client to cough and deep-breathe after surgery. Reminding clients to perform coughing and deep-breathing activities can be delegated to APs.Client education and evaluation are more complex skills that must be done by licensed nurses.

A nurse assesses a client who is receiving Cyclosporine after receiving a kidney transplant. Which of these client findings should the nurse follow up on immediately as it relates to this medication? Hypotension Hyperkalemia Bradycardia Fever

Fever

A client has burns on both legs. These areas appear yellow, dry and hard. No blisters or bleeding are present, and client complains of little to no pain. How does the nurse classify this injury? Partial thickness deep Full thickness Superficial Partial thickness superficial

Full thickness The characteristics of the wound meet the criteria for a full-thickness injury (color that is black, brown, yellow, white or red; no blisters; pain minimal; outer layer firm and inelastic). A superficial burn is bright red and moist, and might appear glistening with blister formation. The healing time for this type of wound is within 21 days. A full thickness burn involves all layers of the skin and may extend into the underlying tissue. These burns take many weeks to heal. Stating that wound healing is individualized does not answer the patient's question about the severity of the burn.

Which change in electrolyte values will the nurse expect in a client with acute pancreatitis who reports numbness around the mouth and leg muscle twitching?

Hypocalcemia

The nurse is caring for a client with kidney failure. Which assessment data indicates the need for increased fluids? Decreased sodium level Pale-colored urine Increased blood urea nitrogen (BUN) Increased creatinine level

Increased blood urea nitrogen (BUN) An increase in BUN can be an indication of dehydration, and a needed increase in fluids.Increased creatinine indicates kidney impairment. Urine that is pale in color is diluted and does not indicate that an increase in fluids is necessary. Sodium is increased, not decreased, with dehydration.

Which signs and symptoms will the nurse expect to see in a client who is diagnosed with advanced pancreatic cancer?

Light-colored urine and dark-colored stools Anorexia and weight loss Splenomegaly Ascites Leg or calf pain Weakness and fatigue

28. The nurse suspects that the client may be developing ARDS. Which assessment data confirms the diagnosis of ARDS? The client presents with dyspnea, tachycardia, and complains of feeling anxious Presence of crackles and rhonchi on-half way up the posterier lung fields Low arterial oxygen when administering high concentration of oxygen The client has jugular vein distention and frothy sputum

Low arterial oxygen when administering high concentration of oxygen The classic signs of ARDS is decreased arterial oxygen level (PaO2) while administering high levels of oxygen; the oxygen is unable to cross the alveolar membrane.

Which actions will the nurse take to help relieve the severe pain in a client with acute pancreatitis?

Maintaining the client on NPO status Providing opioids by patient-controlled analgesia Assisting the client to aside-lying position with knees drawn up to the chest

The nurse teaches a client who is recovering from acute kidney injury to avoid which type of medication? Opioids Nonsteroidal anti-inflammatory drugs (NSAIDs) Calcium channel blockers Angiotensin-converting enzyme (ACE) inhibitors

Nonsteroidal anti-inflammatory drugs (NSAIDs) Clients recovering from acute kidney disease need to be taught to avoid NSAIDs. NSAIDs may be nephrotoxic to a client with acute kidney disease and must be avoided.ACE inhibitors are used for treatment of hypertension and to protect the kidneys, especially in the diabetic client, from progression of kidney disease. Opioids may be used by clients with kidney disease if severe pain is present. Excretion, however, may be delayed. Calcium channel blockers can improve the glomerular filtration rate and blood flow within the kidney.

A client who had a Whipple surgical procedure develops an internal fistula between the pancreas and stomach. For which complication would the nurse monitor? Cirrhosis Crohn disease Peritonitis Peptic ulcer disease

Peritonitis Leakage of pancreatic enzymes, bile, and/or gastric secretions into the abdomen (peritoneal cavity) often causes peritonitis, which requires IV antibiotic therapy to manage.

A client is admitted to the emergency department with possible acute pancreatitis. What is the nurse's priority assessment at this time? Respiratory assessment Cardiovascular assessment Abdominal assessment Pain intensity assessment

Respiratory assessment As for any client, the nurse would want to continually assess for airway, breathing, and circulation. Clients who have acute pancreatitis often develop pleural infusions, atelectasis, or pneumonia. Necrotizing hemorrhagic pancreatitis places the client at risk for acute respiratory distress syndrome (ARDS).

The nurse will teach the client who is to self-administer erythropoetin (EPO) at home to: Routinely supplement the medication with iron Monitor for drug side effects, such as rash and diarrhea Take the medication with a full glass of water Have weekly PT/INR tests done

Routinely supplement the medication with iron

In which position will the nurse place a client after an open Whipple procedure for treatment of pancreatic cancer?

Semi-Fowler position to reduce tension on the suture line

A nurse reviews the laboratory results for a patient who was burned 24 hours ago. Which laboratory result would the nurse report to the health care provider immediately? Hematocrit: 52% Arterial pH: 7.32 Serum sodium: 130 mEq/L Serum potassium: 6.5 mEq/L

Serum potassium: 6.5 mEq/L

A nurse is assessing a client during the first 24 hours after a burn injury. Which sign indicates that fluid replacement therapy is adequate? Urinary output of 15-20 mL/hour Hematocrit level increasing from 50% to 55% Decreasing CVP readings Slowing of a previously rapid pulse

Slowing of a previously rapid pulse

When caring for a client who receives peritoneal dialysis (PD), which finding does the nurse report to the provider (HCP) immediately? Temperature of 101.2° F (38.4° C) Sinus bradycardia, rate of 58 beats/min Pulse oximetry reading of 95% Blood pressure of 148/90 mm Hg

Temperature of 101.2° F (38.4° C) The nurse needs to immediately report a peritoneal dialysis client's temperature of 101.2° F (38.4° C) to the HCP. Peritonitis is the major complication of PD, caused by intra-abdominal catheter site contamination. Meticulous aseptic technique must be used when caring for PD equipment.A pulse oximetry reading of 95% is a normal saturation. Although a heart rate of 58 beats/min is slightly bradycardic, the HCP can be informed upon visiting the client. Clients with kidney failure tend to have slightly higher blood pressures due to fluid retention. This is not as serious as a fever.

A 32-year-old client has small uterine fibroids and is considering options for treatment. To assist the woman to make a decision about whether to have magnetic resonance-guided focused ultrasound or uterine artery embolization, what will the nurse determine? If there is a history of abortion in the past Client's risk for uterine cancer Whether the client desires to have children Age of onset of first menstrual period

Whether the client desires to have children If the woman wants to preserve her fertility, magnetic resonance-guided focused ultrasound is a good option. If the client does not desire pregnancy, then uterine artery embolization may be used.The woman's previous reproductive history, the age of menarche, and the risk for uterine cancer do not influence the decision about which of these two types of procedures are used.

The nurse is caring for a client who has been diagnosed with cirrhosis. Which laboratory result(s) would the nurse expect for this client? (Select all that apply.) a) Increased serum bilirubin b) Increased lactate dehydrogenase c) Decreased serum albumin d) Increased serum alanine aminotransferase e) Increased aspartate aminotransferase f) Increased serum ammonia

a) Increased serum bilirubin b) Increased lactate dehydrogenase c) Decreased serum albumin d) Increased serum alanine aminotransferase e) Increased aspartate aminotransferase f) Increased serum ammonia Cirrhosis is a chronic disease in which the liver progressively degenerates. As a result, liver enzymes and bilirubin increase. Additionally, the liver is unable to synthesize protein leading to decreased serum albumin. Elevated serum ammonia results from the inability of the liver to detoxify protein by-products.

Which criteria will the nurse assess in a client with benign prostatic hyperplasia that indicate the need for a surgical treatment? SATA a. hydronephrosis b. acute urinary tract infection unresponsive to first-line antibiotics c. hematuria d. chronic urinary tract infection secondary to residual urine in bladder e. recurrent kidney stones f. acute urinary retention due to obstruction

a. hydronephrosis c. hematuria d. chronic urinary tract infection secondary to residual urine in bladder f. acute urinary retention due to obstruction

Which technique is best when the nurse assess an obese client who reports symptoms associated with benign prostatic hyperplasia? a. instructing the client to urinate, then using the bedside ultrasound bladder scanner b. applying gentle pressure to the bladder to elicit urgency, then instructing the client to void c. having the client drink several large glasses of water, then percussing the bladder d. instructing the client to undress from the waist down, then inspecting and palpating the bladder

a. instructing the client to urinate, then using the bedside ultrasound bladder scanner

To which common sites does the nurse expect metastasis when a client has prostate cancer? SATA a. liver b. pancreas c. lungs d. lumbar spine e. kidneys f. bones of the pelvis

a. liver c. lungs d. lumbar spine f. bones of the pelvis

4. The nurse is teaching a client with benign prostatic hyperplasia (BPH). What statement indicates a lack of understanding by the client? a.There should be no problem with a glass of wine with dinner each night. b.I am so glad that I weaned myself off of coffee about a year ago. c.I need to inform my allergist that I cannot take my normal decongestant. d.My normal routine of drinking a quart of water during exercise needs to change.

a.There should be no problem with a glass of wine with dinner each night. This client did not associate wine with the avoidance of alcohol, and requires additional teaching. The nurse must teach a client with BPH to avoid alcohol, caffeine, and large quantities of fluid in a short amount of time to prevent overdistention of the bladder. Decongestants also need to be avoided to lower the chance for urinary retention.

18. A 52-year old diagnosed with cirrhosis presents to the ED, vomiting dark brown emesis, and is very anxious and restless. The ED nurse realizes that the first priority should be to: administer Zofran for vomiting assess for signs of hypovolemia test the emesis and stool for blood insert an NG tube and initiate irrigation with iced normal saline

assess for signs of hypovolemia

8. The nurse is administering sulfamethoxazole-trimethoprim (Bactrim) to a client diagnosed with bacterial prostatitis. Which finding causes the nurse to question this medication for this client? a. Urinary tract infection b. Allergy to sulfa medications c. Hematuria d. Elevated serum white blood cells

b. Allergy to sulfa medications Before administering sulfamethoxazole-trimethoprim, the nurse must assess if the client is allergic to sulfa drugs. Urinary tract infection, hematuria, and elevated serum white blood cells are common problems associated with bacterial prostatitis that require long-term antibiotic therapy.

A 72-year-old woman is being assessed by the nurse for an annual physical. Which finding is of concern to the nurse? a. Thinning of pubic hair b. Increased size of the uterus c. Decreased size of the clitoris d. Loss of tone of the pelvic ligaments

b. Increased size of the uterus An increased size of the uterus is an abnormal finding and should be assessed further. Normal changes in the reproductive system related to aging include the graying and thinning of pubic hair, decreased size of the labia majora and clitoris, and loss of tone and elasticity of the pelvic ligaments and connective tissue. The uterus would normally be decreased, not increased, in size due to changes in hormonal levels and atrophy.

What is the nurse's best action when assessing a client after open radical prostatectomy and finding scrotal and penile swelling? a. notify the health care provider and monitor for inability to urinate b. elevate the scrotum and penis, then apply ice to the area intermittently c. assist the client to increase mobility by using early ambulation d. observe the urethral meatus for redness, discharge, and abnormal output

b. elevate the scrotum and penis, then apply ice to the area intermittently

Which symptom does the nurse most likely expect when admitting a client diagnosed with benign prostatic hyperplasia (BPH)? a. erectile dysfunction b. pain in the scrotum c. difficulty passing urine d. constipation

c. difficulty passing urine

Which laboratory test suggests to the nurse that a client with prostate cancer has metastasis to the bone? a. decreased alpha-fetoprotein b. increased blood urea nitrogen (BUN) c. elevated serum alkaline phosphatase d. decreased serum creatinine

c. elevated serum alkaline phosphatase

15. The nurse receives report from that dialysis nurse that the patient has signs of dialysis disequilibrium. The nurse interprets this to mean that the patient has: had significant blood loss during dialysis developed muscle cramping because of the rapid fluid and electrolyte shifts from the extracellular space had cerebral fluid shifts related to the dialysis developed cardiac dysrhythmias during dialysis which caused decreased cardiac output

had cerebral fluid shifts related to the dialysis ??? Dialysis disequilibrium syndrome (DDS) is a condition where solutes are removed too quickly from the blood, causing brain cells to swell with fluid. This results in increased intracranial pressure (ICP) - possibly leading to coma and death.

The nurse is assessing a client who has septic shock. The following assessment data were collected: (Photo should be here) The nurse correlates these findings with which stage of shock? Progressive Non-Progressive Refractory Initial

initial

Which of the following ABG results would the nurse expect in the client who is in the oliguric stage of acute kidney injury? pH = 7.45; PaCO2 = 52; HCO3 = 27; PaO2 = 88 pH = 7.32; PaCO2 = 31; HCO3 = 19; PaO2 = 90 pH = 7.33; PaCO2 = 36; HCO3 = 29; PaO2 = 75 pH = 7.31; PaCO2 = 48; HCO3 = 26; PaO2 = 90

pH = 7.32; PaCO2 = 31; HCO3 = 19; PaO2 = 90

8. Based on the Kidney Disease: Improving Global Outcomes classification (KDIGO), how will the nurse interpret this client data (serum creatinine increases 1.5 times over baseline with urine output of less than 0.5 mL/kg/hr for 6 hours or longer)? A. Stage 1 B. Stage 2 C. Stage 3 D. End-stage kidney disease

✅ A Based on the Kidney Disease: Improving Global Outcomes classification (KDIGO), a value of serum creatinine increases 1.5 times over baseline with urine output of less than 0.5 mL/kg/hr for 6 hours or longer indicates stage 1. See Table 63.2 in your text for information on the other stages for this system.

3. When prerenal and postrenal causes of acute kidney injury occur, how does the nurse expect a client's kidneys to compensate? Select all that apply. A. Constricting of blood vessels in the kidneys B. Restricting of secretion of glucocorticoids C. Releasing antidiuretic hormone (ADH) D. Crushing then passing fragments of kidney stones E. Dilating of peripheral arteries throughout the body F. Activating the renin-angiotensin-aldosterone pathway

✅ A, C, F When prerenal or postrenal causes of AKI occur, the kidneys compensate by three responses: constricting kidney blood vessels, activating the renin-angiotensin-aldosterone pathway, and releasing antidiuretic hormone (ADH)

2. For which causes will the nurse monitor clients for development of intrarenal (intrinsic) acute kidney injury (AKI)? Select all that apply. A. Glomerulonephritis B. Bladder cancer C. Exposure to nephrotoxins D. Embolism in renal blood vessels E. Severe dehydration F. Kidney stones

✅. A, C, D Examples of disorders causing intrinsic renal AKI include allergic disorders, embolism or thrombosis of the renal vessels, and nephrotoxic agents. Severe dehydration causes prerenal failure. Bladder cancer and kidney stones cause postrenal failure. For additional causes, see Table 63.4 Diseases and Conditions That Contribute to Acute Kidney Injury in your text.

7. Which laboratory results will the nurse monitor when a client is receiving IV gentamicin? Select all that apply. A. Platelet count B. Hemoglobin and hematocrit C. Blood urea nitrogen (BUN) D. Prothrombin time E. Creatinine F. Gentamicin peak and trough levels

✅C, E, F If a client is receiving a known nephrotoxic drug, the nurse will closely monitor laboratory values, including BUN, creatinine, and drug (gentamicin) peak and trough levels, for indications of reduced kidney function.

11. Which client does the nurse understand has the greatest risk of developing acute kidney injury (AKI)? A. 23-year-old female who was recently treated for a urinary tract infection B. 32-year-old female who is pregnant and has gestational diabetes C. 49-year-old male who is obese and has a history of skin cancer D. 73-year-old male who has hypertension and peripheral vascular disease

✅D Risk factors for AKI include shock, cardiac surgery, hypotension, prolonged mechanical ventilation, and sepsis. Older adults or adults with diabetes, hypertension, peripheral vascular disease, liver disease, or CKD are at higher risk of AKI if hospitalized. The client in option D is an older adult with two important risk factors. Thus, he is at highest risk of AKI development

22. Which laboratory result will the nurse expect when a client with chronic kidney disease reports fatigue, lethargy with weakness, and mild shortness of breath with dizziness when rising to a standing position? A. Low blood glucose B. Low white blood cell count C. Low blood urea nitrogen (BUN) D. Low hemoglobin/hematocrit

✅D The client's symptoms suggest anemia which is common in clients in the later stages of CKD and makes symptoms worse. The causes of anemia include a decreased production of erythropoietin by the kidneys which causes reduced red blood cell (RBC) production and low hemoglobin and hematocrit levels.

After receiving change-of-shift report on these clients, which client does the nurse plan to assess first? Older adult client who is receiving total parenteral nutrition after a Whipple procedure and has a glucose level of 235 mg/dL (13.1 mmol/L). Adult client admitted with cholecystitis who is experiencing severe right upper quadrant abdominal pain. Young adult client with acute pancreatitis who is dyspneic and has a respiratory rate of 34 to 38 breaths/min. Middle-age client who has an elevated temperature after undergoing endoscopic retrograde cholangiopancreatography.

Young adult client with acute pancreatitis who is dyspneic and has a respiratory rate of 34 to 38 breaths/min. The nurse would first assess the young adult client with acute pancreatitis who is dyspneic and has a respiratory rate of 34 to 38 breaths/min. Airway and breathing are the two most important criteria the nurse will use to determine which client to assess first. The dyspneic client is at greatest risk for rapid deterioration and requires immediate assessment and intervention. Acute respiratory distress syndrome is a possible complication of acute pancreatitis.The client with cholecystitis and the client with an elevated temperature will require further assessment and intervention, but these are not medical emergencies requiring the nurse's immediate attention. The older adult client's glucose level will require intervention but, again, is not a medical emergency.

20. For which emergency procedure does the nurse prepare when a client with chronic kidney disease develops chest pain, tachycardia, low-grade fever, friction rub, and muffled heart tones? A. Hemodialysis B. Removal of pericardial fluid C. Cardioversion D. Endotracheal intubation

✅B The client's signs and symptoms suggest pericarditis which often occurs in CKD and can cause tamponade. Treatment of tamponade, which is a medical emergency, requires immediate removal of pericardial fluid (pericardiocentesis) by placement of a needle, catheter, or drainage tube into the pericardium.

21. Which specific drug therapy will the nurse anticipate for management of the client who has septic shock? A. Antibiotics B. Inotropics C. Crystalloids D. Antidysrhythmics

21. A. Antibiotics Septic shock starts with a localized infection that becomes systemic. Specific therapy for sepsis and septic shock (Sepsis-3) is the use of antibiotics. Inotropics and crystalloids are used for many types of shock, including septic shock. Antidysrhythmics are used whenever life-threatening dysrhythmias are present.

10. Which statement by a client to the nurse indicates that her anterior colporrhaphy has achieved the desired therapeutic outcome? A. "That constipated feeling has resolved." B. "The abdominal pain is almost gone." C. "I have good control over my urination." D. "My vaginal bleeding has completely stopped."

C. "I have good control over my urination."

Which condition does the nurse expect when a female client reports altered nutritional intake resulting in changes in metabolism? A. Excessive bleeding B. Pelvic inflammatory disease C. Endometriosis D. Amenorrhea

D. Amenorrhea

What is a characteristic the nurse might see in a patient with chronic pancreatitis? Oliguia Jaundice Weight gain constipation

Jaundice

The nurse teaches the client who has cirrhosis about foods and other substances that should be avoided to prevent worsening of the disease. Which substance(s) will the nurse include in that health teaching? (Select all that apply.) a) Smoking b) Alcohol c) Illicit drugs d) Acetaminophen e) Sodium f) Protein

a) Smoking b) Alcohol c) Illicit drugs d) Acetaminophen Protein and sodium should be moderately restricted but not completely avoided. The other substances can worsen the disease process, especially drugs and alcohol which are normally metabolized by the liver.

1. A nurse is caring for a client who suffered massive blood loss after trauma. How does the nurse correlate the blood loss with the client's mean arterial pressure (MAP)? a. It causes vasoconstriction and increased MAP. b. Lower blood volume lowers MAP. c. There is no direct correlation to MAP. d. It raises cardiac output and MAP.

b. Lower blood volume lowers MAP. Lower blood volume will decrease MAP. The other answers are not accurate.

Which laboratory tests does the nurse expect to be ordered to screen for prostate cancer in a client with benign prostatic hyperplasia (BPH)? SATA a. urinalysis and urine culture b. complete blood count (CBC) c. prostate-specific antigen (PSA) d. blood urea nitrogen (BUN) e. serum acid phosphatase f. serum creatinine

c. prostate-specific antigen (PSA) e. serum acid phosphatase

29. The clinic nurse is caring for a client who has been diagnosed with uterine cancer stage III. The client asks for a further explanation of the disease. The nurse will base her response on the understanding that: the cancer has metastasized to the lymph nodes the cancer has metastasized to the endometrium the cancer has metastasized to the cervix the cancer has metastasized to the intestines

the cancer has metastasized to the lymph nodes

A client with advanced liver disease is refusing to take lactulose because of diarrhea. Which is the nurse's best response to this client? "You may take Kaopectate liquid daily for loose stools." "We will need to send a stool specimen to the laboratory." "Diarrhea is expected; that's how your body gets rid of ammonia." "Do not take any more of the medication until your stools firm up."

"Diarrhea is expected; that's how your body gets rid of ammonia."

A client with erectile dysfunction has been prescribed a PDE-5 inhibitor. Which client statement demonstrates that nursing teaching about the medication has been effective? "I have to be sure to be careful when I sit up or stand up." "I can drink a little bit of alcohol before intercourse to help me out." "I will have my partner take this medication about 30 minutes before intercourse." "I need to increase fiber in my diet to avoid constipation."

"I have to be sure to be careful when I sit up or stand up." A side effect of PDF-5 inhibitors is vasodilation, which can cause profound hypotension. The client who states he needs to be careful when sitting or standing up has demonstrated understanding of this concern.Alcohol should be avoided when using a PDE-5 inhibitor. The medication is prescribed for the client; not the client's partner. PDE-5 drugs are not generally associated with constipation.

The nurse is teaching a preoperative client who is scheduled for a laparoscopic cholecystectomy ("lap chole"). What statement by the client indicates a need for further teaching? "I will likely need oral pain medications for the first few days after my surgery." "I should only be hospitalized for 2 to 3 days after my surgery." "I will probably not be at risk for complications from this surgery." "I should be able to go back to work in the next week or so."

"I should only be hospitalized for 2 to 3 days after my surgery." A "lap chole" surgery has many advantages over the open traditional surgical method, including a short hospital stay, usually same-day surgery, minimal risk for complications, and the ability to achieve pain control by using oral analgesics.

The nurse is educating a group of young men about testicular self-examination (TSE). Which statement by a member of the group indicates teaching has been effective? "I will squeeze each testicle in my hand to feel any abnormalities." "I will examine my testicles right before taking a shower." "I will look and feel for any lumps or changes to my testes." "I only need to report large masses to my primary health care provider."

"I will look and feel for any lumps or changes to my testes." In TSE, the client would look and feel for any lumps or changes to the testes. Any lumps that are detected would be immediately reported. With early detection by monthly TSE and treatment, testicular cancer can usually be successfully cured.A TSE would be performed immediately following a shower. The client would gently roll each testicle between the thumb and forefinger rather than squeeze them. All masses must be reported to the primary health care provider, no matter the size.

A client is scheduled to undergo kidney transplant surgery. Which teaching point does the nurse include in the preoperative teaching session? "It is essential for you to wash your hands and avoid people who are ill." "The new kidney will be placed directly below one of your old kidneys." "You will receive dialysis the day before surgery and for about a week after." "Your diseased kidney will be removed when the transplant is performed."

"It is essential for you to wash your hands and avoid people who are ill." Teaching the client to wash hands and stay away from sick people are important points for the nurse to include in teaching for a client scheduled for a kidney transplant. Antirejection medications increase the risks for infection, sepsis, and death. Strict aseptic technique and handwashing are essential.Unless severely infected, the client's kidneys are left in place and the graft is placed in the iliac fossa. Dialysis is performed the day before surgery. After the surgery, the new kidney should begin to make urine.

A client asks the nurse, "What are the advantages of ambulatory peritoneal dialysis over hemodialysis?" Which responses by the nurse are accurate? (Select all that apply.) a. "It will give you greater freedom in your scheduling." b. "You have less chance of getting an infection." c. "You only need to do it three times/week." d. "You do not need a machine to do it." e. "You will have fewer dietary restrictions."

"It will give you greater freedom in your scheduling." "You do not need a machine to do it." "You will have fewer dietary restrictions."

Which statement by a client with cirrhosis indicates that further instruction is needed about the disease? "Cirrhosis is a chronic disease that has scarred my liver." "The scars on my liver create problems with blood circulation." "Because of the scars on my liver, blood clotting and blood pressure are affected." "My liver is scarred, but the cells can regenerate themselves and repair the damage."

"My liver is scarred, but the cells can regenerate themselves and repair the damage." The client's statement that, although his liver is scarred, the cells can regenerate and repair the damage indicates that further instruction is needed. Although cells and tissues will attempt to regenerate, destroyed liver cells will result in permanent scarring and irreparable damage.

A client with advanced liver disease is refusing to take lactulose because of diarrhea. Which is the nurse's best response to this client? "Do not take any more of the medication until your stools firm up." "You may take Kaopectate liquid daily for loose stools." "That's how your body gets rid of ammonia." "We will need to send a stool specimen to the laboratory."

"That's how your body gets rid of ammonia."

Discharge teaching has been provided for a client recovering from kidney transplantation. Which client statement indicates understanding of the teaching? "I will drink 8 ounces (236 mL) of water with my medications." "I can stop my medications when my kidney function returns to normal." "If my urine output is decreased, I should increase my fluids." "The antirejection medications will be taken for life."

"The antirejection medications will be taken for life." When the client states that antirejection medications must be taken for life, it indicates that the kidney transplant client understands the discharge teaching. Immune-suppressant therapy must be taken for life to prevent organ rejection. Adherence to immunosuppressive drugs is crucial to survival for clients with transplanted kidneys.Lack of adherence can lead to complications such as rejection, graft loss, return to dialysis, and death. Oliguria (decreased urine output) is a symptom of transplant rejection. If this occurs, the transplant team must be contacted immediately. It is not necessary to take antirejection medication with 8 ounces (236 mL) of water.

A client who had an anterior colporrhaphy is being discharged. What does the nurse tell the client before her discharge? "Use a pillow to splint your abdomen." "Avoid lifting more than 25 lb (11.3 kg)." "Return to the clinic in 6 weeks for suture removal." "Do not have sexual intercourse for at least 2 weeks."

"Use a pillow to splint your abdomen." The client needs to be taught to use a pillow to splint the abdomen, which protects sutures.The client must avoid lifting anything heavier than 5 lb (2.27 kg) and avoid sexual intercourse for 6 weeks. Sutures do not need to be removed because they may be absorbable or they may fall out (slough off) as healing occurs.

The nurse is reviewing the prescription for a client admitted to the hospital with a diagnosis of acute pancreatitis. Which interventions would the nurse expect to be prescribed for the client? Select all that apply. 1. Maintain NPO status 2. Encourage coughing and deep breathing 3. Give small frequent high calorie feedings 4. Maintain the client in a supine and flat position 5. Give hydromorphone IV as prescribed for pain 6. Maintain IV fluids at 10 mL/hour to keep the vein open

1. Maintain NPO status 2. Encourage coughing and deep breathing 5. Give hydromorphone IV as prescribed for pain

The nurse is assessing a client who is experiencing an acute episode of cholecystitis. Which of these clinical manifestations support this diagnosis? Select all that apply. 1. fever 2. positive Cullen's sign 3. complaints of indigestion 4. palpable mass in the left upper quadrant 5. pain in the upper right quadrant after a fatty meal 6. vague lower right quadrant abdominal discomfort

1. fever 3. complaints of indigestion 5. pain in the upper right quadrant after a fatty meal

Which clinical ndings are consistent with sepsis diagnostic criteria? Select all that apply. 1. Urine output 50 mL/hr 2. Hypoactive bowel sounds 3. Temperature of 102° F (38.9° C) 4. Heart rate of 96 beats per minute 5. Mean arterial pressure 65 mm Hg 6. Systolic blood pressure 110 mm

14. Answer: 3, 4, 5Rationale: Sepsis diagnostic criteria with regard to signs and symptoms include the following: Fever (temperature higher than 100.9° F [38.3° C]) or hypothermia (core temperature lower than 97° F [36° C]), heart rate above 90 beats per minute, tachypnea (respiratory rate above 22 breaths per minute), systolic blood pressure (SBP) less than or equal to 100 mm Hg or arterial hypotension (SBP below 90 mm Hg), MAP of less than 70 mm Hg, or a decrease in SBP of more than 40 mm Hg, altered mental status, edema or positive fluid balance, oliguria, ileus (absent bowel sounds), and decreased capillary refill or mottling of skin. Test-Taking Strategy: Note the subject, clinical findings con- sistent with sepsis diagnostic criteria. Recalling that a minimum of 30 mL/hr of urine is adequate will assist in eliminating this option. Noting that hypoactive bowel sounds are not suggestive of an ileus eliminates this option. Lastly, recalling that an SBP of 110 mm Hg does not fit the criteria will assist in eliminating this option.

18. Which newly admitted client does the nurse consider to be at highest risk for development of sepsis? A. 75-year-old with hypertension and early Alzheimer disease B. 68-year-old who is 2 days postoperative from bowel surgery C. 54-year-old with moderate asthma and severe degenerative joint disease of the right knee D. 80-year-old community dweller with no other health problems undergoing cataract surgery

18. B This client has several risk factors. First, he or she is an older adult. Immune function decreases with age. The greatest risk factor is the recent bowel surgery. Not only does major surgery further reduce the immune response but also the bowel cannot be "sterilized" for surgery. Thus, bacteria in the bowel have the chance to escape the site and enter the bloodstream when the bowel is disrupted.

A client is diagnosed with viral hepatitis, complaining of "no appetite" and "losing my taste for food". What instruction should the nurse give the client to provide adequate nutrition? 1. select foods high in fat 2. increase intake of fluids, including juices 3. eat a good super when anorexia is not as severe 4. eat less often, preferably only 3 large meals daily

2. increase intake of fluids, including juices

28. Which changes in condition will the nurse teach a client who is being discharged after successful management of sepsis to check daily as an indicator of a new or ongoing infection? Select all that apply. A. Shortness of breath B. Temperature elevation C. Cloudy, foul-smelling urine D. New onset of a productive cough E. Paleness or blue tinge to mouth membranes F. Redness and tenderness of an open skin lesion

28. A, B, C, D, E, F All conditions listed above can be indicators of a new or ongoing infection.

13. Which statement made by a client at high risk for hypovolemic shock is of greatest concern to the nurse? A. "I live alone in my house and my family lives in a different state." B. "Do you have any idea when I might go home? No one is feeding my cat." C. "Something feels wrong, but I'm not sure what is causing me to feel this way." D. "I would usually go golfing with my friends today. I hope they're not worried about me."

3. C. "Something feels wrong, but I'm not sure what is causing me to feel this way." Anxiety and a sense of impending doom are common changes in mental status that occur with the hypoxemia and sympathetic nervous system associated with shock

he nurse is assessing a client with epididymitis. The nurse anticipates which findings on physical examination? 1. fever, diarrhea, groin pain, and ecchymosis 2. nausea, painful scrotal edema, and ecchymosis 3. fever, nausea, vomiting, and painful scrotal edema 4. diarrhea, groin pain, testicular torsion, and scrotal edema

3. fever, nausea, vomiting, and painful scrotal edema Rationale: typical signs and symptoms of epididymitis include scrotal pain and edema, which often are accompanied by fever, nausea and vomiting, and chills. Epididymitis most often is caused by infection, although sometimes it can be caused by trauma. The remaining options do not present all of the accurate manifestations

The nurse is caring for a client following a gastrojejunostomy (Billroth II procedure). Which postoperative prescription should the nurse question and verify? 1. leg exercises 2. early ambulation 3. irrigating the NG tube 4. coughing and deep-breathing exercises

3. irrigating the NG tube

A medical surgical nurse is concerned about which of the following patients at highest risk of developing cholecystitis: 89-year-old Caucasian male that takes metformin on daily basis that has forgotten to take his fish oil and multivitamin for the last week. 21-year-old Caucasian male with obesity who has been consuming alcohol and has shown marked stress from work and takes pantoprazole. 45-year-old African American female who just became pregnant and used to take birth control medication, recently started on prenatal vitamins. 70-year-old Native American Female with Crohn's disease and has been taking Lantus and lovastatin on time.

70-year-old Native American Female with Crohn's disease and has been taking Lantus and lovastatin on time.

8. Which body area on a client with darker skin who is at high risk for shock will the nurse examine for indications of pallor and cyanosis? A. Oral mucous membranes B. Soles of the hands and feet C. Earlobes and bridge of the nose D. Sclera closest to the inner corner of the eye

8. A. Oral mucous membranes In dark-skinned clients, pallor or cyanosis is best assessed in the oral mucous membranes. The hands and feet may indicate a temperature change but are not reliable indicators of pallor or cyanosis. Sclera can only indicate the possible presence of jaundice. Although the earlobes and nose can become cyanotic, this would occur much later than changes observed in the oral mucous membranes.

78. A patient weighing 68 kg has a 50% total body surface area (BSA) burn. Using the Parkland formula for fluid replacement, how many mLs/hr should the pump be set for in the first 8 hours? (Answer must be numerical value only)

850

Which client will the nurse prioritize when teaching about prostate cancer screening? A 50-year-old African American A 22-year-old Indian American A 30-year-old Hispanic American A 45-year-old Native American

A 50-year-old African American In the United States, prostate cancer affects African-American men the most and at an earlier age. African-American men and men who have a first-degree relative with prostate cancer have a higher risk for prostate cancer.

Which client has a risk for hypovolemic shock? A client with esophageal varices A client with kidney failure A client with arthritis taking daily acetaminophen A client with pain from a kidney stone

A client with esophageal varices The client with esophageal varices is at risk for hypovolemic shock. Esophageal varices are caused by portal hypertension where the portal vessels are under high pressure. With this high pressure, the portal vessels are prone to rupture, causing massive upper gastrointestinal tract bleeding and hypovolemic shock.As the kidneys fail, fluid is typically retained, causing fluid volume excess, not hypovolemia. Arthritis and daily acetaminophen use do not cause GI bleeding and hypovolemia. Nonsteroidal anti-inflammatory drugs such as naproxen and ibuprofen may predispose the client to gastrointestinal (GI) bleeding and hypovolemia. Although a kidney stone may cause hematuria, massive blood loss or hypovolemia generally does not occur.

The nurse is caring for client who is receiving erythropoietin. Which assessment finding indicates a positive response to the medication? A decrease in fatigue Potassium within normal range Absence of spontaneous fractures Hematocrit of 26.7%

A decrease in fatigue The assessment finding of less fatigue is considered a positive response to erythropoietin. Treatment of anemia with erythropoietin will result in increased hemoglobin and hematocrit (H&H) and decreased shortness of breath and fatigue.A hematocrit value of 26.7% is low. Erythropoietin would restore the hematocrit to at least 36% to be effective. Erythropoietin stimulates the bone marrow to increase red blood cell production and maturation, increasing H&H, not potassium. Calcium supplements and phosphate binders prevent renal osteodystrophy and do not treat anemia.

What instruction will the nurse provide to a client to prepare him or her to undergo ultrasonography of the right upper abdominal quadrant to diagnose gallstones?

A small instrument will be rolled over upper abdomen and there will be no pain.

A client with testicular cancer is worried about sterility and the ability to conceive children later. To which resource does the nurse refer the client before surgery takes place? A psychologist American Cancer Society A sperm bank facility The National Infertility Association

A sperm bank facility If the client is interested in having children, he would be encouraged to arrange for semen storage as soon as possible after diagnosis. A sperm bank facility provides comprehensive information on semen collection, storage of semen, the storage contract, costs, and the insemination process. Sperm collection needs to be completed before other treatment options commence.The client is referred to the American Cancer Society for more generalized information on testicular cancer. A psychologist may be helpful if the client has difficulty with sperm banking or has other concerns associated with testicular cancer. RESOLVE: The National Infertility Association is an appropriate referral if permanent sterility occurs and sperm storage has not been feasible.

11. Which client does the nurse understand has the greatest need for evaluation of possible endometrial cancer? A. 63-year-old having bleeding after menopause B. 51-year-old having irregular menses for 6 months C. 35-year-old with report of multiple sexual partners D. 23-year-old with no menstrual period for 3 months

A. 63-year-old having bleeding after menopause

Which actions are most effective for nurses and other health care workers to prevent occupational transmission of viral hepatitis? Select all that apply. A. Washing hands before and after contact with all clients B. Using needleless systems for parenteral therapy C. Using Standard Precautions with all clients regardless of age or sexual orientation D. Obtaining an immunoglobulin injection after exposure to hepatitis A E. Being fully vaccinated with the hepatitis B vaccine F. Wearing gloves during direct contact with all clients

A. Washing hands before and after contact with all clients B. Using needleless systems for parenteral therapy C. Using Standard Precautions with all clients regardless of age or sexual orientation D. Obtaining an immunoglobulin injection after exposure to hepatitis A E. Being fully vaccinated with the hepatitis B vaccine

Which nursing actions are included in the care of a client who had a transurethral resection of the prostate (TURP)? SATA a. helping the client out of bed to the chair as soon as permitted to prevent complications of immobility b. using normal saline solution for the intermittent bladder irrigant unless otherwise prescribed c. monitoring and documenting the color, consistency, and amount of urine output d. providing a safe environment for the client because of temporary changes in mental status e. assessing the client for reports of severe bladder spasms with decreased urinary output f. checking the drainage tubing frequently for external obstructions such as kinks, and internal obstructions such as blood clots

ALL

A nurse prepares to discharge a client with chronic pancreatitis. Which question should the nurse ask to ensure safety upon discharge? a. "Do you have a one- or two-story home?" b. "Can you check your own pulse rate?" c. "Do you have any alcohol in your home?" d. "Can you prepare your own meals?"

ANS: A A client recovering from chronic pancreatitis should be limited to one floor until strength and activity increase. The client will need a bathroom on the same floor for frequent defecation. Assessing pulse rate and preparation of meals is not specific to chronic pancreatitis. Although the client should be encouraged to stop drinking alcoholic beverages, asking about alcohol availability is not adequate to assess this client's safety.

16. The nurse is teaching a client with chronic kidney disease (CKD) about the sodium restriction needed in the diet to prevent edema and hypertension. Which statement by the client indicates more teaching is needed? a. "I am thrilled that I can continue to eat fast food." b. "I will cut out bacon with my eggs every morning." c. "My cooking style will change by not adding salt." d. "I will probably lose weight by cutting out potato chips."

ANS: A Fast food restaurants usually serve food that is high in sodium. This statement indicates that more teaching needs to occur. The other statements show a correct understanding of the teaching.

11. A male client with chronic kidney disease (CKD) is refusing to take his medication and has missed two hemodialysis appointments. What is the best initial action for the nurse? a. Discuss what the treatment regimen means to him. b. Refer the client to a mental health nurse practitioner. c. Reschedule the appointments to another date and time. d. Discuss the option of peritoneal dialysis.

ANS: A The initial action for the nurse is to assess anxiety, coping styles, and the client's acceptance of the required treatment for CKD. The client may be in denial of the diagnosis. While rescheduling hemodialysis appointments may help, and referral to a mental health practitioner and the possibility of peritoneal dialysis are all viable options, assessment of the client's acceptance of the treatment should come first.

A nurse cares for a client who presents with tachycardia and prostration related to biliary colic. Which actions should the nurse take? (Select all that apply.) a. Contact the provider immediately. b. Lower the head of the bed. c. Decrease intravenous fluids. d. Ask the client to bear down. e. Administer prescribed opioids.

ANS: A, B Clients who are experiencing biliary colic may present with tachycardia, pallor, diaphoresis, prostration, or other signs of shock. The nurse should stay with the client, lower the client's head, and contact the provider or Rapid Response Team for immediate assistance. Treatment for shock usually includes intravenous fluids; therefore, decreasing fluids would be an incorrect intervention. The client's tachycardia is a result of shock, not pain. Performing the vagal maneuver or administering opioids could knock out the client's compensation mechanism.

A nurse cares for a client with hepatic portal-systemic encephalopathy (PSE). The client is thin and cachectic in appearance, and the family expresses distress that the client is receiving little dietary protein. How should the nurse respond? a. "A low-protein diet will help the liver rest and will restore liver function." b. "Less protein in the diet will help prevent confusion associated with liver failure." c. "Increasing dietary protein will help the client gain weight and muscle mass." d. "Low dietary protein is needed to prevent fluid from leaking into the abdomen."

ANS: B A low-protein diet is ordered when serum ammonia levels increase and/or the client shows signs of PSE. A low-protein diet helps reduce excessive breakdown of protein into ammonia by intestinal bacteria. Encephalopathy is caused by excess ammonia. A low-protein diet has no impact on restoring liver function. Increasing the client's dietary protein will cause complications of liver failure and should not be suggested. Increased intravascular protein will help prevent ascites, but clients with liver failure are not able to effectively synthesize dietary protein.

After teaching a client who has alcohol-induced cirrhosis, a nurse assesses the client's understanding. Which statement made by the client indicates a need for additional teaching? a. "I cannot drink any alcohol at all anymore." b. "I need to avoid protein in my diet." c. "I should not take over-the-counter medications." d. "I should eat small, frequent, balanced meals."

ANS: B Based on the degree of liver involvement and decreased function, protein intake may have to be decreased. However, some protein is necessary for the synthesis of albumin and normal healing. The other statements indicate accurate understanding of self-care measures for this client.

27. A client is recovering from a kidney transplant. The client's urine output was 1500 mL over the last 12-hour period since transplantation. What is the priority assessment by the nurse? a. Checking skin turgor b. Taking blood pressure c. Assessing lung sounds d. Weighing the client

ANS: B By taking blood pressure, the nurse is assessing for hypotension that could compromise perfusion to the new kidney. The nurse then should notify the provider immediately. Skin turgor, lung sounds, and weight could give information about the fluid status of the client, but they are not the priority assessment.

25. The nurse is teaching a client how to increase the flow of dialysate into the peritoneal cavity during dialysis. Which statement by the client demonstrates a correct understanding of the teaching? a. "I should leave the drainage bag above the level of my abdomen." b. "I could flush the tubing with normal saline if the flow stops." c. "I should take a stool softener every morning to avoid constipation." d. "My diet should have low fiber in it to prevent any irritation."

ANS: C Inflow and outflow problems of the dialysate are best controlled by preventing constipation. A daily stool softener is the best option for the client. The drainage bag should be below the level of the abdomen. Flushing the tubing will not help with the flow. A diet high in fiber will also help with a constipation problem.

21. The charge nurse is orienting a float nurse to an assigned client with an arteriovenous (AV) fistula for hemodialysis in her left arm. Which action by the float nurse would be considered unsafe? a. Palpating the access site for a bruit or thrill b. Using the right arm for a blood pressure reading c. Administering intravenous fluids through the AV fistula d. Checking distal pulses in the left arm

ANS: C The nurse should not use the arm with the AV fistula for intravenous infusion, blood pressure readings, or venipuncture. Compression and infection can result in the loss of the AV fistula. The AV fistula should be monitored by auscultating or palpating the access site. Checking the distal pulse would be an appropriate assessment.

5. The nurse is teaching a client who is undergoing brachytherapy about what to immediately report to her health care provider. Which signs and symptoms would be included in this teaching? (Select all that apply.) a. Constipation for 3 days b. Temperature of 99 F c. Abdominal pain d. Visible blood in the urine e. Heavy vaginal bleeding

ANS: C, D, E Health teaching for a client having brachytherapy should emphasize reporting abdominal pain, visible blood in the urine, and heavy vaginal bleeding. Severe diarrhea (not constipation), urethral burning, extreme fatigue, and a fever over 100 F should also be reported.

13. A client has just returned from a total abdominal hysterectomy and needs postoperative nursing care. What action can the nurse delegate to the unlicensed assistive personnel (UAP)? a. Assess heart, lung, and bowel sounds. b. Check the hemoglobin and hematocrit levels. c. Evaluate the dressing for drainage. d. Empty the urine from the urinary catheter bag.

ANS: D The UAP is able to empty the urinary output from the catheter. The nurse would assess the heart, lung, and bowel sounds; check the hemoglobin and hematocrit levels; and evaluate the drainage on the dressing.

7. A client has undergone a vaginal hysterectomy with a bilateral salpingo-oophorectomy. She is concerned about a loss of libido. What intervention by the nurse would be best? a. Suggest increasing vitamins and supplements daily. b. Discuss the value of a balanced diet and exercise. c. Reinforce that weight gain may be inevitable. d. Teach that estrogen cream inserted vaginally may help.

ANS: D Use of vaginal estrogen cream and gentle dilation can help with vaginal changes and loss of libido. Weight gain and masculinization are misperceptions after a vaginal hysterectomy. Vitamins, supplements, a balanced diet, and exercise are helpful for healthy living, but are not necessarily going to increase libido.

Which statement does the nurse identify as accurate regarding the prostate-specific antigen (PSA) test? Select all that apply. Elevated PSA levels are specific only to prostate cancer. African-American men may benefit from starting PSA screening at age 40. Health care providers may interpret the results of the PSA test differently. The PSA test can be used to monitor the disease course after treatment. PSA levels less than 7.5 ng/mL (7.5 mcg/L) may be considered normal.

African-American men may benefit from starting PSA screening at age 40. Health care providers may interpret the results of the PSA test differently. The PSA test can be used to monitor the disease course after treatment. Although elevated PSA levels may be associated with prostate cancer, there is variance among health care providers in interpretation of results. Levels less than 2.5 to 4.0 ng/mL may be considered normal depending on the resource used. The PSA test can be used to screen for prostate cancer, as well as to monitor the disease after treatment.Certain factors such as digital rectal examination (DRE), prostatitis, acute urinary retention, and ejaculation can cause transient rises in PSA. African-American men are 1.6 times as likely to develop prostate cancer and twice as likely to die from it compared to white men. For this reason, teach African-American male patients to begin prostate cancer screening at age 40.

Which action will the nurse take first when an 80-year-old client with acute pancreatitis has no breath sounds in the left lower lung lobe?

Apply oxygen.

A client is exhibiting signs and symptoms of early shock. Which nursing actions support the psychosocial integrity of the client? (Select all that apply.) Select all that apply. Ask family members to stay with the client. Increase IV and oxygen rates. Call the health care provider. Remain with the client. Reassure the client that everything is being done for him or her.

Ask family members to stay with the client. Remain with the client. Reassure the client that everything is being done for him or her. To support the psychosocial integrity of a client in early shock, the nurse would have a familiar person nearby to comfort the client. The nurse would also remain with the client and offer genuine support to reassure the client that everything is being done for her.The health care provider would be notified, and increasing IV and oxygen rates may be needed, but these actions do not support the client's psychosocial integrity.

Which is the most effective action for the nurse to take to assess adequate bowel function in a client acute pancreatitis who is at risk Ibr development of paralytic (adynamic) ileus?

Asking the client if he or she has passed flatus or had a stool

40. A patient has had an anterior cervical discectomy with fusion and has returned from the recovery room. What is the priority assessment? Assessment of the airway and respiratory effort Assess for bleeding and drainage at the incision site Assess for the gag reflex and ability to handle own secretions Monitor vital signs and check neurologic status

Assessment of the airway and respiratory effort

An older client with benign prostatic hyperplasia and hypertension is being treated with Doxazosin while in the hospital. Which activity does the nurse delegate to the unlicensed assistive personnel as a priority? Encouraging the client to use the incentive spirometer hourly Keep the head of bed at a strict 45 degrees Frequently re-orienting the client to his surroundings Assisting the client whenever he get out of bed

Assisting the client whenever he get out of bed

A client with chronic kidney disease reports chest pain. The nurse notes tachycardia and low-grade fever. Which additional assessment is warranted? Monitor for decreased peripheral pulses. Determine if the client is able to ambulate. Auscultate for pericardial friction rub. Assess for crackles.

Auscultate for pericardial friction rub. The additional assessment needed for the client with uremia is to auscultate the pericardium for friction rub. Clients with CKD are prone to pericarditis. Signs/symptoms of pericarditis include inspiratory chest pain, tachycardia, narrow pulse pressure, low-grade fever, and pericardial friction rub.Crackles and tachycardia are symptomatic of fluid overload. Fever is not present with fluid overload. Although the nurse will monitor pulses, and ambulation is important to prevent weakness and deep vein thrombosis, these are not pertinent to the constellation of signs/symptoms of pericarditis that the client presents with.

Which instruction by the nurse will help a client with chronic kidney disease prevent renal osteodystrophy? Increasing dairy products enriched with vitamin D Drinking cola beverages only once daily Consuming a low-calcium diet Avoiding peas, nuts, and legumes

Avoiding peas, nuts, and legumes To prevent renal osteodystrophy in a chronic kidney disease client, the nurse needs to instruct the client to avoid peas, nuts, and legumes. Kidney failure causes hyperphosphatemia, so phosphorus-containing foods such as beans, peas, nuts (peanut butter), and legumes must be restricted.Calcium would not be restricted. Hyperphosphatemia results in a decrease in serum calcium and demineralization of the bone. Cola beverages and dairy products are high in phosphorus, contributing to hypocalcemia and bone breakdown.

Which priority question will the nurse ask to evaluate the current condition of a 40-year-old client who is experiencing heavy vaginal bleeding? A. "Are you sexually active and do you use oral contraceptives?" B. "Are you feeling weak, dizzy, or light-headed?" C. "Is the bleeding related to your menstrual cycle or intercourse?" D. "Are you having any sensations of pain or cramping?"

B. "Are you feeling weak, dizzy, or light-headed?"

16. For which client problems associated with hypovolemic shock will the nurse specifically prepare to administer a blood product rather than an IV crystalloid? Select all that apply. A. Acidosis B. Hypoxemia C. Dehydration D. Hypotension E. Hyponatremia F. Low hematocrit and hemoglobin levels

B. Hypoxemia F. Low hematocrit and hemoglobin levels Blood products, such as packed red blood cells (PRBCs), are used when shock is caused by blood loss resulting in hypoxemia. PRBCs increase hematocrit and hemoglobin levels along with some fluid volume. Crystalloid fluids contain only minerals, salts, sugars, and nonprotein substances. They can help restore volume, electrolyte balance, and may buffer lactic acid. However, these fluids neither correct low hematocrit or hemoglobin levels nor correct hypoxemia.

Which factors will the nurse assess in a male client who reports decreased libido? Select all that apply. A. Timing of exercise program B. Use of tobacco C. Occupation D. Consumption of alcohol E. Weight gain or loss F. Illicit substance use

B. Use of tobacco D. Consumption of alcohol F. Illicit substance use

An 18-year-old female is diagnosed with possible toxic shock syndrome (TSS) and has these vital signs: T 103.2° F (39.6° C), P 124 beats/min, R 36 breaths/min, BP 84/30 mm Hg. Which primary health care provider request does the nurse implement first? Begin infusion of normal saline IV Obtain blood cultures × 2 sites. Administer corticosteroids as prescribed. Give cefazolin as prescribed.

Begin infusion of normal saline IV The client with possible toxic shock syndrome (TSS) must be treated rapidly. After securing the airway, the client needs to have blood cultures quickly drawn, prior to administration of antibiotics (cefazolin), for the best opportunity to identify the organism. The antibiotics are needed to fight the infection, while the saline infusion is needed to support fluid and electrolyte balance. Corticosteroids may be prescribed to treat skin changes, so this can be accomplished last.

The nurse is caring for a female client with uterine leiomyoma. What is the most likely assessment finding the nurse anticipates? Anxiety Pain Bleeding Constipation

Bleeding The most likely problem for women with uterine leiomyoma is the potential for prolonged or heavy bleeding.Pain is not common but may occur if the fibroid becomes twisted. Constipation may occur if fibroids are large and press on other organs. Anxiety may be present; however, bleeding is much more common.

Which postprocedural findings will the nurse instruct a client who had a prostate biopsy to expect? A. Swelling of the biopsy area and difficulty urinating are expected during the first week. B. A low-grade fever and bright red penile discharge are normal for several days. C. Slight soreness and light rectal bleeding that is bright red are expected for a few days. D. Seminal fluid will appear normal within a day after the procedure.

C. Slight soreness and light rectal bleeding that is bright red are expected for a few days.

A client has undergone the Whipple procedure (radical pancreaticoduodenectomy) for pancreatic cancer. Which nursing actions would the nurse implement to prevent potential complications? (Select all that apply.) Select all that apply. Ensure that drainage color is clear. Check blood glucose often. Place the client in the supine position. Check bowel sounds and stools. Monitor mental status.

Check blood glucose often. Check bowel sounds and stools. Monitor mental status. To prevent potential complications after a Whipple procedure, the nurse would check the client's glucose often to monitor for diabetes mellitus. Bowels sounds and stools would be checked to monitor for bowel obstruction. A change in mental status or level of consciousness could be indicative of hemorrhage.Clear, colorless, bile-tinged drainage or frank blood with increased output may indicate disruption or leakage of a site of anastomosis but is not a precautionary action for the nurse to implement. The client should be placed in semi-Fowler and not supine position to reduce tension on the suture line and the anastomosis site and to optimize lung expansion.

The nurse is caring for a patient who had a nephrectomy yesterday. The nurse notes that the urine flow was 50 mL/hour at the beginning of the shift, but several hours later has dropped to 30 mL/hour. What would the nurse do first? Notify the health care provider for an order for an IV fluid bolus Check the drainage system for kinks or obstructions to flow Document the finding and continue to monitor for a downward trend. Gently irrigate the catheter with 30 mL normal saline to ensure it is patent

Check the drainage system for kinks or obstructions to flow

5. Which of the following assessments if made by the nurse in a client who has SOB would lead the nurse to suspect a tension pneumothorax? [Select All That Apply] Chest wall asymmetry Anxiety/Agitation Crackles and Rhonchi on the affected side Blood presure 73/39 mmHg Subcutaneous emphysema on the affected side

Chest wall asymmetry Blood presure 73/39 mmHg Anxiety/Agitation

The nurse is caring for a client with cholecystitis. Which assessment finding indicates to the nurse that the condition is chronic rather than acute? Rebound tenderness in the right upper quadrant Abdomen that is hyperresonant to percussion Hyperactive bowel sounds and diarrhea Clay-colored stools and dark amber urine

Clay-colored stools and dark amber urine

Which client demonstrates the highest risk for hypovolemic shock? Client receiving a blood transfusion Client with syndrome of inappropriate antidiuretic hormone (SIADH) secretion Client with myocardial infarction Client with severe ascites

Client with severe ascites A client with severe ascites best demonstrates the problem with the highest risk for hypovolemic shock. Fluid shifts from vascular to intra-abdominal may cause decreased circulating blood volume and poor tissue perfusion.The client receiving a blood transfusion does not have as high a risk as the client with severe ascites. Myocardial infarction results in tissue necrosis in the heart muscle, but no blood or fluid losses occur. Owing to excess antidiuretic hormone secretion, the client with SIADH will retain fluid and therefore is not at risk for hypovolemic shock.

The nurse will explain to the client who is to undergo a percutaneous renal biopsy that the client will need to have which of the following done prior to the procedure? Antibiotic treatment for 3 days prior to the biopsy Insertion of a foley catheter Coagulation studies prior to the biopsy Injection of IV contrast

Coagulation studies prior to the biopsy

The assistive personnel (AP) is concerned about a postoperative client with blood pressure (BP) of 90/60 mm Hg, heart rate of 80 beats/min, and respirations of 22 breaths/min. What is the appropriate nursing action? Compare these vital signs with the last several readings. Increase the rate of intravenous fluids. Request that the surgeon see the client. Reassess vital signs using different equipment.

Compare these vital signs with the last several readings. The nurse will take the vital sign trends into consideration. A BP of 90/60 mm Hg may be normal for this client.Calling the surgeon is not necessary at this point, and increasing IV fluids is not indicated. The same equipment must be used when vital signs are taken postoperatively.

To prevent prerenal acute kidney injury, which person will the nurse encourage to increase fluid consumption? Office secretary Construction worker School teacher Taxicab driver

Construction worker Construction workers perform physical labor and work outdoors, especially in warm weather. Working in this type of atmosphere causes diaphoresis and places this worker at risk for dehydration and prerenal azotemia.The office secretary and schoolteacher work indoors and, even without air conditioning, will not lose as much fluid to diaphoresis as someone performing physical labor. The taxicab driver, even without air conditioning, will not experience diaphoresis and fluid loss like the construction worker.

The nurse is caring for a client following a kidney transplant. Which assessment data indicate to the nurse possible rejection of the kidney? (Select all that apply.) Select all that apply. Crackles in the lung fields Temperature of 98.8° F (37.1° C) Blood pressure of 164/98 mm Hg Blood urea nitrogen (BUN) 21 mg/dL (7.5 mmol/L), creatinine 0.9 mg/dL (80 mcmol/L) 3+ edema of the lower extremities

Crackles in the lung fields Blood pressure of 164/98 mm Hg 3+ edema of the lower extremities Signs and symptoms indicating rejection of a transplanted kidney include: crackles in the lung fields, blood pressure of 164/78 mm Hg, and 3+ edema of lower extremities. These are assessment findings related to fluid retention and transplant rejection.Increasing BUN and creatinine are symptoms of rejection; however, a BUN of 21 mg/dL (7.5 mmol/L) and a creatinine of 0.9 mg/dL (80 mcmol/L) reflect normal values. Fever, not normothermia, is symptomatic of transplant rejection.

21. Which question does the nurse ask when a 22-year-old female client is being evaluated for possible toxic shock syndrome (TSS)? A. "How many pads do you use on heavy flow days?" B. "Have you every used intravaginal estrogen therapy?" C. "Do you have a history of multiple sexual partners?" D. "Do you use an insertable contraceptive device?"

D. "Do you use an insertable contraceptive device?"

What does the nurse tell an African-American client whose prostate-specific antigen (PSA) level is less than 2.5 ng/mL? A. African-American men typically have lower-than-normal PSA levels. B. This level indicates the need for follow-up for possible prostate cancer. C. The test should be repeated on an annual basis to monitor for abnormalities. D. A PSA level of less than 2.5 ng/mL is generally considered a normal value.

D. A PSA level of less than 2.5 ng/mL is generally considered a normal value.

20. What self-management strategy will the nurse recommend to a client to prevent vulvovaginitis? A. Wear lightweight nylon underwear. B. Apply antiseptic cream to the perineal area daily. C. Cleanse the inner labial mucosa with antiseptic soap. D. Choose breathable fabrics to wear such as cotton.

D. Choose breathable fabrics to wear such as cotton.

During the resusitative phase, a priority nursing diagnosis for the patient with a 18% full thickness burn to the left leg is: Hyperthermia Acute pain Ineffective airway clearance Deficient fluid volume

Deficient fluid volume

Which actions and precautions will the nurse educate a client with chronic pancreatitis about when starting pancreatic enzyme replacement therapy (PERT)?

Do not crush or chew the capsules. Take these drugs with all meals and snacks. Check your stools for amount and presence of fat to assess whether the drugs are working

A client has returned to the nursing unit after a total abdominal hysterectomy. During the initial assessment, the nurse auscultates the client's abdomen and does not hear bowel sounds. Which is the nurse's priority intervention? Position the client on the right side. Measure abdominal girth. Irrigate the nasogastric tube. Document the finding in the chart.

Document the finding in the chart.

A client with end-stage kidney disease has been placed on fluid restrictions. Which assessment data indicates to the nurse that the fluid restriction has not been followed? Dyspnea and anxiety at rest Blood pressure of 118/78 mm Hg Central venous pressure (CVP) of 6 mm Hg Weight loss of 3 lb (1.4 kg) during hospitalization

Dyspnea and anxiety at rest The assessment finding that shows that the client has not adhered to fluid restriction is dyspnea and anxiety at rest. Dyspnea is a sign of fluid overload and possible pulmonary edema. The nurse needs to assist the client in correlating symptoms of fluid overload with nonadherence to fluid restriction.Nonadherence to fluid restriction results in fluid volume excess and higher blood pressures. 118/78 mm Hg is a normal blood pressure. Excess fluid intake and fluid retention are manifested by an elevated CVP (>8 mm Hg) and weight gain, not weight loss.

The nurse is teaching a client how to adapt to physical and psychological changes after surgery for ovarian cancer. Which factor is included in the teaching plan? (Select all that apply.) Select all that apply. Encouraging the expression of grief and fears Suggesting the use of support groups and counseling Refraining from sexual intercourse for 6 weeks after surgery. Offering vaginal dilators Recommending the use of oil-based lubricants for vaginal dryness

Encouraging the expression of grief and fears Suggesting the use of support groups and counseling Refraining from sexual intercourse for 6 weeks after surgery. Support groups are advisable for clients with ovarian cancer because the loss of reproductive organs involves a grief reaction. Ovarian cancer particularly carries the connotation of being serious and incurable in the view of many women. The client needs to refrain from sexual intercourse for 6 weeks after surgery.The use of a vaginal dilator is not indicated. After the woman becomes sexually active, she may have a problem with vaginal dryness as the result of hormonal changes. Water-based, rather than oil-based, lubricants would be suggested.

The nurse is planning care for a client with late-phase septic shock. All of the following treatments have been prescribed. Which prescription does the nurse question? Transfusion of 2 units of fresh frozen plasma Regular insulin intravenous drip per protocol Enoxaparin (Lovenox) 40 mg subcutaneous twice daily Cefazolin (Ancef) 1 g IV every 6 hours

Enoxaparin (Lovenox) 40 mg subcutaneous twice daily

When caring for a client with a left forearm arteriovenous fistula created for hemodialysis, which nursing actions are required? (Select all that apply.) Select all that apply. Ensure that no blood pressures are taken in that arm. Teach the client to palpate for a thrill over the site. Elevate the arm above heart level. Auscultate for a bruit every 8 hours. Check brachial pulses daily.

Ensure that no blood pressures are taken in that arm. Teach the client to palpate for a thrill over the site. Auscultate for a bruit every 8 hours.

The nurse is caring for a client who had a nephrectomy 24 hours previously. The nurse notes that the urine flow has decreased from 50 mL/hour to 30 mL/hour. What is the nurse's priority action? Notify the health care provider for an order for an IV fluid bolus Draw labs for hemoglobin and hematocrit Document the finding and continue to monitor for downward trend Evaluate the drainage system for patency

Evaluate the drainage system for patency

32. A clients urinary catheter is removed on the 3rd post-operative day following his transurethral resection of the prostate. Afterward, he is upset because he is dribbling urine. Which is the most appropriate nursing intervention in this situation? Instruct the client to limit fluid intake to 1000 ml/day to prevent dribbling Obtain urine for culture, because he may have a urinary infection Notify the physician Explain to him that dribbling may take some time to clear up and assist him to keep dry.

Explain to him that dribbling may take some time to clear up and assist him to keep dry.

A nurse review the following data in the chart of a patient with burn injuries: 35 year old male with bilateral leg burns with a white leather like appearance. No blisters or bleeding noted. patient rates pain 2/10. History of asthma. Allergies to PCN. Based on the data provided, how would the nurse categorize this patient's injuries? Partial- thickness superficial Full thickness Partial thickness-deep Superficial

Full thickness

The nurse is teaching a group of young women. Which factor does the nurse teach increases a women's risk for development of cervical cancer? Having more than six pregnancies Having sexual intercourse at a very early age Using a diaphragm with spermicidal jelly for contraception Eating a diet that is high in fat content

Having sexual intercourse at a very early age Having intercourse at a very early age and/or multiple sex partners places a woman at high risk for the development of cervical cancer.Eating a diet that is high in fat content, the number of pregnancies, and using a diaphragm have not been identified as increasing the risk for cervical cancer.

The nurse would expect to assess which of the following manifestations in the client who has been diagnosed with nephrotic syndrome: [Select all that apply]. Hyperlipidemia Anasarca Hypoalbuminemia Leukocytosis Proteinuia

Hypoalbuminemia Proteinuia Anasarca

A patient is admitted with severe acute pancreatitis. While taking the client's blood pressure the nurse notices a carpal spasm (Trousseau sign) with the hand on the side where the blood pressure is being taking. What lab result would the nurse expect in response to this finding? Hypocalcemia Hyperkalemia Decreased bilirubin Decreased albumin

Hypocalcemia

The nurse is caring for a patient who is demonstrating signs of being in the diuretic phase of acute renal failure. During this phase, the client should be monitored for signs of: Respiratory acidosis Hyperkalemia Hypertension Hypovolemia

Hypovolemia

27. A client who was treated one year ago with an orchiectomy for a seminoma now has an elevated serum alpha-fetoprotein level. Which topic is most important for the nurse to teach this client about? Sperm banking options Importance of yearly monitoring Increased risk of cryptorchidism Hospice philosophy

Increased risk of cryptorchidism Alpha-fetoprotein is a tumor marker that is not produced in significant amounts by normal adult tissues. An increase in the level of alpha-fetoprotein after treatment most commonly indicates recurrence or metastasis. Chemotherapy is used to treat recurrent or metastatic disease. The client should already have been taught about sperm banking. Hospice is not indicated at this time. Because a rise in these levels indicates recurrence or metastasis, yearly monitoring is not appropriate.

A postoperative client is admitted to the intensive care unit (ICU) with hypovolemic shock. Which nursing action will the nurse delegate to an experienced assistive personnel (AP)? Assess level of alertness. Obtain vital signs every 15 minutes. Measure hourly urine output. Check oxygen saturation.

Measure hourly urine output. The nurse delegates to an experienced ICU AP the measurement of hourly urine output for a client with hypovolemic shock. Monitoring hourly urine output is included in nursing assistant education and does not require special clinical judgment. The nurse will evaluate the results.Obtaining vital signs, monitoring oxygen saturation, and assessing mental status in critically ill clients requires the clinical judgment of the critical care nurse because immediate intervention may be needed.

What action will the nurse take when, 12 hours a traditional cholecystectomy* a client's Jackson-Pratt (JP) drain shows serosanguineous drainage stained with bile?

Measuring the drainage and documenting ' findings

Which statement indicates to the nurse that a client who is experiencing frequent episodes of "indigestion" and flatulence may have cholecystitis?

My right arm and shoulder always seem to hurt after I eat fried foods."

A patient with hypovolemia is restless and anxious. The skin is cool and pale, pulse is thready at a rate of 140 beats/min, blood pressure is 89/42 mm Hg, and respirations are 32/min. What actions must the nurse take? Select all that apply. Obtain a stat order for an IV normal saline bolus. Place the patient in a semi-Fowler's position. Administer supplemental oxygen. Check vital signs at least every 15 minutes. Call a "code blue." Notify the Rapid Response Team.

Obtain a stat order for an IV normal saline bolus. Administer supplemental oxygen. Check vital signs at least every 15 minutes. Notify the Rapid Response Team.

6. The nurse is teaching a class on the risk factors associated with the development of ovarian cancer. Which is the nurse sure to include in the teaching? Over age 30 yrs. at first pregnancy, and family history of breast cancer Age 25, and taking oral contraceptives. Younger than 30, and sedentary lifestyle 10 year history of smoking, and daily alcohol consumption

Over age 30 yrs. at first pregnancy, and family history of breast cancer

Which assessment finding is the priority for the nurse to report to the health care provider? Painless testicular lump Penile discharge Hematuria Decrease increase in libido

Painless testicular lump A painless lump or swelling in the testicles is the most common assessment finding of testicular cancer; this finding should be reported to the health care provider immediately.All other symptoms require nursing intervention, but these can be addresses after the painless testicular lump.

A patient who had a kidney transplant 5 years ago is experiencing progressively reduced function of the organ. Which intervention is appropriate for this patient? Patient should be immediately prepped for surgical removal of the organ. Drug management may limit the damage and allow the graft to be maintained. Patient should be admitted to intensive care for observation and possible dialysis. Patient should be educated about retransplantation; related living donor should be sought.

Patient should be educated about retransplantation; related living donor should be sought.

Which fluid and electrolyte balance assessment action will the nurse perform most often for a client with pancreatic cancer after surgery With a traditional Whipple procedure?

Pinching up skin over the sternum and checking for tenting

Which problem places a client at highest risk for sepsis? Client owns an iguana Pericarditis Post kidney transplant Pernicious anemia

Post kidney transplant A client with post kidney transplant is the highest risk for sepsis. This client will need to take lifelong immune suppressant therapy and is at risk for infection from internal and external organisms.Pernicious anemia is related to lack of vitamin B12, not to bone marrow failure (aplastic anemia), which would place the client at risk for infection. Inflammation of the pericardial sac is an inflammatory condition that does not pose a risk for septic shock. Although owning pets, especially cats and reptiles, poses a risk for infection, the immune-suppressed kidney transplant client has a greater risk for infection, sepsis, and death.

A nurse cares for a client who is recovering from an open Whipple procedure. Which action should the nurse take? a. Clamp the nasogastric tube. b. Place the client in semi-Fowler's position. c. Assess vital signs once every shift. d. Provide oral rehydration.

Postoperative care for a client recovering from an open Whipple procedure should include placing the client in a semi-Fowler's position to reduce tension on the suture line and anastomosis sites, setting the nasogastric tube to low suction to remove free air buildup and pressure, assessing vital signs frequently to assess fluid and electrolyte complications, and providing intravenous fluids.

A client with a diagnosis of sepsis develops acute kidney injury. A nurse interprets that this type of renal injury is due to: intrarenal causes extrarenal source prerenal source post-renal causes

Prerenal

Which signs and symptoms will the nurse expect to find on assessment of a client who is admitted with obstructive jaundice?

Pruritus Pale, clay-colored stools Dark, coffee-colored urine

A client has postcholecystectomy syndrome (PCS) with persistent abdominal pain accompanied by vomiting for several weeks after removal of the gallbladder. Which possible causes or complications will the nurse remain alert for in this client?"

Pseudocyst Common bile duct leak Diverticular compression. Ductal stricture or obstruction Sphincter of Oddi dysfunction Primary sclerosis cholangitis — . Retained or new gallstones

During the oliguric phase of acute renal failure (ARF), the nurse monitors the patient for: Kussmauls respirations and hypotension Decreased serum magnesium and increased uric acid Pulmonary edema and ECG changes Hypokalemia and acidosis

Pulmonary edema and ECG changes

A client admitted to the medical unit with a history of vomiting and diarrhea and an increased blood urea nitrogen requires 1 L of normal saline infused over 2 hours. Which staff member would be assigned to care for the client? LPN/LVN with experience working on the medical unit. New graduate RN who just finished a 6-week orientation. RN who has floated from pediatrics for this shift. RN who usually works on the general surgical unit.

RN who usually works on the general surgical unit. The RN who usually works on the general surgical unit would have the most experience in taking care of surgical clients and would be most capable of monitoring the client receiving rapid fluid infusions. This client is at risk for complications such as pulmonary edema and acute kidney failure.The pediatric float RN and the new graduate RN will have less experience in caring for this type of client. The LPN/LVN would not be assigned to a client requiring IV therapy and who is at high risk for complications.

The client admitted with end-stage liver failure is admitted with hepatic encephalopathy. Which diet restriction should be implemented by the nurse to address this complication? Limit oral fluids to 1,500 mL/day Reduce protein intake to 60-80 grams/day Restrict carbohydrate intake to less than 300 mg/day Reduce sodium intake to 2 grams/day

Reduce protein intake to 60-80 grams/day

55. The client admitted with end-stage liver failure is admitted with hepatic encephalopathy. Which diet restriction should be implemented by the nurse to address this complication? Reduce protein intake to 60-80 grams/day Restrict carbohydrate intake to less than 300 mg/day Limit oral fluids to 1,500 mL/day reduce sodium intake to 2 grams/day

Reduce protein intake to 60-80 grams/day Ammonia is a byproduct of protein

A patient is admitted to the emergency department with toxic shock syndrome. Which action by the nurse is most important? Remove the tampon as the source of infection Administer IV fluid to maintain fluid and electrolyte balance Collect a blood specimen for culture and sensitivity Transfuse the patient to manage low blood count

Remove the tampon as the source of infection The source of infection should be removed first. All of the other answers are possible interventions depending on the clients symptoms and vital signs, but removing the tampon is the priority.

A nurse is caring for a burn client who is scheduled for an escharotomy procedure being performed for a full thickness circumferential arm burn. The nurse understands that the anticipated therapeutic outcome of the escharotomy is: Return of distal pulses Formation of granulation tissue Brisk bleeding from the site Decreasing edema formation

Return of distal pulses I found: The nurse is preparing to care for a burn client scheduled for an escharotomy procedure beingperformed for a third-degree circumferential arm burn. The nurse understands that which findingis the anticipated therapeutic outcome of the escharotomy? 1. Return of distal pulses 2. Brisk bleeding from the site 3. Decreasing edema formation 4. Formation of granulation tissue

A client reports taking a supplement that his partner recommended for benign prostatic hyperplasia (BPH). Which supplement does the nurse anticipate the client is taking? Magnesium Calcium Saw palmetto Feverfew

Saw palmetto Serenoa repens (saw palmetto), a plant extract, is often used by men with early to moderate BPH. Some studies show this supplement to be effective in treating BPH, while other studies show no benefit. The nurse will remind clients wanting to use complementary and integrative health therapies to check with their primary health care provider before taking anything.Magnesium, calcium, and feverfew are not therapies used for BPH.

Which nurse would be assigned to care for a client who is intubated with septic shock due to a methicillin-resistant Staphylococcus aureus (MRSA) infection? The RN who will also be caring for a client who had coronary artery bypass graft (CABG) surgery 12 hours ago. The RN with 2 years of experience in intensive care unit (ICU). The LPN/LVN who has 20 years of experience. The new RN who recently finished orienting and is working independently with moderately complex clients.

The RN with 2 years of experience in intensive care unit (ICU). The RN with 2 years ICU experience would be assigned to care for an intubated client with septic shock due to a MRSA infection. This RN with current intensive care experience who is not caring for a postoperative client is an appropriate nurse to care for this client.Care of the unstable client with intubation and mechanical ventilation is not within the scope of practice for the LPN/LVN. A client who is experiencing septic shock is too complex for the new RN. Although the RN who is also caring for the post-CABG client is experienced, this assignment will put the post-CABG client at risk for MRSA infection.

The client who is to start taking Tolterodine should be instructed by the nurse to notify the prescribing health provider if which of the following side effects associated with this medication occurs? Edema of the lower extremities Urinary retention Hydronephrosis Ecchymosis

Urinary retention

When caring for a client with portal hypertension, the nurse assesses for which potential complications? (Select all that apply.) a) Esophageal varices b) Ascites c) Hematuria d) Hemorrhoids e) Fever

a) Esophageal varices b) Ascites d) Hemorrhoids Potential complications of portal hypertension include esophageal varices, ascites, and hemorrhoids. Portal hypertension results from increased resistance to or obstruction (blockage) of the flow of blood through the portal vein and its branches. The blood meets resistance to flow and seeks collateral (alternative) venous channels around the high-pressure area. Veins become dilated in the esophagus (esophageal varices), rectum (hemorrhoids), and abdomen (ascites due to excessive abdominal [peritoneal] fluid).Hematuria may indicate insufficient production of clotting factors in the liver and decreased absorption of vitamin K. Fever indicates an inflammatory process.

10. A 34-year-old client comes to the clinic with concerns about an enlarged left testicle and heaviness in his lower abdomen. Which diagnostic test would the nurse expect to be ordered to confirm testicular cancer? a. Alpha-fetoprotein (AFP) b. Prostate-specific antigen (PSA) c. Prostate acid phosphatase (PAP) d. C-reactive protein (CRP)

a. Alpha-fetoprotein (AFP) AFP is a glycoprotein that is elevated in testicular cancer. PSA and PAP testing is used in the screening of prostate cancer. CRP is diagnostic for inflammatory conditions.

The nurse is assessing a client who is diagnosed as having Hepatitis A and asks how someone gets this disease. What is the most likely cause of the client's Hepatitis A? a) Being exposed to blood or blood products b) Eating contaminated food or water c) Having unprotected sex d) Sharing needles for illicit drugs

b) Eating contaminated food or water Hepatitis A is transmitted through the fecal-oral route rather than via blood. Therefore, contaminated food or water with Escherichia coli or other microbes can cause this liver infection.

14. A client is being discharged home after a large myocardial infarction and subsequent coronary artery bypass grafting surgery. The client's sternal wound has not yet healed. What statement by the client most indicates a higher risk of developing sepsis after discharge? a. "All my friends and neighbors are planning a party for me." b. "I hope I can get my water turned back on when I get home." c. "I am going to have my daughter scoop the cat litter box." d. "My grandkids are so excited to have me coming home!"

b. "I hope I can get my water turned back on when I get home." All these statements indicate a potential for leading to infection once the client gets back home. A large party might include individuals who are themselves ill and contagious. Having litter boxes in the home can expose the client to microbes that can lead to infection. Small children often have upper respiratory infections and poor hand hygiene that spread germs. However, the most worrisome statement is the lack of running water for handwashing and general hygiene and cleaning purposes.

The nurse is reviewing discharge instructions with a client who has just experienced an endometrial biopsy. Which finding should be reported to the health care provider immediately? a. Mild cramping b. Slight chills and fever c. Spotting of blood on a perineal pad d. Fatigue after anesthesia

b. Slight chills and fever Chills and fever could indicate an infection and should be reported immediately to the health care provider. Mild cramping, spotting, and fatigue are normal findings after an endometrial biopsy.

Which common serum tumor markers does the nurse expect will be ordered to confirm a client's suspected diagnosis of testicular cancer? SATA a. early prostate cancer antigen (EPCA-2) b. lactate dehydrogenase (LDH) c. alpha-fetoprotein (AFP) d. beta human chorionic gonadotropin (hCG) e. BRCA1 mutation f. glutathione S-transferase (GST P1)

b. lactate dehydrogenase (LDH) c. alpha-fetoprotein (AFP) d. beta human chorionic gonadotropin (hCG)

Which action by the nurse would most likely help to relieve symptoms associated with ascites? a) Monitoring serum albumin levels b) Lowering the head of the bed c) Administering oxygen therapy d) Administering intravenous fluids

c) Administering oxygen therapy The best action by the nurse caring for a client with ascites is to elevate the head of the bed and provide supplemental oxygen. The enlarged abdomen of ascites limits respiratory excursion. Fowler position will increase excursion and reduce shortness of breath.Monitoring serum albumin levels will detect anticipated decreased levels associated with cirrhosis and hepatic failure but does not relieve the symptoms of ascites. Administering IV fluids will contribute to fluid volume excess and fluid shifts into the peritoneal cavity, worsening ascites.

Which action does the nurse teach a client with BPH to perform that can help relieve obstructive symptoms? a. urinate before going to bed and immediately upon waking b. consume fluids regularly throughout the day c. increase the frequency of sexual intercourse d. urinate forcefully after drinking a large glass of water

c. increase the frequency of sexual intercourse

The nurse is caring for a client who was recently diagnosed with Laennec cirrhosis. What is the nurse's priority assessment during client care? a) Cardiovascular assessment b) Abdominal assessment, including bowel sounds c) Respiratory assessment d) Cognitive and neurologic assessment

d) Cognitive and neurologic assessment The type of cirrhosis that this client has is caused by alcoholism. Withdrawal from alcohol can cause cognitive and neurologic changes, such as confusion and delirium tremens (DTs).

How would the home care nurse best modify the client's home environment to manage side effects of lactulose? a) Obtains a walker for the client. b) Rearranges furniture to declutter the home. c) Removes throw rugs to prevent falls. d) Requests a bedside commode for the client.

d) Requests a bedside commode for the client. The home care nurse would modify the client's home environment to manage side effects of lactulose by making a bedside commode available to the client. Lactulose therapy increases the frequency of stools. A bedside commode is especially necessary if the client has difficulty reaching the toilet.

The nurse collaborates with the registered dietitian nutritionist in providing teaching for a client who has ascites from cirrhosis. What daily dietary restriction would the nurse include in the health teaching? a) Calcium b) Potassium c) Magnesium d) Sodium

d) Sodium Mild to moderate sodium restriction is often tried as the first intervention to decrease body fluid retention, including ascites.

Which question will the nurse ask a client to determine whether the drug tamsulosin is achieving the desired therapeutic effect? a. "do you have a green or yellow discharge from your penis?" b. "are you having any problems achieving an erection?" c. "does your urine have a strong odor or appear cloudy?" d. "Are you continuing to have difficulty passing urine?"

d. "Are you continuing to have difficulty passing urine?"

6. A nurse and an unlicensed assistive personnel (UAP) are caring for a client with an open radical prostatectomy. Which comfort measure could the nurse delegate to the UAP? a. Administering an antispasmodic for bladder spasms b. Managing pain through patient-controlled analgesia c. Applying ice to a swollen scrotum and penis d. Helping the client transfer from the bed to the chair

d. Helping the client transfer from the bed to the chair The UAP could aid the client in transferring from the bed to the chair and with ambulation. The nurse would be responsible for medication administration, assessment of swelling, and the application of ice if needed.

The nurse is developing a teaching plan for a client who is scheduled for her first Papanicolaou test. What instruction by the nurse is the most accurate? a. The timing of the Pap smear does not matter. b. Sexual intercourse will not interfere with the results. c. Results can be interpreted immediately in the office. d. Results are best if you do not douche 24 hours before the test.

d. Results are best if you do not douche for 24 hours before the test. In order to prevent false interpretation, the client must not douche or have sexual intercourse for at least 24 hours before the Pap smear. Timing is important, with the test scheduled between the clients menstrual periods so that the menstrual flow does not interfere with laboratory analysis. The specimens are placed on a glass slide and sent to the laboratory for examination and cannot be interpreted immediately

24. The nurse will instruct the client who is to undergo an abdominal pelvic ultrasound that the client should: maintain NPO status before the procedure void prior to entering the exam room eat a light breakfast only drink six to eight glasses of water without voiding before the test

drink six to eight glasses of water without voiding before the test A pelvic ultrasound requires the ingestion of large volumes of water just before the procedure. A full bladder isnecessary so that it will be visualized as such and not mistaken for a possible pelvic growth. An abdominalultrasound may require that the client abstain from food or fluid for several hours before the procedure. Option 3is unrelated to this specific proced

The nurse is caring for a client who has cirrhosis of the liver. The client's latest laboratory testing shows a prolonged prothrombin time. For what assessment finding would the nurse monitor: deep vein thrombosis. jaundice. hematemesis. pressure injury.

hematemesis. The client who has cirrhosis is at risk for bleeding due to decreased production of prothrombin by the liver. Portal hypertension that occurs in clients with cirrhosis causes esophageal blood veins to become fragile, distended, and tortuous. Therefore, these veins tend to bleed as evidenced by either hematemesis or melena.

4. Which health promotion teaching will the nurse stress to healthy adults to prevent harm from acute kidney injury (AKI)? A. Check your blood pressure every day. B. Find out if you have a family history of diabetes. C. Avoid dehydration by drinking 2 to 3 liters of water daily. D. Have annual testing for blood urea nitrogen (BUN), creatinine, protein, and glucose.

✅C Dehydration (severe blood volume depletion) reduces perfusion and can lead to AKI even in adults who have no known kidney problems. The nurse urges all healthy adults to avoid dehydration by drinking 2 to 3 liters of water daily.

9. Which nonsurgical treatments for pelvic organ prolapse would the nurse teach a client about? Select all that apply. A. Pelvic floor muscle (Kegel) exercises B. Space-filling vaginal pessaries C. Bladder training program D. Low-fiber diet E. Stool softeners F. Laxatives

A. Pelvic floor muscle (Kegel) exercises B. Space-filling vaginal pessaries C. Bladder training program E. Stool softeners F. Laxatives

Which clients will the nurse suggest to be immunized against hepatitis B (HBV)? Select all that apply. A. People who have unprotected sex with more than one partner B. Men who have sex with men C. Any client scheduled for a surgical procedure D. Firefighters E. Health care providers F. Clients prescribed immunosuppressant drugs

A. People who have unprotected sex with more than one partner B. Men who have sex with men D. Firefighters E. Health care providers F. Clients prescribed immunosuppressant drugs

Which signs and symptoms will the nurse expect to find on assessment of a client with chronic liver disease who has an elevated serum bilirubin level? Select all that apply. A. Pruritus B. Icterus C. Hypertension D. Jaundice E. Pale, clay-colored stools F. Dark, coffee-colored urine

A. Pruritus B. Icterus D. Jaundice E. Pale, clay-colored stools F. Dark, coffee-colored urine

12. Which body parts does the nurse teach a client are removed during surgery for endometrial cancer? Select all that apply. A. Uterus B. Vagina C. Fallopian tubes D. Rectum E. Ovaries F. Bladder

A. Uterus C. Fallopian tubes E. Ovaries

When the normal vaginal flora is disrupted, what condition does the nurse expect that a female client is most likely to experience? A. Vaginal infection B. Vaginal dryness C. Irregular menstrual cycles D. Infertility

A. Vaginal infection

6. A client has a serum potassium level of 6.5 mmol/L, a serum creatinine level of 2 mg/dL, and a urine output of 350 mL/day. What is the best action by the nurse? a. Place the client on a cardiac monitor immediately. b. Teach the client to limit high-potassium foods. c. Continue to monitor the client's intake and output. d. Ask to have the laboratory redraw the blood specimen.

ANS: A The priority action by the nurse should be to check the cardiac status with a monitor. High potassium levels can lead to dysrhythmias. The other choices are logical nursing interventions for acute kidney injury but not the best immediate action.

A nurse delegates hygiene care for a client who has advanced cirrhosis to an unlicensed nursing personnel (UAP). Which statements should the nurse include when delegating this task to the UAP? (Select all that apply.) a. "Apply lotion to the client's dry skin areas." b. "Use a basin with warm water to bathe the client." c. "For the client's oral care, use a soft toothbrush." d. "Provide clippers so the client can trim the fingernails." e. "Bathe with antibacterial and water-based soaps."

ANS: A, C, D Clients with advanced cirrhosis often have pruritus. Lotion will help decrease itchiness from dry skin. A soft toothbrush should be used to prevent gum bleeding, and the client's nails should be trimmed short to prevent the client from scratching himself or herself. These clients should use cool, not warm, water on their skin, and should not use excessive amounts of soap.

2. A nurse is caring for a postoperative 70-kg client who had major blood loss during surgery. Which findings by the nurse should prompt immediate action to prevent acute kidney injury? (Select all that apply.) a. Urine output of 100 mL in 4 hours b. Urine output of 500 mL in 12 hours c. Large amount of sediment in the urine d. Amber, odorless urine e. Blood pressure of 90/60 mm Hg

ANS: A, C, E The low urine output, sediment, and blood pressure should be reported to the provider. Postoperatively, the nurse should measure intake and output, check the characteristics of the urine, and report sediment, hematuria, and urine output of less than 0.5 mL/kg/hour for 3 to 4 hours. A urine output of 100 mL is low, but a urine output of 500 mL in 12 hours should be within normal limits. Perfusion to the kidneys is compromised with low blood pressure. The amber odorless urine is normal.

24. A client is having a peritoneal dialysis treatment. The nurse notes an opaque color to the effluent. What is the priority action by the nurse? a. Warm the dialysate solution in a microwave before instillation. b. Take a sample of the effluent and send to the laboratory. c. Flush the tubing with normal saline to maintain patency of the catheter. d. Check the peritoneal catheter for kinking and curling.

ANS: B An opaque or cloudy effluent is the first sign of peritonitis. A sample of the effluent would need to be sent to the laboratory for culture and sensitivity in order to administer the correct antibiotic. Warming the dialysate in a microwave and flushing the tubing are not safe actions by the nurse. Checking the catheter for obstruction is a viable option but will not treat the peritonitis.

20. The nurse is teaching the main principles of hemodialysis to a client with chronic kidney disease. Which statement by the client indicates a need for further teaching by the nurse? a. "My sodium level changes by movement from the blood into the dialysate." b. "Dialysis works by movement of wastes from lower to higher concentration." c. "Extra fluid can be pulled from the blood by osmosis." d. "The dialysate is similar to blood but without any toxins."

ANS: B Dialysis works using the passive transfer of toxins by diffusion. Diffusion is the movement of molecules from an area of higher concentration to an area of lower concentration. The other statements show a correct understanding about hemodialysis.

A nurse cares for a client who is scheduled for a paracentesis. Which intervention should the nurse delegate to an unlicensed assistive personnel (UAP)? a. Have the client sign the informed consent form. b. Assist the client to void before the procedure. c. Help the client lie flat in bed on the right side. d. Get the client into a chair after the procedure.

ANS: B For safety, the client should void just before a paracentesis. The nurse or the provider should have the client sign the consent form. The proper position for a paracentesis is sitting upright in bed or, alternatively, sitting on the side of the bed and leaning over the bedside table. The client will be on bedrest after the procedure.

After teaching a client who has plans to travel to a non-industrialized country, the nurse assesses the client's understanding regarding the prevention of viral hepatitis. Which statement made by the client indicates a need for additional teaching? a. "I should drink bottled water during my travels." b. "I will not eat off another's plate or share utensils." c. "I should eat plenty of fresh fruits and vegetables." d. "I will wash my hands frequently and thoroughly."

ANS: C The client should be advised to avoid fresh, raw fruits and vegetables because they can be contaminated by tap water. Drinking bottled water, and not sharing plates, glasses, or eating utensils are good ways to prevent illness, as is careful handwashing.

A nurse assesses clients on the medical-surgical unit. Which client is at greatest risk for the development of carcinoma of the liver? a. A 22-year-old with a history of blunt liver trauma b. A 48-year-old with a history of diabetes mellitus c. A 66-year-old who has a history of cirrhosis d. An 82-year-old who has chronic malnutrition

ANS: C The risk of contracting a primary carcinoma of the liver is higher in clients with cirrhosis from any cause. Blunt liver trauma, diabetes mellitus, and chronic malnutrition do not increase a person's risk for developing liver cancer.

14. A client has a long history of hypertension. Which category of medications would the nurse expect to be ordered to avoid chronic kidney disease (CKD)? a. Antibiotic b. Histamine blocker c. Bronchodilator d. Angiotensin-converting enzyme (ACE) inhibitor

ANS: D ACE inhibitors stop the conversion of angiotensin I to the vasoconstrictor angiotensin II. This category of medication also blocks bradykinin and prostaglandin, increases renin, and decreases aldosterone, which promotes vasodilation and perfusion to the kidney. Antibiotics fight infection, histamine blockers decrease inflammation, and bronchodilators increase the size of the bronchi; none of these medications helps slow the progression of CKD in clients with hypertension.

26. A client with chronic kidney disease states, "I feel chained to the hemodialysis machine." What is the nurse's best response to the client's statement? a. "That feeling will gradually go away as you get used to the treatment." b. "You probably need to see a psychiatrist to see if you are depressed." c. "Do you need help from social services to discuss financial aid?" d. "Tell me more about your feelings regarding hemodialysis treatment."

ANS: D The nurse needs to explore the client's feelings in order to help the client cope and enter a phase of acceptance or resignation. It is common for clients to be discouraged because of the dependency of the treatment, especially during the first year. Referrals to a mental health provider or social services are possibilities, but only after exploring the client's feelings first. Telling the client his or her feelings will go away is dismissive of the client's concerns.

5. A client with acute kidney injury has a blood pressure of 76/55 mm Hg. The health care provider ordered 1000 mL of normal saline to be infused over 1 hour to maintain perfusion. The client is starting to develop shortness of breath. What is the nurse's priority action? a. Calculate the mean arterial pressure (MAP). b. Ask for insertion of a pulmonary artery catheter. c. Take the client's pulse. d. Slow down the normal saline infusion.

ANS: D The nurse should assess that the client could be developing fluid overload and respiratory distress and slow down the normal saline infusion. The calculation of the MAP also reflects perfusion. The insertion of a pulmonary artery catheter would evaluate the client's hemodynamic status, but this should not be the initial action by the nurse. Vital signs are also important after adjusting the intravenous infusion.

A client who is receiving fluid resuscitation per the Parkland formula after a serious burn continues to have urine output ranging from 0.2 to 0.25 mL/kg/hour. After the health care provider checks the client, which order does the nurse question? Administer furosemide (Lasix) 40 mg IV push. Draw blood for serum electrolytes stat. Increase IV fluids by 100 mL/hr. Continue to monitor urine output hourly.

Administer furosemide (Lasix) 40 mg IV push.

For which client is the nurse most likely to administer an iron supplement? A. 23-year-old who has pelvic inflammatory disease B. 32-year-old with heavy menstrual bleeding and an intrauterine device C. 53-year-old who is entering menopause and has a breast mass D. 70-year-old who is diagnosed with benign prostatic hyperplasia

B. 32-year-old with heavy menstrual bleeding and an intrauterine device

Which client does the nurse prioritize as most in need of a pelvic examination? A. 25-year-old with possible urinary tract infection (UTI) B. 53-year-old who reports decreased libido and fatigue C. 62-year-old who reports resumption of menses D. 74-year-old with 25-lb intentional weight loss

C. 62-year-old who reports resumption of menses

Which preprocedural instructions will the nurse give a client before a mammogram? A. Do not eat or drink anything 2 to 3 hours before the procedure. B. Abstain from sexual relations for 24 hours prior to the procedure. C. Do not use lotions, creams, or powder on your breasts before the study. D. Wear a supportive bra and bring a breast pad for use after testing.

C. Do not use lotions, creams, or powder on your breasts before the study.

2. What does the nurse expect was documented as the presenting symptom for a female client admitted with uterine leiomyomas? A. Foul-smelling vaginal discharge B. Intermittent abdominal pain C. Heavy vaginal bleeding D. Urinary stress incontinence

C. Heavy vaginal bleeding

When the nurse explains to a group of women that annual screening mammograms are not recommended for women under 40 years of age, what is the underlying rationale for this decision? A. Breast tumors are uncommon among women under the age of 40 years. B. The amount of radiation exposure outweighs the benefit for women of child-bearing age. C. In younger women there is little difference in the density of normal tissue and malignant tumors. D. In younger women the tumors are likely too small to be detected by mammography.

C. In younger women there is little difference in the density of normal tissue and malignant tumors.

While managing care for a client with chronic kidney disease, which action does the registered nurse (RN) plan to delegate to assistive personnel (AP)? (Select all that apply.) Select all that apply. Explain the components of a low-sodium diet. Document the amount the client drinks throughout the shift. Auscultate the client's lung sounds every 4 hours. Check the arteriovenous (AV) fistula for a thrill and bruit. Obtain the client's prehemodialysis weight.

Document the amount the client drinks throughout the shift. Obtain the client's prehemodialysis weight. Actions the RN delegates to the UAP include: obtaining the client's weight and documenting oral fluid intake. These are routine tasks that can be performed by a UAP.Assessment skills (checking the AV fistula and auscultating lung sounds) and client education (explaining special diet) require more education and are in the legal scope of practice of the RN.

A client with cancer of the cervix has an intracavitary radioactive sealed implant in place. Which of the following interventions should be incorporated into the plan of care for this client? Ensure that visitors maintain a six-foot distance from the source Dispose of body fluids in special marked containers Cohort two clients who have implanted radiation therapy in the same room Maintain the client on a high protein, high calorie diet while the implant is in place

Ensure that visitors maintain a six-foot distance from the source

Which of the following lab results would indicate to the nurse that the patient who is in the acute recovery phase from a burn injury is at risk of developing the most serious complication which occurs at this phase? C-Reative Protein 3.2 mg/dL Potassium 4.0 WBC = 14,000 mm3 Platelets = 140,000

WBC = 14,000 mm3

2. The nurse caring for hospitalized clients includes which actions on their care plans to reduce the possibility of the clients developing shock? (Select all that apply.) a. Assessing and identifying clients at risk b. Monitoring the daily white blood cell count c. Performing proper hand hygiene d. Removing invasive lines as soon as possible e. Using aseptic technique during procedures f. Limiting the client's visitors until more stable

a. Assessing and identifying clients at risk c. Performing proper hand hygiene d. Removing invasive lines as soon as possible e. Using aseptic technique during procedures Assessing and identifying clients at risk for shock is probably the most critical action the nurse can take to prevent shock from occurring. Proper hand hygiene, using aseptic technique, and removing IV lines and catheters are also important actions to prevent shock. Monitoring laboratory values does not prevent shock but can indicate a change. Limiting the client's visitors is not a caring action. The nurse would ensure they perform proper hand hygiene on entering and leaving the room and that visitors are not ill themselves.

2. A client is interested in learning about the risk factors for prostate cancer. Which factors does the nurse include in the teaching? (Select all that apply.) a. Family history of prostate cancer b. Smoking c. Obesity d. Advanced age e. Eating too much red meat f. Race

a. Family history of prostate cancer d. Advanced age e. Eating too much red meat f. Race Advanced family history of prostate cancer, age, a diet high in animal fat, and race are all risk factors for prostate cancer. Smoking and obesity are not known risk factors.

3. A client came to the clinic with erectile dysfunction. What are some possible causes of this condition that the nurse could discuss with the client during history taking? (Select all that apply.) a. Recent prostatectomy b. Long-term hypertension c. Diabetes mellitus d. Hour-long exercise sessions e. Consumption of beer each night

a. Recent prostatectomy b. Long-term hypertension c. Diabetes mellitus e. Consumption of beer each night Organic erectile dysfunction can be caused by surgical procedures, hypertension and its treatment, diabetes mellitus, and alcohol consumption. There is no evidence that exercise is related to this problem.

8. A client arrives in the emergency department after being in a car crash with fatalities. The client has a nearly amputated leg that is bleeding profusely. What action by the nurse takes priority? a. Apply direct pressure to the bleeding. b. Ensure the client has a patent airway. c. Obtain a pulse oximetry reading d. Start two large-bore IV catheters.

b. Ensure the client has a patent airway. Airway is the priority, followed by breathing (pulse oximetry) and circulation (IVs and direct pressure).

The nurse is caring for a postoperative client who reports pain in the shoulder blades following laparoscopic cholecystectomy surgery. Which direction does the nurse give to the nursing assistant to help relieve the client's pain? "Remind the client to cough and deep breathe every hour." "Encourage the client to take sips of hot tea or broth." "Apply a cold compress to the client's back." "Ambulate the client in the hallway."

"Ambulate the client in the hallway." The client who has undergone a laparoscopic cholecystectomy may report free air pain because of retention of carbon dioxide in the abdomen. The nurse assists the client with early ambulation to promote absorption of the carbon dioxide. Coughing and deep breathing are important postoperative activities, but they are not related to discomfort from carbon dioxide. Cold compresses and drinking tea would not be helpful.

The nurse has given a client information about the diagnosis of testicular cancer. Which of these statements, if made by the client, would indicate that the client has the correct understanding of the information? "Testicular cancer always results in sterility." "Cryptorchidism is a major risk factor for the development of testicular cancer." "Pain in the testicle is a major symptom of this disease." "This tumor is diagnosed with the PSA test."

"Cryptorchidism is a major risk factor for the development of testicular cancer."

The nurse is conducting a reproductive assessment of a young adult client. What assessment question will the nurse ask? (Select all that apply.) Select all that apply. "Have you had any sexually transmitted infections?" "What changes would you like to see in your appearance?" "If you engage in sexual activities, do you practice 'safe' sex?" "Are you currently experiencing any reproductive concerns?" "When did you first start menstruating?"

"Have you had any sexually transmitted infections?" "If you engage in sexual activities, do you practice 'safe' sex?" "Are you currently experiencing any reproductive concerns?" "When did you first start menstruating?" Asking the client about a history of sexually transmitted infections is a question included in the health perception/health management pattern for performing a reproductive assessment. If the answer is "yes," the nurse continues with "When?" and "What type?" The nurse will ask if the client has any current reproductive concerns that should be noted. It is important to assess, if the client is sexually active, that he or she practices (and understands) "safe" sex. This might include the use of condoms, being tested for human immunodeficiency virus, and other measures to keep from acquiring sexually transmitted infections.Although the nurse might inquire whether a client has experienced changes in his or her body appearance or function, asking about changes the client might want to see is not important in doing a reproductive assessment.

The nurse is educating a 22-year-old female about the Papanicolaou (Pap) test. Which client statement indicates that further teaching is needed? (Select all that apply.) Select all that apply. "The specimen will be sent to a laboratory for evaluation." "I need to have yearly Pap tests until I am 30 years old." "I can have sexual intercourse the night before the test." "Pap smears help detect precancerous and cancerous cells." "I will douche the morning before I have the Pap test performed."

"I can have sexual intercourse the night before the test." "I will douche the morning before I have the Pap test performed." The client must not have sexual intercourse, douche, or use vaginal medications or deodorants for at least 24 hours before the test. These all may interfere with test interpretation; therefore, these misconceptions require further nursing teaching.The other client statements are accurate, and do not require further nursing teaching. Annual screening is recommended to 30 years of age with the conventional Pap test. The Pap smear is a cytologic study that is effective in detecting precancerous and cancerous cells in the cervix. The specimen-containing slides from a Pap smear are sent to a laboratory for evaluation.

The nurse is performing discharge teaching for a client who is recovering from a total abdominal hysterectomy (TAH). Which client statement indicates a need for clarification? "I will no longer have menstrual periods." "I will need to find a new form of birth control." "I must take my temperature twice a day for the first few days after surgery." "I cannot jog for 2 to 6 weeks."

"I will need to find a new form of birth control." The client who has had a TAH can no longer become pregnant. Therefore, birth control is no longer necessary.The client must avoid jogging, aerobic exercise, participating in sports, or any strenuous activity for 2 to 6 weeks. The client must take her temperature twice a day for the first few days after surgery as a precaution to monitor for infection. The client will no longer have a period, although she may have some vaginal discharge for a few days after going home.

Which response by a client indicates an understanding of measures to facilitate the flow of peritoneal dialysate fluid? "I will keep the drainage bag at the level of my abdomen." Flushing the catheter is needed with each exchange." "I will take my stool softeners every day." Warmed dialysate infusion increases the speed of flow."

"I will take my stool softeners every day."

26. Which statement indicates that a client understands how to take an alpha blocker drug for treatment of benign prostatic hyperplasia (BPH)? "I should take this medication at the same time as the diuretic." "As soon as I get up, I will take my medication." "I'll take my medication at bedtime." "I'll take my medication on an empty stomach."

"I'll take my medication at bedtime." Bedtime dosing should decrease the risk of hypotension with an alpha blocker drug. Giving the medication during the day will increase the client's risk of experiencing weakness, lightheadedness, and dizziness.

A client had a total abdominal hysterectomy 2 days ago and is to be discharged on antibiotics. What does the nurse include in discharge teaching? "Stop the antibiotic if you feel nauseated because it will lose its effectiveness." "You will need to take the drug until your incision heals." "After your first day at home, you can stop them if you do not have a fever." "It is important to take them as directed until they are all gone."

"It is important to take them as directed until they are all gone." The client must finish the entire course of antibiotics and take them even after she has diminished signs or symptoms. This is a fundamental principle of antibiotic administration.The client would never be instructed to stop a course of antibiotics. In cases in which clients are unable to take an antibiotic (due to nausea or another problem), an alternative antibiotic will be prescribed. The client's incision is anticipated to be healed by the time that the antibiotic course is completed.

An older adult client reports uncomfortable sexual intercourse associated with vaginal dryness. Which nursing response is appropriate? "You may need to have additional pelvic examinations." "Products such as water-soluble lubricants may be helpful." "Be sure to tell your primary health care provider about this." "Let me teach you how to do Kegel exercises."

"Products such as water-soluble lubricants may be helpful." Information about vaginal estrogen therapy and water-soluble lubricants need to be provided to the older adult client with vaginal dryness.There is no need to inform the primary health care provider because vaginal dryness is a normal change associated with aging. Additional pelvic examinations are not indicated for this client. Kegel exercises are used for clients with incontinence.

The health care provider has completed a cervical biopsy on a client. Which postprocedure teaching will the nurse provide? "Use the antiseptic solution rinses to clean your perineum." "Abstain from intercourse for 24 hours after the procedure." "Rest for at least 12 hours after the procedure." "There is no limit on activity or weight-lifting."

"Use the antiseptic solution rinses to clean your perineum." The client must keep the perineum clean and dry by using antiseptic solution rinses as directed by the primary health care provider, and needs to change pads frequently.The client is told not have intercourse or lift heavy objects for about 2 weeks after the procedure. The client also needs to rest for 24 hours after the procedure.

A client is being treated for kidney failure. Which nursing statement encourages the client to express his or her feelings? "All of this is new. What can't you do?" "How are you doing this morning?" "Are you afraid of dying?" "What concerns do you have about your kidney disease?"

"What concerns do you have about your kidney disease?" Asking the client about any concerns regarding your disease is an open-ended statement and specific to the client's concerns.Asking the client to explain what he or she can't do implies inadequacy on the client's part. Asking the client if he or she is afraid of dying is too direct and would likely cause the client to be anxious. Asking the client how he or she is doing is too general and does not encourage the client to share thoughts on a specific topic.

What teaching does the nurse provide to a client prior to a mammogram? "Do not eat anything for 12 hours before having a mammogram." "You must not wear deodorant the day of your mammogram." "You will not feel any discomfort during the mammography procedure." "A mammogram will x-ray the hard tissue of your breasts."

"You must not wear deodorant the day of your mammogram." Remind the client not to use creams, lotions, powders, or deodorant on the breasts or underarms before the mammogram, because these products may be visible on the mammogram and lead to misdiagnosis.Mammography is an x-ray of the soft tissue of the breast. Dietary restrictions are not necessary before a mammogram. The client may experience some temporary discomfort when the breast is compressed during positioning and the test itself.

A client is preparing to have a hepatobiliary scan (HIDA scan). What health teaching would the nurse include about what the client can expect during the test? "This test measures how inflamed your gallbladder and liver may be." "You may eat and drink as much as you'd like before you have this test." "You will have to lie still for some time while the camera is very close to your body." "I need to know if you are allergic to shellfish because the contrast will be iodine-based."

"You will have to lie still for some time while the camera is very close to your body." The HIDA scan requires the injection of radioactive medium which is given about 20 minutes before a large camera is positioned very close to the body. The camera moves to assess for biliary flow and to determine if any obstruction is present.

1. Which statements about shock are true? Select all that apply. A. Affects all body organs B. Occurs only in the acute care setting C. Is a whole-body response to tissue hypoxia D. Results in widespread abnormal cellular metabolism E. Is classified as a disease rather than a discreet disorder F. May occur in older clients in response to urinary tract infections

1. A, C, D, F Shock is widespread abnormal cellular metabolism that occurs when gas exchange with oxygenation and tissue perfusion needs are insufficient to maintain cell function. It is a condition rather than a disease and is the "whole-body" response that occurs with tissue hypoxia. All body organs are affected by shock and either work harder to adapt and compensate for reduced gas exchange or perfusion or fail to function because of hypoxia. Urinary tract infections that enter the bloodstream (urosepsis) is a common cause of shock in older clients. Shock can occur in any setting.

Anurse is caring for a patient with acute pancreatitis who is experiencing abdominal pain. What is the intervention the nurse should do to manage this pain? 1. Administer IV opioids through patient-controlled analgesia (PCA) 4. Provide small frequent meals 2. Place the patient in semi-Fowler's position with HOB elevated to provide comfort 3. Administer oral opioids such as morphine sulfate every 4 hours

1. Administer IV opioids through patient-controlled analgesia (PCA)

The nurse is caring for a client hospitalized for heart failure exacerbation and suspects the client may be entering a state of shock. The nurse plans for which intervention as the priority for this client? 1. Administration of dopamine 2. Administration of whole blood 3. Administration of intravenous Fluids 4. Administration of packed red blood cells

1. Administration of dopamine The client in this question is likely experiencing cardiogenic shock secondary to heart failure exacerbation. It is important to note that if the shock state is cardiogenic in nature, the infusion of volume-expanding fluids may result in pulmonary edema; therefore, restoration of cardiac func- tion is the priority for this type of shock. Cardiotonic medica- tions such as digoxin, dopamine, or norepinephrine may be administered to increase cardiac contractility and induce vaso- constriction. Whole blood, intravenous fluids, and packed red blood cells are volume-expanding fluids and may further complicate the client's clinical status; therefore, they should be avoided.

A client being hemodialyzed suddenly becomes short of breath and complains of chest pain. The client is tachycardic, place, and anxious and the nurse suspects air embolism. What are the priority nursing actions? Select all that apply. 1. administer oxygen to the client 2. continue dialysis at a slower rate after checking the lines for air 3. notify the HCP and RRT 4. stop dialysis, and turn the client on the left side with head lower than feet 5. bolus the client with 500 mL of normal saline to break up the air embolus

1. administer oxygen to the client 3. notify the HCP and RRT 4. stop dialysis, and turn the client on the left side with head lower than feet Rationale: if the client experiences air embolus during hemodialysis, the nurse should terminate dialysis immediately, position the client so the air embolus is in the right side of the heart, notify the HCP and RRT, and administer oxygen as needed. Slowing the dialysis treatment or giving an intravenous bolus will not correct the air embolism or prevent complications

The nurse monitoring a client receiving peritoneal dialysis notes that the client's outflow is less than the inflow. Which actions should the nurse take? Select all that apply. 1. check the level of the drainage bag 2. reposition the client to his or her side 3. contact the HCP 4. place the client in good body alignment 5. check the peritoneal dialysis system for kinks 6. increase the flow rate of the peritoneal dialysis solution

1. check the level of the drainage bag 2. reposition the client to his or her side 4. place the client in good body alignment 5. check the peritoneal dialysis system for kinks Rationale: if outflow drainage is inadequate, the nurse attempts to stimulate outflow by changing the client's position. Turning the client to the side or making sure that the client is in good body alignment may assist with outflow drainage. The drainage bag needs to be lower than the client's abdomen to enhance gravity drainage. The connecting tubing and peritoneal dialysis system are also checked for kinks or twisting and the clamps on the system are checked to ensure that they are open. There is no reason to contact the HCP. Increasing the flow rate should not be done and also is not associated with the amount of outflow solution

The nurse is reviewing a client's record and notes that HCP has documented that the client has chronic renal disease. On review of the lab results, the nurse most likely would expect to note which finding? 1. elevated creatinine level 2. decreased hemoglobin level 3. decreased RBC count 4. increased number of WBCs in the urine

1. elevated creatinine level Rationale: the creatinine level is the most specific lab test to determine renal function. The creatinine level increases when at least 50% of renal function is lost. A decreased hemoglobin level and RBC count are associated with anemia or blood loss and not specifically with decreased renal function. Increased white blood cells in the urine are noted with UTI

The nurse discusses plans for future treatment options with a client with symptomatic polycystic kidney disease. Which treatment should be included in this discussion? Select all that apply. 1. hemodialysis 2. peritoneal dialysis 3. kidney transplant 4. bilateral nephrectomy 5. intense immunosuppression therapy

1. hemodialysis 3. kidney transplant 4. bilateral nephrectomy Rationale: polycystic kidney disease is a genetic familial disease in which the kidney enlarge with cysts that rupture and scar the kidney, eventually resulting in end stage renal disease. Treatment options include hemodialysis or kidney transplant. Clients usually undergo bilateral nephrectomy to remove the large, painful, cyst-filled kidneys. Peritoneal dialysis is not a treatment option due to the infected cysts. The condition does not respond to immunosuppression

A client is admitted to the ED following a fall from a horse and the HCP prescribes insertion of a urinary catheter. While preparing for the procedure, the nurse notes blood at the urinary meatus. The nurse should take which action? 1. notify the HCP before performing the catheterization 2. use a small-sized catheter and an anesthetic gel as a lubricant 3. administer parenteral pain medication before inserting the catheter 4. clean the meatus with soap and water before opening the catheterization kit

1. notify the HCP before performing the catheterization Rationale: the presence of blood at the urinary meatus may indicate urethral trauma or disruption. The nurse notifies the HCP, knowing that the client should not be catheterized until the cause of the bleeding is determined by diagnostic testing. The other options include performing the catheterization procedure and therefore are incorrect

The nurse is assessing the patency of a client's left arm arteriovenous fistula prior to initiating hemodialysis. Which finding indicates that the fistula is patent? 1. palpation of a thrill over the fistula 2. presence of a radial pulse in the left wrist 3. visualization of enlarged blood vessels at the fistula site 4. capillary refill less than 3 seconds in the nail beds of the fingers on the left hand

1. palpation of a thrill over the fistula Rationale: the nurse assesses the patency of the fistula by palpating for the presence of a thrill or auscultating for a bruit. The presence of a thrill and bruit indicate patency of the fistula. Enlarged visible blood vessels at the fistula site are a normal observation but are not indicative of fistula patency. Although the presence of a radial pulse in the left wrist and capillary refill less than 3 seconds in the nail beds of the fingers on the left hand indicate adequate circulation to the hand, they do not assess fistula patency

A client with acute kidney injury has a serum potassium level of 7.0 mEq/L. The nurse should plan which actions as a priority? Select all that apply. 1. place the client on a cardiac monitor 2. notify the HCP 3. put the client on NPO status except for ice chips 4. review the client's medications to determine if any contain or retain potassium 5. allow an extra 500 mL of intravenous fluid intake to dilute the electrolyte concentration

1. place the client on a cardiac monitor 2. notify the HCP 4. review the client's medications to determine if any contain or retain potassium Rationale: the normal potassium level is 3.5-5.0 mEq/L. A potassium level of 7.0 is elevated. The client with hyperkalemia is at risk of developing cardiac dysrhythmias and cardiac arrest. Because of this, the client should be placed on a cardiac monitor. The nurse should notify the HCP and also review medications to determine if any contain potassium or are potassium retaining. The client does not need to be put on NPO status. Fluid intake is not increased because it contributes to fluid overload and would not affect the serum potassium level significantly

10. Which laboratory values in a client with hypovolemic shock will the nurse associate with the progressive stage of shock? Select all that apply. A. Arterial blood pH 7.32 B. Serum lactate 9 mg/dL (1.03 mmol/L) C. Sodium 147 mEq/L (mmol/L) D. Blood urea nitrogen 15 mg/dL E. Potassium 6.3 mEq/L (mmol/L) F. Neutrophil count 5,000/mm3 (5 x 109/L)

10. A, B, E Laboratory indicators of the progressive stage of hypovolemic shock are a low blood pH, along with rising lactic acid and potassium levels. The sodium level rises during the compensatory stage. The level listed is only slightly higher than normal. The BUN is within the normal range. The neutrophil count is within the normal range.

11. Which actions are most appropriate for the nurse to take first when a client with blunt trauma to the abdomen who has been NPO for several hours now reports thirst and anxiety? Select all that apply. A. Obtain an order for a stat hematocrit and hemoglobin. B. Get the client a few ice chips or a moistened swab. C. Compare current vital signs to baseline. D. Check for obvious blood in the urine. E. Measure abdominal girth. F. Increase the IV rate.

11. C, E A client with blunt trauma to the abdomen may have internal bleeding and is at risk for shock. Thirst and anxiety are subjective symptoms of shock. The nurse would assess for other indications of shock or internal bleeding by first assessing vital signs and comparing them to the client's baseline and measure abdominal girth. If the vital signs or abdominal assessment are consistent with shock, the nurse will notify the Rapid Response Team who would order a stat hematocrit and hemoglobin. Although keeping an IV open and available is important, until the source of bleeding is found and corrected, the IV rate may not be increased in order to slow any hemorrhage. The kidneys are located on the back wall and not usually directly involved in abdominal trauma. Until assessment is complete, the client is kept NPO.

17. Which actions are priorities for the nurse to perform to prevent harm for a client with hypovolemic shock who is receiving an infusion of dobutamine? Select all that apply. A. Assessing hourly urine output B. Assessing for chest pain throughout the infusion C. Covering the infusion bag to protect it from light D. Measuring blood pressure at least every 15 minutes E. Ensuring the drug is infused only with Ringer's lactate F. Checking the infusion site every 30 minutes for extravasation

17. B, D, F Dobutamine is a beta-adrenergic agonist and a positive inotropic agent that can improve cardiac contractility. The increased contractility increases cardiac muscle oxygen consumption, which may not be met during shock, and can lead to angina or myocardial infarction. Although the main action is in the heart muscle, it also acts on blood vessels and can cause a transient hypotension from vascular dilation. When doses are too high, vasoconstriction can occur and is an indication of overdose. In addition, if extravasation occurs local vasoconstriction can cause tissue damage. The drug is not light sensitive and can be run with other crystalloids, not just Ringer's lactate. Assessing urine output is not the priority when administering dobutamine.

9. Which client parameters will the nurse report to the health care provider as consistent with the quick Sequential Organ Failure Assessment (qSOFA) indicating the possible presence of sepsis or septic shock? Select all that apply. A. Core body temperature 100°F (37.8°C) B. Eyes are open but does not respond to questions C. Respiratory rate of 28 breaths/min D. SpO2 is 94% primary E. Systolic blood pressure of 92 mm Hg F. Urine output of 18 mL/hr

19. B, C, E The quick Sequential Organ Failure Assessment (qSOFA) can quickly alert clinicians to the need for further assessment for organ dysfunction. This assessment has three parameters and clients are assigned one point for each abnormal parameter. Abnormal parameters include: Systolic blood pressure ≤ 100 mm Hg Respiratory rate ≥ 22 breaths/min Any change in mental status

2. A nurse is reviewing discharge instructions with a client who had spontaneous passage of a calcium phosphate renal calculus. Which of the following instructions should the nurse include in the teaching? (select all that apply.) A. Limit intake of food high in animal protein. B. reduce sodium intake. C. strain urine for 48 hr. d. report burning with urination to the provider. E. increase fluid intake to 3 L/day.

2. A. CORRECT: The client should limit the intake of food high in animal protein, which contains calcium phosphate. B. CORRECT: The client should limit intake of sodium, which affects the precipitation of calcium phosphate in the urine. C. The client does not need to continue straining urine once the calculus has passed. d. CORRECT: The client should report burning with urination to the provider because this can indicate a urinary tract infection. e. CORRECT: The client should increase fluid intake to 2 to 3 L/day. A decrease in fluid intake can cause dehydration, which increases the risk of calculi formation. NCLEX® Connection: Reduction of Risk Potential, Therapeutic Procedures

The nurse is instructing a client with diabetes mellitus about peritoneal dialysis. The nurse tells the client that it is important to maintain the prescribed dwell time for the dialysis because of the risk of which complication? 1. peritonitis 2. hyperglycemia 3. hyperphosphatemia 4. disequilibrium syndrome

2. hyperglycemia Rationale: an extended dwell time increases the risk of hyperglycemia in the client with DM as a result of absorption of glucose from the dialysate and electrolyte changes. Diabetic clients may require extra insulin when receiving peritoneal dialysis. Peritonitis is a risk associated with breaks in aseptic technique. Hyperphosphatemia is an electrolyte imbalance that occurs with renal dysfunction. Disequilibrium syndrome is a complication associated with hemodialysis

A client with chronic kidney disease returns to the nursing unit following a hemodialysis treatment. On assessment, the nurse notes that the client's temperature is 101.2 F. Which nursing action is most appropriate? 1. encourage fluid intake 2. notify the HCP 3. continue to monitor vital signs 4. monitor the site of the shunt for infection

2. notify the HCP Rationale: a temperature of 101.2 F is significantly elevated and may indicate infection. The nurse should notify the HCP. Dialysis clients cannot have fluid intake encouraged. Vital signs and the shunt site should be monitored, but the HCP should be notified first

23. Which symptom in a client with sepsis does the nurse consider a late indication of septic shock? A. Warm skin B. Bounding pulse C. Severe hypotension D. Decreased urine output

23. C. Severe hypotension Late in septic shock, the client has hypovolemia and greatly decreased cardiac output with severe hypotension. Decreased urine output is an earlier indicator of shock. Although earlier in sepsis the skin is warm from vasodilation, in late septic shock the skin is cool, clammy, and cyanotic. The pulse is weak and thready.

24. How will the nurse interpret a change in the white blood cell (WBC) count of a client with sepsis in which the band neutrophil count is increasing and the segmented neutrophil count is decreasing? A. Antibiotic therapy is successful. B. The infection is becoming worse. C. The client is allergic to the drug therapy. D. Disseminated intravascular coagulation (DIC) is now present.

24. B. The infection is becoming worse. When the band neutrophil count is increasing and the segmented neutrophil count is decreasing, a left shift is occurring, in which the bone marrow is becoming depleted of mature neutrophils that can fight the infection. The infection is getting worse, indicating that antibiotic therapy is not effective and the client's ability to fight infection is greatly decreased. Changes in these specific WBCs are not associated with an allergic reaction or DIC.

25. When sepsis is diagnosed in a client, when will the nurse initiate the prescribed antibiotic therapy? A. Within the first hour after diagnosis B. Within the first 24 hours after diagnosis C. After the results of blood cultures are known D. When blood lactate levels have increased to 9 mg/dL (1.03 mmol/L)

25. A. Within the first hour after diagnosis Antibiotic therapy is initiated as soon as possible, preferably within the first hour after diagnosis, even if blood cultures have not been obtained. It is not delayed until after blood culture results are known. Lactate levels are used to diagnose the condition, not guide the timing of antibiotic therapy.

27. What is the nurse's best first action when a client who has sepsis is found to have a blood glucose level of 310 mg/dL? A. Check the electronic health record to determine when the last dose of antidiabetic drug was given. B. Ask the family how long the client has had diabetes. C. Notify the primary health care provider. D. Document the finding as the only action.

27. C. Notify the primary health care provider. Sepsis alone can elevate blood glucose levels in any client. Levels above 180 mg/dL are associated with poor outcomes and management must be started immediately. The nurse's best first action is to notify the primary health care provider immediately.

The nurse has taught the client about upcoming endoscopic retrograde cholangiopancreatography (ERCP) procedure. The nurse determines that the client needs further information if the client makes which statement? 1. "i know i must sign the consent form" 2. "i hope the throat spray keeps me from gagging" 3. "im glad i dont have to lie still for this procedure" 4. "im glad some intravenous medications will be given to relax me"

3. "im glad i dont have to lie still for this procedure"

3. A nurse is teaching a client who is scheduled for extracorporeal shock wave lithotripsy (esWL). Which of the following statements by the client indicates understanding of the teaching? A. "i will be fully awake during the procedure." B. "Lithotripsy will reduce my chances of having stones in the future." C. "i will report any bruising that occurs to my doctor." D. "straining my urine following the procedure is important."

3. A. The client receives moderate (conscious) sedation for this procedure. The client is not fully awake. B. Lithotripsy does not decrease the recurrence rate of renal calculi. The procedure breaks the calculi into fragments so they will pass into urine. C. Bruising is an expected finding following lithotripsy and does not need to be reported to the provider. d. CORRECT: A client is instructed to strain urine following lithotripsy to verify that the calculi have passed. NCLEX® Connection: Basic Care and Comfort, Nutrition and Oral Hydration

The nurse is assessing a client 24 hours following a cholecystectomy. The nurse notes that the T-tube has drained 750 mL of green-brown drainage since the surgery. Which nursing intervention is most appropriate? 1. Clamp the T-tube 2. Irrigate the T-tube 3. Document the findings 4. Notify the HCP

3. Document the findings

client arrives at the emergency department with complaints of low abdominal pain and hematuria. The client is afebrile. The nurse next assess the client to determine a history of which condition? 1. pyelonephritis 2. glomerulonephritis 3. trauma to the bladder or abdomen 4. renal cancer in the client's family

3. trauma to the bladder or abdomen Rationale: bladder trauma or injury should be considered or suspected in the client with low abdominal pain and hematuria. Glomerulonephritis and pyelonephritis would be accompanied by fever and are thus not applicable to the client described in this question. Renal cancer would not cause pain that is felt in the low abdomen; rather, the pain would be in the flank area

4. A nurse is caring for a client who has a left renal calculus and an indwelling urinary catheter. Which of the following assessment findings is the priority for the nurse to report to the provider? A. Flank pain that radiates to the lower abdomen B. Client report of nausea C. Absent urine output for 1 hr d. Blood WBC count 15,000/mm3

4. A. Flank pain radiating to the lower abdomen is a finding associated with renal calculi, but there is another finding that is a greater risk to the client. B. Client report of nausea is a finding associated with renal calculi, but there is another finding that is a greater risk to the client. C. CORRECT: The greatest risk to this client is damage to the kidney resulting from obstruction of urine flow by the renal calculus. Therefore, the priority finding to report to the provider is anuria. d. An elevated serum WBC is a finding associated with renal calculus and can indicate a urinary tract infection, but there is another finding that is a greater risk to the client. NCLEX® Connection: Physiological Adaptation, Unexpected Response to Therapies

A client is admitted to the hospital with a diagnosis of benign prostatic hyperplasia, and a transurethreal resection of the prostate is performed. Four hours after surgery, the nurse takes the client's vital signs and empties the urinary drainage bag. Which assessment finding indicates the need to notify the HCP? 1. red bloody urine 2. pain rated as 2 on a 0-10 pain scale 3. urinary output of 200 mL higher than intake 4. BP 100/50, pulse 130 bpm

4. BP 100/50, pulse 130 bpm Rationale: frank bleeding (arterial or venous) may occur during the first day after surgery. A urinary output of 200 mL more than intake is adequate. A client pain rating of 2 on a 0-10 scale indicates adequate pain control. A rapid pulse with a low BP is a potential sign of excessive blood loss. The HCP should be notified

A male client ha a tentative diagnosis of urethritis. The nurse should assess the client for which manifestation of the disorder? 1. hematuria and pyuria 2. dysuria and proteinuria 3. hematuria and urgency 4. dysuria and penile discharge

4. dysuria and penile discharge Rationale: urethritis in the male client often results from chlamydial infection and is characterized by dysuria, which is accompanied by a clear to mucopurulent discharge. Because this disorder often coexists with gonorrhea, diagnostic tests are done for both and include culture and rapid assays. Hematuria is not associated with urethritis. Proteinuria is associated with kidney dysfunction

A client admitted to the hospital with a suspected diagnosis of acute pancreatitis is being assessed by the nurse. Which assessment findings would be consistent with acute pancreatitis? Select all that apply. 1. diarrhea 2. black tarry stools 3. hyperactive bowel sounds 4. gray blue color at the flank 5. abdominal guarding and tenderness 6. left upper quadrant pain with radiation to the back

4. gray blue color at the flank 5. abdominal guarding and tenderness 6. left upper quadrant pain with radiation to the back

The client newly diagnosed with chronic kidney disease recently has begun hemodialysis. Knowing that the client is at risk for disequilibrium syndrome, the nurse should assess the client during dialysis for which associated manifestation? 1. hypertension, tachycardia, and fever 2. hypotension, bradycardia, and hypothermia 3. restlessness, irritability, and generalized weakness 4. headache, deteriorating level of consciousness, and twitching

4. headache, deteriorating level of consciousness, and twitching Rationale: disequilibrium syndrome is characterized by headache, mental confusion, decreasing level of consciousness, nausea, vomiting, twitching, and possible seizure activity. Disequilibrium syndrome is caused by rapid removal of solutes from the body during hemodialysis. At the same time, the blood-brain barrier interferes with the efficient removal of wastes from brain tissue. As a result, water does into cerebral cells because of the osmotic gradient, causing increased intracranial pressure and onset of symptoms. The syndrome most often occurs in clients who are new to dialusis and is prevented by dialyzing for shorter times or at reduced blood flow rates. Tachycardia and fever are associated with infection. Generalized weakness is associated with low BP and anemia. Restlessness and irritability are not associated with disequilibrium syndrome

A week after kidney transplantation, a client develops a temperature of 101 F, the BP is elevated, and there is tenderness over the transplanted kidney. The serum creatinine is rising and urine output is decreased. The x-ray indicates that the transplanted kidney is enlarged. Based on these assessment findings, the nurse anticipates which treatment? 1. antibiotic therapy 2. peritoneal dialysis 3. removal of the transplanted kidney 4. increased immunosuppression therapy

4. increased immunosuppression therapy Rationale: acute rejection most often occurs within 1 week after transplantation but can occur any time posttransplantation. Clinical manifestation include fever, malaise, elevated WBC count, acute hypertension, graft tenderness, and manifestations of deteriorating renal function. Treatment consists of increasing immunosuppressive therapy. Antibiotics are used to treat infection. Peritoneal dialysis cannot be used with a newly transplanted kidney due to the recent surgery. Removal of the transplanted kidney is indicated with hyper acute rejection, which occurs within 48 hours of the transplant surgery

A client with severe back pain and hematuria is found to have hydronephrosis due to urolithiasis. The nurse anticipates which treatment will be done to relieve the obstruction? Select all that apply. 1. peritoneal dialysis 2. analysis of the urinary stone 3. intravenous opioid analgesics 4. insertion of a nephrostomy tube 5. placement of a ureteral stent with ureteroscopy

4. insertion of a nephrostomy tube 5. placement of a ureteral stent with ureteroscopy Rationale: urolithiasis is the condition that occurs when a stone forms in the urinary system. Hydronephrosis develops when the stone has blocked the ureter and urine backs up and dilates and damages the kidney. Priority treatment is to allow the urine to drain and relieve the obstruction in the ureter. This is accomplished by placement of a percutaneous nephrostomy tube to drain urine from the kidney and placement of a ureteral stent to keep the ureter open. Peritoneal dialysis is not needed since the kidney is functioning. Stone analysis will be done later when the stone has been retrieved and analyzed. Opioid analgesics are necessary for pain relief but do not treat the obstruction

The nurse is performing an assessment on a client who has returned from the dialysis unit following hemodialysis. The client is complaining of headache and nausea and is extremely restless. Which is the priority nursing action? 1. monitor the client 2. elevate the head of the bed 3. assess the fistula site and dressing 4. notify the HCP

4. notify the HCP Rationale: disequilibrium syndrom may be caused b y rapid removal of solutes from the body during hemodialysis. These changes can cause cerebral edema that leads to increased intracranial pressure. The client is exhibiting early signs and symptoms of disequilibrium syndrome and appropriate treatments with anticonvulsive medications and barbiturates may be necessary to prevent a life-threatening situation. The HCP must be notified. Monitoring the client, elevating the head of bed, and assessing the fistula site are correct actions, the priority action is to notify the HCP

A hemodialysis client with a left arm fistula is at risk for arterial steal syndrome. The nurse should assess for which manifestations of this complication? 1. warmth, redness, and pain in the left hand 2. ecchymosis and audible bruit over the fistula 3. edema and reddish discoloration of the left arm 4. pallor, diminished pulse, and pain in the left hand

4. pallor, diminished pulse, and pain in the left hand Rationale: steal syndrome results from vascular insufficiency after creation of a fistula. The client exhibits pallor and a diminished pulse distal to the fistula. The client also complains of pain distal to the fistula, caused by tissue ischemia. Warmth and redness probably would characterize a problem with infection. Ecchymosis and a bruit are normal findings for a fistula

5. A nurse is completing discharge instructions with a client who has spontaneously passed a calcium oxalate calculus. To decrease the chance of recurrence, the nurse should instruct the client to avoid which of the following foods? (select all that apply.) A. red meat B. Black tea C. Cheese d. Whole grains e. spinach

5. A. A client who has renal calculi composed of calcium phosphate, struvite, uric acid, or cysteine should limit intake of animal protein. ✅B. CORRECT: A client who has renal calculi composed of calcium oxalate should avoid intake of black tea because it is a source of oxalate. C. A client who has renal calculi composed of calcium phosphate or struvite should limit intake of dairy products. d. A client who has renal calculi composed of struvite should limit intake of whole grains. ✅e. CORRECT: A client who has renal calculi composed of calcium oxalate should avoid intake of spinach because it is a source of oxalate. NCLEX® Connection: Reduction of Risk Potential,

5. Which conditions will the nurse consider as increasing any client's risk for hypovolemic shock? Select all that apply. A. Hypoglycemia B. Diuretic therapy C. Severe head injury D. Prolonged diarrhea E. Liver failure with ascites F. Continuous nasogastric suction G. Large draining abdominal wound

5. B, D, F, G In addition to loss of blood, hypovolemic shock can be caused by dehydration that decreases circulating blood volume. Conditions increasing the risk for dehydration-induced hypovolemic shock include fluid losses resulting from diuretic therapy, diarrhea, continuous nasogastric suction, and excessive wound drainage. Hypoglycemia does not result in fluid loss. Severe head injury can lead to neural-induced distributive shock, not hypovolemic shock. Liver failure with ascites is another cause of distributive shock caused by capillary leak.

6. Which changes in vital signs of a client in the early postoperative period indicates to the nurse that the client may be in the initial stage of hypovolemic shock? Select all that apply? A. Increased heart rate B. Increased respiratory rate C. Decreased systolic blood pressure D. Decreased urine output E. Increased diastolic blood pressure F. Increased pulse pressure

6. A, B, E The initial stage of shock is characterized by sympathetic nervous system compensation with vasoconstriction. Thus, the indicators of the initial stage of hypovolemic shock are subtle and include only increased heart and respiratory rates or a slight increase in diastolic blood pressure.

7. Which subjective symptom will the nurse expect to find in a client during the compensatory stage of hypovolemic shock? A. Thirst B. Hunger C. Headache D. Numbness of the fingers and toes

7. A. Thirst Hormonal compensation for hypovolemic shock includes secretion of antidiuretic hormone (ADH), renin, and aldosterone as a result of decreased tissue perfusion. Subjective changes of this compensation include thirst and anxiety.

Based on the assessment data, which client will the nurse identify as having a higher risk for development of sepsis and septic shock? (Select all that apply.) Select all that apply. A 40-year-old female with a history of a double lung transplant 4 years ago. A 41-year-old male client with a closed fracture of the femur. A 44-year-old female client with a history of anxiety and infertility. A 38-year-old male with HIV who has a low viral load. A 54-year-old female with breast cancer who is receiving chemotherapy. A 44-year-old male client who has a history of alcoholism and diabetes mellitus. An 86-year-old male with acute onset confusion.

A 40-year-old female with a history of a double lung transplant 4 years ago. A 38-year-old male with HIV who has a low viral load. A 54-year-old female with breast cancer who is receiving chemotherapy. A 44-year-old male client who has a history of alcoholism and diabetes mellitus. An 86-year-old male with acute onset confusion. While any person can develop sepsis, there are certain conditions that predispose clients to sepsis and septic shock. The 54-year-old female is at higher risk due to cancer and chemotherapy. The 86-year-old male is higher risk due to an age above 80 years. The 44 year old has a higher risk due to alcoholism and diabetes. The 38 year old has an increased risk due to immunosuppression and HIV. The 40 year old is at higher risk due to immunosuppression and transplant status. A closed fracture does not increase risk, nor does anxiety and infertility.

Which gynecologic client does the charge nurse assign to an LPN/LVN? (Select all that apply.) A 52 year old who just returned to the unit following a total abdominal hysterectomy. A 42 year old who had an abdominal hysterectomy whose primary health care provider wants to remove sutures at her bedside. A 48 year old who is receiving IV chemotherapy to treat stage II cervical cancer. A 23 year old who is nauseated after her laparotomy and needs to receive antiemetic drugs. A 34 year old who had a total hysterectomy for invasive cervical cancer and has a blood pressure (BP) of 88/54 mm Hg.

A 42 year old who had an abdominal hysterectomy whose primary health care provider wants to remove sutures at her bedside. A 23 year old who is nauseated after her laparotomy and needs to receive antiemetic drugs. The client who is nauseated after laparotomy and needs to receive antiemetic drugs and the client who had an abdominal hysterectomy and whose primary health care provider wants to remove sutures can be cared for by an LPN/LVN.The client with a total hysterectomy and low BP has a deteriorating status (dropping BP) and requires care provided by an RN. Also, the client who is receiving IV chemotherapy requires an RN to assess and care for. The client who has just returned from surgery must first be assessed by the RN for stability.

Which client does the RN assess first after receiving change-of-shift report? A 45 year old with a history of hypothyroidism who is scheduled for a hysterectomy and bladder suspension. A 54 year old with an anterior and posterior colporrhaphy who has an elevated heart rate and an oral temperature of 101.2° F (38.4° C). A 50 year old who is receiving morphine through a client-controlled analgesia (PCA) device after a hysterectomy and who rates her pain at a level 3 (0-10 scale). A 48 year old who is reporting abdominal pain and light vaginal spotting after an endometrial biopsy.

A 54 year old with an anterior and posterior colporrhaphy who has an elevated heart rate and an oral temperature of 101.2° F (38.4° C). The client with an anterior and posterior colporrhaphy with an elevated heart rate and fever is showing signs of postoperative infection and warrants frequent assessments. This information needs to be communicated to the surgeon as soon as possible.The client with a history of hypothyroidism who is scheduled for a hysterectomy and bladder suspension, the client with abdominal pain and light vaginal spotting after an endometrial biopsy, and the client receiving morphine through a PCA device with a pain level of 3 are not unusual cases and do not require rapid intervention by the nurse.

The nurse is caring for four clients. With which client does the nurse discuss prostate cancer screening? A 23 year old with a history of urinary tract infections A 33 year old who sustained an injury to the external genitaliac. A 46 year old with uncontrolled hypertension A 57 year old who has fathered four children

A 57 year old who has fathered four children A man who is between 55 and 69 years old should make an informed decision about whether to have prostate cancer screening. The risk increases for men who have a first-degree relative (brother, father) with the disease.A history of urinary tract infections, injury to the external genitalia, and uncontrolled hypertension are not risk factors for prostate cancer.

Four women phone the gynecology clinic about having new-onset of spotty vaginal bleeding. Which call does the RN decide to return first? A 34 year old with a history of multiple leiomyomas A 65 year old with no previous gynecologic problems A 48 year old who had an endocervical curettage yesterday A 23 year old using medroxyprogesterone acetate

A 65 year old with no previous gynecologic problems Vaginal bleeding in a postmenopausal woman is abnormal and may be an indication of serious problems such as endometrial cancer. Bleeding in the 23 year old using medroxyprogesterone acetate, the 34 year old with a history of multiple leiomyomas, and the 48 year old who had endocervical curettage yesterday is not unusual. The nurse will need to follow up with these clients after talking with the 65-year-old client.

22. Which clinical signs and symptoms will the nurse assess for when a client is admitted with a diagnosis of toxic shock syndrome (TSS)? Select all that apply. A. Itching B. Fever C. Macular rash D. Hypertension E. Myalgias F. Vaginal lesions

B. Fever C. Macular rash E. Myalgias

The nurse is assessing a client for reproductive health problems. What would be the priority assessments? (Select all that apply.) a. Bleeding b. Pain c. Sexual orientation d. Masses e. Discharge

A, B, D, E Bleeding, pain, masses, and discharge are common health problems that bring a client to a health care provider. Sexual orientation is not considered a health problem. Sexual activity should be assessed as part of the clients history.

4. Which client will the nurse recognize as having a higher risk for obstructive shock? A. 32-year-old with a pulmonary embolus B. 42-year-old with stable angina C. 52-year-old with chronic atrial fibrillation D. 72-year-old with a history of heart failure

A. 32-year-old with a pulmonary embolus Obstructive shock is caused by problems that impair the ability of the normal heart to pump effectively. The heart itself remains normal, but conditions outside the heart prevent either adequate filling of the heart or adequate contraction of the healthy heart muscle. Although the most common cause of obstructive shock is cardiac tamponade, other causes include arterial stenosis, pulmonary embolism, pulmonary hypertension, pericarditis, thoracic tumor, and tension pneumothorax. The other health problems listed are causes associated with cardiogenic shock.

Which actions will the nurse include in the postoperative care of a client who had a laparoscopy? Select all that apply. A. Administering oral analgesics for incisional pain B. Notifying the health care provider for postoperative shoulder pain C. Instructing the client to change the small adhesive bandage as needed D. Reassuring the client that most painful sensations disappear within 1 to 2 weeks E. Teaching the client to observe the incision for signs of infection or hematoma F. Reminding the client to avoid strenuous activity for 3 to 4 weeks after the procedure

A. Administering oral analgesics for incisional pain C. Instructing the client to change the small adhesive bandage as needed E. Teaching the client to observe the incision for signs of infection or hematoma

Which new-onset assessment findings in a client with Laennec cirrhosis indicates to the nurse that the client may be starting to have delirium tremens (DTs) from alcohol withdrawal? Select all that apply. A. Anxiety B. Tachycardia C. Hypotension D. Hypertension E. Cool, clammy skin F. Psychotic behavior

A. Anxiety B. Tachycardia D. Hypertension F. Psychotic behavior

Which assessment findings will the nurse expect in a client with late-stage liver cirrhosis whose total serum albumin level is low? Select all that apply. A. Ascites B. Hypotension C. Hyperkalemia D. Hyponatremia E. Dependent edema F. Decreased serum ammonia levels

A. Ascites B. Hypotension D. Hyponatremia E. Dependent edema

Which neuromuscular assessment change indicates to the nurse that a client who has late-stage liver cirrhosis now has encephalopathy? A. Asterixis B. Positive Chvostek sign C. Increased deep tendon reflex responses D. Decreased deep tendon reflex responses

A. Asterixis

Which precaution is most important for the nurse to instruct a client with cirrhosis and his or her family about continuing care in the home? A. Avoid taking acetaminophen or drinking alcohol. B. Maintain one-floor living to prevent excessive fatigue. C. Use cool baths to reduce the sensation of itching. D. Report any change in cognition to the health care provider.

A. Avoid taking acetaminophen or drinking alcohol.

Which findings will the nurse expect to see when assessing the scrotum of a client? Select all that apply. A. Contracts with exposure to cold B. Suspended below the pubic bone C. Skin of scrotum is lightly pigmented D. Sparse hair follicles E. Pouch skin is thin walled F. Warm compared to surrounding tissues

A. Contracts with exposure to cold B. Suspended below the pubic bone D. Sparse hair follicles E. Pouch skin is thin walled

Which symptoms in a client with cirrhosis and encephalopathy indicate to the nurse that the prescribed lactulose therapy is effective? Select all that apply. A. Decreased confusion B. Increased urine output C. Musty odor to the breath D. Two to three soft stools daily E. Lower serum bilirubin levels F. Lower serum ammonia levels

A. Decreased confusion D. Two to three soft stools daily F. Lower serum ammonia levels

What does the nurse expect that a 30-year-old female client is most likely to experience when she reports a history of blockages in her fallopian tubes? A. Difficulty conceiving B. Vaginal discharge C. Difficulty controlling weight D. Irregular menses

A. Difficulty conceiving

Which actions will the nurse instruct the client to avoid to prevent harm after having a colposcopy procedure? A. Do not douche, use tampons, and have sexual intercourse for 1 week after the procedure. B. Wear a perineal pad and expect bleeding with small clots for the first 24 hours. C. Do not drive or operate heavy machinery while taking the prescribed pain medication. D. Perform breast self-examination every month and report changes to the health care provider.

A. Do not douche, use tampons, and have sexual intercourse for 1 week after the procedure.

16. What information will the nurse include when providing discharge teaching to a client who had a local cervical ablation? Select all that apply. A. Do not use tampons or douche. B. Take tub baths instead of showers. C. Refrain from sexual activity. D. Avoid lifting heavy objects. E. Expect to run a fever for the first week to 10 days. F. Report heavy bleeding or foul-smelling drainage.

A. Do not use tampons or douche. C. Refrain from sexual activity. D. Avoid lifting heavy objects. F. Report heavy bleeding or foul-smelling drainage.

Which reproductive physiologic changes will the nurse expect in an older female client? Select all that apply. A. Drying, smoothing, and thinning of the vaginal walls B. Increased size of the uterus C. Loss of tone and elasticity of the pelvic ligaments and connective tissue D. Hypertrophy of the endometrium E. Increased flabbiness and fibrosis of the breasts F. Decreased size of the labia majora and clitoris

A. Drying, smoothing, and thinning of the vaginal walls C. Loss of tone and elasticity of the pelvic ligaments and connective tissue E. Increased flabbiness and fibrosis of the breasts F. Decreased size of the labia majora and clitoris

4. For which complications will the nurse monitor in a client after hysteroscopic surgery for uterine leiomyomas? Select all that apply. A. Embolism B. Fluid overload C. Pneumonia D. Hemorrhage E. Persistent increased menstrual bleeding F. Cardiac dysrhythmias such as atrial fibrillation

A. Embolism B. Fluid overload D. Hemorrhage E. Persistent increased menstrual bleeding

For clients with which types of hepatitis will the nurse teach about prevention of infection spread through the oral-fecal contamination route? Select all that apply. A. Hepatitis A (HAV) B. Hepatitis B (HBV) C. Hepatitis C (HCV) D. Hepatitis D (HDV) E. Hepatitis E (HEV) F. Toxic hepatitis

A. Hepatitis A (HAV) E. Hepatitis E (HEV)

7. Which activity restrictions will the nurse teach a client to follow after vaginal hysterectomy? Select all that apply. A. Limit stair climbing to less than five times a day. B. Do not lift anything heavier than 5 to 10 lb. C. Gradually increase walking as exercise. D. Do not cross your legs at the knees. E. Avoid sitting for extended periods of time. F. Do not drive until the surgeon tells you it's permitted.

A. Limit stair climbing to less than five times a day. B. Do not lift anything heavier than 5 to 10 lb. C. Gradually increase walking as exercise. D. Do not cross your legs at the knees. E. Avoid sitting for extended periods of time. F. Do not drive until the surgeon tells you it's permitted.

Which actions are appropriate for the nurse to perform to prevent harm in a client with cirrhosis and ascites who has just undergone an esophagogastroduodenoscopy (EGD)? Select all that apply. A. Measuring oxygen saturation B. Checking for leakage from the site C. Assessing for return of the gag reflex D. Monitoring heart rate and blood pressure E. Auscultating bowel sounds in all four quadrants F. Comparing weight with that obtained before the procedure

A. Measuring oxygen saturation C. Assessing for return of the gag reflex D. Monitoring heart rate and blood pressure

For which conditions are a pelvic examination and Pap test indicated to assess for? Select all that apply. A. Menstrual irregularities B. Rape trauma or other pelvic injury C. Unexplained abdominal or vaginal pain D. Vaginal discharge, itching, sores, or infection E. Physical changes in the vagina, cervix, and uterus F. Pregnancy and infertility

A. Menstrual irregularities B. Rape trauma or other pelvic injury C. Unexplained abdominal or vaginal pain D. Vaginal discharge, itching, sores, or infection E. Physical changes in the vagina, cervix, and uterus F. Pregnancy and infertility

17. When teaching a client about the loop electrosurgical excision procedure (LEEP) for cervical cancer, what does the nurse instruct the client is expected after the surgery? A. Spotting with slight pain B. Menses-like vaginal bleeding C. Cramps lasting during the first 48 hours D. Watery discharge during the first week

A. Spotting with slight pain

14. Which vital sign change in a client with hypovolemic shock indicates to the nurse that the therapy is effective? A. Urine output increases from 5 mL/hr to 25 mL/hr. B. Pulse pressure decreases from 35 mm Hg to 28 mm Hg. C. Respiratory rate increases from 22 breaths/min to 26 breaths/min. D. Core body temperature increases from 98.2°F (36.8°C) to 98.8°F (37.1°C).

A. Urine output increases from 5 mL/hr to 25 mL/hr. During shock, the kidneys and baroreceptors sense an ongoing decrease in MAP and trigger the release of renin, antidiuretic hormone (ADH), aldosterone, epinephrine, and norepinephrine to start kidney compensation, which is very sensitive to changes in fluid volume from normal. Renin, secreted by the kidney, causes decreased urine output. ADH increases water reabsorption in the kidney, further reducing urine output. These actions compensate for shock by attempting to prevent further fluid loss. This response is so sensitive that urine output is a very good indicator of fluid resuscitation adequacy. If the therapy was not effective, urine output would not increase.

1. Post transurethral resection of the prostate, a client has a three-way catheter with a continuous bladder irrigation. Over the last 12 hours, there has been 1400 mL of irrigation solution infused and 2000 mL measured in output from the drainage bag. What is the recording of the urinary output for the 12-hour period? (Record your answer using a whole number.) ____ mL

ANS: 600 mL 2000 mL from the drainage bag (including both the irrigation fluid and urine) minus the 1400 mL of irrigation fluid equals 600 mL of urine: 2000 mL 1400 mL = 600 mL.

1. A client who had a hysterectomy has a 200-mg dose of ciprofloxacin (Cipro) ordered to infuse in 30 minutes. At what rate should the nurse infuse the medication if the pharmacy provides 200 mg in a 100-mL bag of normal saline? (Record your answer using a whole number.) ___ mL/hr

ANS: 200 mL/hr 100 mL 2 = 200 mL/hr.

1. A client in the intensive care unit with acute kidney injury (AKI) must maintain a mean arterial pressure (MAP) of 65 mm Hg to promote kidney perfusion. What is the client's MAP if the blood pressure is 98/50 mm Hg? (Record your answer using a whole number.) _____ mm Hg

ANS: 66 mm Hg

12. A client is taking furosemide (Lasix) 40 mg/day for management of chronic kidney disease (CKD). To detect the positive effect of the medication, what action of the nurse is best? a. Obtain daily weights of the client. b. Auscultate heart and breath sounds. c. Palpate the client's abdomen. d. Assess the client's diet history.

ANS: A Furosemide (Lasix) is a loop diuretic that helps reduce fluid overload and hypertension in clients with early stages of CKD. One kilogram of weight equals about 1 liter of fluid retained in the client, so daily weights are necessary to monitor the response of the client to the medication. Heart and breath sounds should be assessed if there is fluid retention, as in heart failure. Palpation of the client's abdomen is not necessary, but the nurse should check for edema. The diet history of the client would be helpful to assess electrolyte replacement since potassium is lost with this diuretic, but this does not assess the effect of the medication.

A nurse assesses clients at a community health fair. Which client is at greatest risk for the development of hepatitis B? a. A 20-year-old college student who has had several sexual partners b. A 46-year-old woman who takes acetaminophen daily for headaches c. A 63-year-old businessman who travels frequently across the country d. An 82-year-old woman who recently ate raw shellfish for dinner

ANS: A Hepatitis B can be spread through sexual contact, needle sharing, needle sticks, blood transfusions, hemodialysis, acupuncture, and the maternal-fetal route. A person with multiple sexual partners has more opportunities to contract the infection. Hepatitis B is not transmitted through medications, casual contact with other travelers, or raw shellfish. Although an overdose of acetaminophen can cause liver cirrhosis, this is not associated with hepatitis B. Hepatitis E is found most frequently in international travelers. Hepatitis A is spread through ingestion of contaminated shellfish.

8. A client in the intensive care unit is started on continuous venovenous hemofiltration (CVVH). Which finding is the cause of immediate action by the nurse? a. Blood pressure of 76/58 mm Hg b. Sodium level of 138 mEq/L c. Potassium level of 5.5 mEq/L d. Pulse rate of 90 beats/min

ANS: A Hypotension can be a problem with CVVH if replacement fluid does not provide enough volume to maintain blood pressure. The specially trained nurse needs to monitor for ongoing fluid and electrolyte replacement. The sodium level is normal and the potassium level is slightly elevated, which could be normal findings for someone with acute kidney injury. A pulse rate of 90 beats/min is normal.

12. A client has scheduled brachytherapy sessions and states that she feels as though she is not safe around her family. What is the best response by the nurse? a. You are only reactive when the radioactive implant is in place. b. To be totally safe, it is a good idea to sleep in a separate room. c. It is best to stay a safe distance from friends or family between treatments. d. You should use a separate bathroom from the rest of the family.

ANS: A In brachytherapy, the surgeon inserts an applicator into the uterus. After placement is verified, the radioactive isotope is placed in the applicator for several minutes for a single treatment. There are no restrictions for the woman to stay away from her family or the public between treatments.

A nurse cares for a client who has obstructive jaundice. The client asks, "Why is my skin so itchy?" How should the nurse respond? a. "Bile salts accumulate in the skin and cause the itching." b. "Toxins released from an inflamed gallbladder lead to itching." c. "Itching is caused by the release of calcium into the skin." d. "Itching is caused by a hypersensitivity reaction."

ANS: A In obstructive jaundice, the normal flow of bile into the duodenum is blocked, allowing excess bile salts to accumulate on the skin. This leads to itching, or pruritus. The other statements are not accurate.

A nurse assesses a client who is recovering from a paracentesis 1 hour ago. Which assessment finding requires action by the nurse? a. Urine output via indwelling urinary catheter is 20 mL/hr b. Blood pressure increases from 110/58 to 120/62 mm Hg c. Respiratory rate decreases from 18 to 14 breaths/min d. A decrease in the client's weight by 6 kg

ANS: A Rapid removal of ascetic fluid causes decreased abdominal pressure, which can contribute to hypovolemia. This can be manifested by a decrease in urine output to below 30 mL/hr. A slight increase in systolic blood pressure is insignificant. A decrease in respiratory rate indicates that breathing has been made easier by the procedure. The nurse would expect the client's weight to drop as fluid is removed. Six kilograms is less than 3 pounds and is expected.

2. The nurse is educating a client on the prevention of toxic shock syndrome (TSS). Which statement by the client indicates a lack of understanding? a. I need to change my tampon every 8 hours during the day. b. At night, I should use a feminine pad rather than a tampon. c. If I dont use tampons, I should not get TSS. d. It is best if I wash my hands before inserting the tampon.

ANS: A Tampons need to be changed every 3 to 6 hours to avoid infection by such organisms as Staphylococcus aureus. All of the other responses are correct: use of feminine pads at night, not using tampons at all, and washing hands before tampon insertion are all strategies to prevent TSS. DIF: Applying/Application REF: 1485

A nurse cares for a client with end-stage pancreatic cancer. The client asks, "Why is this happening to me?" How should the nurse respond? a. "I don't know. I wish I had an answer for you, but I don't." b. "It's important to keep a positive attitude for your family right now." c. "Scientists have not determined why cancer develops in certain people." d. "I think that this is a trial so you can become a better person because of it."

ANS: A The client is not asking the nurse to actually explain why the cancer has occurred. The client may be expressing his or her feelings of confusion, frustration, distress, and grief related to this diagnosis. Reminding the client to keep a positive attitude for his or her family does not address the client's emotions or current concerns. The nurse should validate that there is no easy or straightforward answer as to why the client has cancer. Telling a client that cancer is a trial is untrue and may diminish the client-nurse relationship.

A nurse cares for a client who is recovering from laparoscopic cholecystectomy surgery. The client reports pain in the shoulder blades. How should the nurse respond? a. "Ambulating in the hallway twice a day will help." b. "I will apply a cold compress to the painful area on your back." c. "Drinking a warm beverage can relieve this referred pain." d. "You should cough and deep breathe every hour."

ANS: A The client who has undergone a laparoscopic cholecystectomy may report free air pain due to retention of carbon dioxide in the abdomen. The nurse assists the client with early ambulation to promote absorption of the carbon dioxide. Cold compresses and drinking a warm beverage would not be helpful. Coughing and deep breathing are important postoperative activities, but they are not related to discomfort from carbon dioxide.

A nurse cares for a client with hepatopulmonary syndrome who is experiencing dyspnea with oxygen saturations at 92%. The client states, "I do not want to wear the oxygen because it causes my nose to bleed. Get out of my room and leave me alone!" Which action should the nurse take? a. Instruct the client to sit in as upright a position as possible. b. Add humidity to the oxygen and encourage the client to wear it. c. Document the client's refusal, and call the health care provider. d. Contact the provider to request an extra dose of the client's diuretic.

ANS: A The client with hepatopulmonary syndrome is often dyspneic. Because the oxygen saturation is not significantly low, the nurse should first allow the client to sit upright to see if that helps. If the client remains dyspneic, or if the oxygen saturation drops further, the nurse should investigate adding humidity to the oxygen and seeing whether the client will tolerate that. The other two options may be beneficial, but they are not the best choices. If the client is comfortable, his or her agitation will decrease; this will improve respiratory status.

A nurse cares for a client who is prescribed lactulose (Heptalac). The client states, "I do not want to take this medication because it causes diarrhea." How should the nurse respond? a. "Diarrhea is expected; that's how your body gets rid of ammonia." b. "You may take Kaopectate liquid daily for loose stools." c. "Do not take any more of the medication until your stools firm up." d. "We will need to send a stool specimen to the laboratory."

ANS: A The purpose of administering lactulose to this client is to help ammonia leave the circulatory system through the colon. Lactulose draws water into the bowel with its high osmotic gradient, thereby producing a laxative effect and subsequently evacuating ammonia from the bowel. The client must understand that this is an expected and therapeutic effect for him or her to remain compliant. The nurse should not suggest administering anything that would decrease the excretion of ammonia or holding the medication. There is no need to send a stool specimen to the laboratory because diarrhea is the therapeutic response to this medication.

18. A client with chronic kidney disease (CKD) is experiencing nausea, vomiting, visual changes, and anorexia. Which action by the nurse is best? a. Check the client's digoxin (Lanoxin) level. b. Administer an anti-nausea medication. c. Ask if the client is able to eat crackers. d. Get a referral to a gastrointestinal provider.

ANS: A These signs and symptoms are indications of digoxin (Lanoxin) toxicity. The nurse should check the level of this medication. Administering antiemetics, asking if the client can eat, and obtaining a referral to a specialist all address the client's symptoms but do not lead to the cause of the symptoms.

2. A marathon runner comes into the clinic and states "I have not urinated very much in the last few days." The nurse notes a heart rate of 110 beats/min and a blood pressure of 86/58 mm Hg. Which action by the nurse is the priority? a. Give the client a bottle of water immediately. b. Start an intravenous line for fluids. c. Teach the client to drink 2 to 3 liters of water daily. d. Perform an electrocardiogram.

ANS: A This athlete is mildly dehydrated as evidenced by the higher heart rate and lower blood pressure. The nurse can start hydrating the client with a bottle of water first, followed by teaching the client to drink 2 to 3 liters of water each day. An intravenous line may be ordered later, after the client's degree of dehydration is assessed. An electrocardiogram is not necessary at this time.

A nurse teaches a client with hepatitis C who is prescribed ribavirin (Copegus). Which statement should the nurse include in this client's discharge education? a. "Use a pill organizer to ensure you take this medication as prescribed." b. "Transient muscle aching is a common side effect of this medication." c. "Follow up with your provider in 1 week to test your blood for toxicity." d. "Take your radial pulse for 1 minute prior to taking this medication."

ANS: A Treatment of hepatitis C with ribavirin takes up to 48 weeks, making compliance a serious issue. The nurse should work with the client on a strategy to remain compliant for this length of time. Muscle aching is not a common side effect. The client will be on this medication for many weeks and does not need a blood toxicity examination. There is no need for the client to assess his or her radial pulse prior to taking the medication.

2. The nurse is giving discharge instructions to a client who had a total abdominal hysterectomy. Which statements by the client indicate a need for further teaching? (Select all that apply.) a. I should not have any problems driving to see my mother, who lives 3 hours away. b. Now that I have time off from work, I can return to my exercise routine next week. c. My granddaughter weighs 23 pounds, so I need to refrain from picking her up. d. I will have to limit the times that I climb our stairs at home to morning and night. e. For 1 month, I will need to refrain from sexual intercourse.

ANS: A, B Driving and sitting for extended periods of time should be avoided until the surgeon gives permission. For 2 to 6 weeks, exercise participation should also be avoided. All of the other responses demonstrate adequate knowledge for discharge. The client should not lift anything heavier than 10 pounds, should limit stair climbing, and should refrain from sexual intercourse.

1. The nurse is caring for five clients on the medical-surgical unit. Which clients would the nurse consider to be at risk for post-renal acute kidney injury (AKI)? (Select all that apply.) a. Man with prostate cancer b. Woman with blood clots in the urinary tract c. Client with ureterolithiasis d. Firefighter with severe burns e. Young woman with lupus

ANS: A, B, C Urine flow obstruction, such as prostate cancer, blood clots in the urinary tract, and kidney stones (ureterolithiasis), causes post-renal AKI. Severe burns would be a pre-renal cause. Lupus would be an intrarenal cause for AKI.

3. The nurse is taking the history of a 24-year-old client diagnosed with cervical cancer. What possible risk factors would the nurse assess? (Select all that apply.) a. Smoking b. Multiple sexual partners c. Poor diet d. Nulliparity e. Younger than 18 at first intercourse

ANS: A, B, C, E Smoking, multiple sexual partners, poor diet, and age less than 18 for first intercourse are all risk factors for cervical cancer. Nulliparity is a risk factor for endometrial cancer.

6. A client is undergoing hemodialysis. The client's blood pressure at the beginning of the procedure was 136/88 mm Hg, and now it is 110/54 mm Hg. What actions should the nurse perform to maintain blood pressure? (Select all that apply.) a. Adjust the rate of extracorporeal blood flow. b. Place the client in the Trendelenburg position. c. Stop the hemodialysis treatment. d. Administer a 250-mL bolus of normal saline. e. Contact the health care provider for orders.

ANS: A, B, D Hypotension occurs often during hemodialysis treatments as a result of vasodilation from the warmed dialysate. Modest decreases in blood pressure, as is the case with this client, can be maintained with rate adjustment, Trendelenburg positioning, and a fluid bolus. If the blood pressure drops considerably after two boluses and cooling dialysate, the hemodialysis can be stopped and the health care provider contacted.

7. A client is unsure of the decision to undergo peritoneal dialysis (PD) and wishes to discuss the advantages of this treatment with the nurse. Which statements by the nurse are accurate regarding PD? (Select all that apply.) a. "You will not need vascular access to perform PD." b. "There is less restriction of protein and fluids." c. "You will have no risk for infection with PD." d. "You have flexible scheduling for the exchanges." e. "It takes less time than hemodialysis treatments."

ANS: A, B, D PD is based on exchanges of waste, fluid, and electrolytes in the peritoneal cavity. There is no need for vascular access. Protein is lost in the exchange, which allows for more protein and fluid in the diet. There is flexibility in the time for exchanges, but the treatment takes a longer period of time compared to hemodialysis. There still is risk for infection with PD, especially peritonitis.

A nurse assesses a male client who has symptoms of cirrhosis. Which questions should the nurse ask to identify potential factors contributing to this laboratory result? (Select all that apply.) a. "How frequently do you drink alcohol?" b. "Have you ever had sex with a man?" c. "Do you have a family history of cancer?" d. "Have you ever worked as a plumber?" e. "Were you previously incarcerated?"

ANS: A, B, E When assessing a client with suspected cirrhosis, the nurse should ask about alcohol consumption, including amount and frequency; sexual history and orientation (specifically men having sex with men); illicit drug use; history of tattoos; and history of military service, incarceration, or work as a firefighter, police officer, or health care provider. A family history of cancer and work as a plumber do not put the client at risk for cirrhosis.

6. A postmenopausal client is experiencing low back and pelvic pain, fatigue, and bloody vaginal discharge. What laboratory tests would the nurse expect to see ordered for this client if endometrial cancer is suspected? (Select all that apply.) a. Cancer antigen-125 (CA-125) b. White blood cell (WBC) count c. Hemoglobin and hematocrit (H&H) d. International normalized ratio (INR) e. Prothrombin time (PT)

ANS: A, C Serum tumor markers such as CA-125 assess for metastasis, especially if elevated. H&H would evaluate the possibility of anemia, a common finding with postmenopausal bleeding with endometrial cancer. WBC count is not indicated since there are no signs of infection. The INR and PT are coagulation tests to measure the time it takes for a fibrin clot to form. They are used to evaluate the extrinsic pathway of coagulation in clients receiving oral warfarin.

A nurse plans care for a client who has hepatopulmonary syndrome. Which interventions should the nurse include in this client's plan of care? (Select all that apply.) a. Oxygen therapy b. Prone position c. Feet elevated on pillows d. Daily weights e. Physical therapy

ANS: A, C, D Care for a client who has hepatopulmonary syndrome should include oxygen therapy, the head of bed elevated at least 30 degrees or as high as the client wants to improve breathing, elevated feet to decrease dependent edema, and daily weights. There is no need to place the client in a prone position, on the client's stomach. Although physical therapy may be helpful to a client who has been hospitalized for several days, physical therapy is not an intervention specifically for hepatopulmonary syndrome.

An infection control nurse develops a plan to decrease the number of health care professionals who contract viral hepatitis at work. Which ideas should the nurse include in this plan? (Select all that apply.) a. Policies related to consistent use of Standard Precautions b. Hepatitis vaccination mandate for workers in high-risk areas c. Implementation of a needleless system for intravenous therapy d. Number of sharps used in client care reduced where possible e. Postexposure prophylaxis provided in a timely manner

ANS: A, C, D, E Nurses should always use Standard Precautions for client care, and policies should reflect this. Needleless systems and reduction of sharps can help prevent hepatitis. Postexposure prophylaxis should be provided immediately. All health care workers should receive the hepatitis vaccinations that are available.

A nurse plans care for a client who has acute pancreatitis and is prescribed nothing by mouth (NPO). With which health care team members should the nurse collaborate to provide appropriate nutrition to this client? (Select all that apply.) a. Registered dietitian b. Nursing assistant c. Clinical pharmacist d. Certified herbalist e. Health care provider

ANS: A, C, E Clients who are prescribed NPO while experiencing an acute pancreatitis episode may need enteral or parenteral nutrition. The nurse should collaborate with the registered dietitian, clinical pharmacist, and health care provider to plan and implement the more appropriate nutritional interventions. The nursing assistant and certified herbalist would not assist with this clinical decision.

3. A client is hospitalized in the oliguric phase of acute kidney injury (AKI) and is receiving tube feedings. The nurse is teaching the client's spouse about the kidney-specific formulation for the enteral solution compared to standard formulas. What components should be discussed in the teaching plan? (Select all that apply.) a. Lower sodium b. Higher calcium c. Lower potassium d. Higher phosphorus e. Higher calories

ANS: A, C, E Many clients with AKI are too ill to meet caloric goals and require tube feedings with kidney-specific formulas that are lower in sodium, potassium, and phosphorus, and higher in calories than are standard formulas.

1. A 28-year-old client is diagnosed with endometriosis and is experiencing severe symptoms. Which actions by the nurse are the most appropriate at this time? (Select all that apply.) a. Reduce the pain by low-level heat. b. Discuss the high risk of infertility with this diagnosis. c. Relieve anxiety by relaxation techniques and education. d. Discuss in detail the side effects of laparoscopic surgery. e. Suggest resources such as the Endometriosis Association.

ANS: A, C, E With endometriosis, pain is the predominant symptom, with anxiety occurring because of the diagnosis. Interventions should be directed to pain and anxiety relief, such as low-level heat, relaxation techniques, and education about the pathophysiology and possible treatment of endometriosis. The nurse could suggest resources to give more information about the diagnosis. Discussion of the possibility of infertility and side effects of laparoscopic surgery is premature and may increase the anxiety.

A nurse assesses a client with cholelithiasis. Which assessment findings should the nurse identify as contributors to this client's condition? (Select all that apply.) a. Body mass index of 46 b. Vegetarian diet c. Drinking 4 ounces of red wine nightly d. Pregnant with twins e. History of metabolic syndrome f. Glycosylated hemoglobin level of 15%

ANS: A, D, F Obesity, pregnancy, and diabetes are all risk factors for the development of cholelithiasis. A diet low in saturated fats and moderate alcohol intake may decrease the risk. Although metabolic syndrome is a precursor to diabetes, it is not a risk factor for cholelithiasis. The client should be informed of the connection.

1. A nurse obtains a client's health history at a community health clinic. Which statement alerts the nurse to provide health teaching to this client? a. "I drink two glasses of red wine each week." b. "I take a lot of Tylenol for my arthritis pain." c. "I have a cousin who died of liver cancer." d. "I got a hepatitis vaccine before traveling."

ANS: B Acetaminophen (Tylenol) can cause liver damage if taken in large amounts. Clients should be taught not to exceed 4000 mg/day of acetaminophen. The nurse should teach the client about this limitation and should explore other drug options with the client to manage his or her arthritis pain. Two glasses of wine each week, a cousin with liver cancer, and the hepatitis vaccine do not place the client at risk for a liver disorder, and therefore do not require any health teaching.

After teaching a client who is recovering from laparoscopic cholecystectomy surgery, the nurse assesses the client's understanding. Which statement made by the client indicates a correct understanding of the teaching? a. "Drinking at least 2 liters of water each day is suggested." b. "I will decrease the amount of fatty foods in my diet." c. "Drinking fluids with my meals will increase bloating." d. "I will avoid concentrated sweets and simple carbohydrates."

ANS: B After cholecystectomy, clients need a nutritious diet without a lot of excess fat; otherwise a special diet is not recommended for most clients. Good fluid intake is healthy for all people but is not related to the surgery. Drinking fluids between meals helps with dumping syndrome, which is not seen with this procedure. Restriction of sweets is not required.

A nurse cares for a client with hepatitis C. The client's brother states, "I do not want to contract this infection, so I will not go into his hospital room." How should the nurse respond? a. "If you wear a gown and gloves, you will not get this virus." b. "Viral hepatitis is not spread through casual contact." c. "This virus is only transmitted through a fecal specimen." d. "I can give you an update on your brother's status from here."

ANS: B Although family members may be afraid that they will contract hepatitis C, the nurse should educate the client's family about how the virus is spread. Viral hepatitis, or hepatitis C, is spread via blood-to-blood transmission and is associated with illicit IV drug needle sharing, blood and organ transplantation, accidental needle sticks, unsanitary tattoo equipment, and sharing of intranasal cocaine paraphernalia. Wearing a gown and gloves will not decrease the transmission of this virus. Hepatitis C is not spread through casual contact or a fecal specimen. The nurse would be violating privacy laws by sharing the client's status with the brother.

14. A 20-year-old client is interested in protection from the human papilloma virus (HPV) since she may become sexually active. Which response from the nurse is the most accurate? a. You are too old to receive an HPV vaccine. b. Either Gardasil or Cervarix can provide protection. c. You will need to have three injections over a span of 1 year. d. The most common side effect of the vaccine is itching at the injection site.

ANS: B Current HPV vaccines are Gardasil and Cervarix, which should be given before the first sexual contact to protect against the highest risk HPV types associated with cervical cancer. The client is not too old since it is recommended that young women up to 26 years should receive an HPV vaccine. The entire series consists of three injections over 6 months, not 1 year. Local pain and redness surrounding the injection site are very common, but this does not include itching.

A telehealth nurse speaks with a client who is recovering from a liver transplant 2 weeks ago. The client states, "I am experiencing right flank pain and have a temperature of 101° F." How should the nurse respond? a. "The anti-rejection drugs you are taking make you susceptible to infection." b. "You should go to the hospital immediately to have your new liver checked out." c. "You should take an additional dose of cyclosporine today." d. "Take acetaminophen (Tylenol) every 4 hours until you feel better."

ANS: B Fever, right quadrant or flank pain, and jaundice are signs of liver transplant rejection; the client should be admitted to the hospital as soon as possible for intervention. Anti-rejection drugs do make a client more susceptible to infection, but this client has signs of rejection, not infection. The nurse should not advise the client to take an additional dose of cyclosporine or acetaminophen as these medications will not treat the acute rejection.

9. The nurse is doing preoperative teaching for a client who is scheduled for removal of cervical polyps in the office. Which statement by the client indicates a correct understanding of the procedure? a. I hope that I do not have cancer of the cervix. b. There should be little or no discomfort during the procedure. c. There may be a lot of bleeding after the polyp is removed. d. This may prevent me from having any more children.

ANS: B Polyp removal is a simple office procedure with the client feeling no pain. The other responses are incorrect. Cervical polyps are the most common benign growth of the cervix. Cautery is used to stop any bleeding, and there is no evidence that cervical polyps have a relationship to childbearing.

1. The nurse is assessing a client with a diagnosis of pre-renal acute kidney injury (AKI). Which condition would the nurse expect to find in the client's recent history? a. Pyelonephritis b. Myocardial infarction c. Bladder cancer d. Kidney stones

ANS: B Pre-renal causes of AKI are related to a decrease in perfusion, such as with a myocardial infarction. Pyelonephritis is an intrinsic or intrarenal cause of AKI related to kidney damage. Bladder cancer and kidney stones are post-renal causes of AKI related to urine flow obstruction.

A nurse collaborates with an unlicensed assistive personnel (UAP) to provide care for a client who is in the healing phase of acute pancreatitis. Which statements focused on nutritional requirements should the nurse include when delegating care for this client? (Select all that apply.) a. "Do not allow the client to eat between meals." b. "Make sure the client receives a protein shake." c. "Do not allow caffeine-containing beverages." d. "Make sure the foods are bland with little spice." e. "Do not allow high-carbohydrate food items."

ANS: B, C, D During the healing phase of pancreatitis, the client should be provided small, frequent, moderate- to high-carbohydrate, high-protein, low-fat meals. Protein shakes can be provided to supplement the diet. Foods and beverages should not contain caffeine and should be bland.

4. A client is scheduled to start external beam radiation therapy (EBRT) for her endometrial cancer. Which teaching by the nurse is accurate? (Select all that apply.) a. You will need to be hospitalized during this therapy. b. Your skin needs to be inspected daily for any breakdown. c. It is not wise to stay out in the sun for long periods of time. d. The perineal area may become damaged with the radiation. e. The technician applies new site markings before each treatment.

ANS: B, C, D EBRT is usually performed in ambulatory care and does not require hospitalization. The client needs to know to evaluate the skin, especially in the perineal area, for any breakdown, and avoid sunbathing. The technician does not apply new site markings, so the client needs to avoid washing off the markings that indicate the treatment site.

A nurse cares for a client with pancreatic cancer who is prescribed implanted radioactive iodine seeds. Which actions should the nurse take when caring for this client? (Select all that apply.) a. Dispose of dirty linen in a red "biohazard" bag. b. Place the client in a private room. c. Wear a lead apron when providing client care. d. Bundle care to minimize exposure to the client. e. Initiate Transmission-Based Precautions.

ANS: B, C, D The client should be placed in a private room and dirty linens kept in the client's room until the radiation source is removed. The nurse should wear a lead apron while providing care, ensuring that the apron always faces the client. The nurse should also bundle care to minimize exposure to the client. Transmission-Based Precautions will not protect the nurse from the implanted radioactive iodine seeds.

5. A nurse is giving discharge instructions to a client recently diagnosed with chronic kidney disease (CKD). Which statements made by the client indicate a correct understanding of the teaching? (Select all that apply.) a. "I can continue to take antacids to relieve heartburn." b. "I need to ask for an antibiotic when scheduling a dental appointment." c. "I'll need to check my blood sugar often to prevent hypoglycemia." d. "The dose of my pain medication may have to be adjusted." e. "I should watch for bleeding when taking my anticoagulants."

ANS: B, C, D, E In discharge teaching, the nurse must emphasize that the client needs to have an antibiotic prophylactically before dental procedures to prevent infection. There may be a need for dose reduction in medications if the kidney is not excreting them properly (antacids with magnesium, antibiotics, antidiabetic drugs, insulin, opioids, and anticoagulants).

A nurse assesses a client who is recovering from a Whipple procedure. Which clinical manifestations alert the nurse to a complication from this procedure? (Select all that apply.) a. Clay-colored stools b. Substernal chest pain c. Shortness of breath d. Lack of bowel sounds or flatus e. Urine output of 20 mL/6 hr

ANS: B, C, D, E Myocardial infarction (chest pain), pulmonary embolism (shortness of breath), adynamic ileus (lack of bowel sounds or flatus), and renal failure (urine output of 20 mL/6 hr) are just some of the complications for which the nurse must assess the client after the Whipple procedure. Clay-colored stools are associated with cholecystitis and are not a complication of a Whipple procedure.

4. The nurse is teaching a client with diabetes mellitus how to prevent or delay chronic kidney disease (CKD). Which client statements indicate a lack of understanding of the teaching? (Select all that apply.) a. "I need to decrease sodium, cholesterol, and protein in my diet." b. "My weight should be maintained at a body mass index of 30." c. "Smoking should be stopped as soon as I possibly can." d. "I can continue to take an aspirin every 4 to 8 hours for my pain." e. "I really only need to drink a couple of glasses of water each day."

ANS: B, D, E Weight should be maintained at a body mass index (BMI) of 22 to 25. A BMI of 30 indicates obesity. The use of nonsteroidal anti-inflammatory drugs such as aspirin should be limited to the lowest time at the lowest dose due to interference with kidney blood flow. The client should drink at least 2 liters of water daily. Diet adjustments should be made by restricting sodium, cholesterol, and protein. Smoking causes constriction of blood vessels and decreases kidney perfusion, so the client should stop smoking.

A nurse teaches a client who is recovering from acute pancreatitis. Which statements should the nurse include in this client's teaching? (Select all that apply.) a. "Take a 20-minute walk at least 5 days each week." b. "Attend local Alcoholics Anonymous (AA) meetings weekly." c. "Choose whole grains rather than foods with simple sugars." d. "Use cooking spray when you cook rather than margarine or butter." e. "Stay away from milk and dairy products that contain lactose." f. "We can talk to your doctor about a prescription for nicotine patches."

ANS: B, D, F The client should be advised to stay sober, and AA is a great resource. The client requires a low-fat diet, and cooking spray is low in fat compared with butter or margarine. If the client smokes, he or she must stop because nicotine can precipitate an exacerbation. A nicotine patch may help the client quit smoking. The client must rest until his or her strength returns. The client requires high carbohydrates and calories for healing; complex carbohydrates are not preferred over simple ones. Dairy products do not cause a problem.

A nurse prepares to assess the emotional state of a client with end-stage pancreatic cancer. Which action should the nurse take first? a. Bring the client to a quiet room for privacy. b. Pull up a chair and sit next to the client's bed. c. Determine whether the client feels like talking about his or her feelings. d. Review the health care provider's notes about the prognosis for the client.

ANS: C Before conducting an assessment about the client's feelings, the nurse should determine whether he or she is willing and able to talk about them. If the client is open to the conversation and his or her room is not appropriate, an alternative meeting space may be located. The nurse should be present for the client during this time, and pulling up a chair and sitting with the client indicates that presence. Because the nurse is assessing the client's response to a terminal diagnosis, it is not necessary to have detailed information about the projected prognosis; the nurse knows that the client is facing an end-of-life illness.

A nurse cares for a client who has chronic cirrhosis from substance abuse. The client states, "All of my family hates me." How should the nurse respond? a. "You should make peace with your family." b. "This is not unusual. My family hates me too." c. "I will help you identify a support system." d. "You must attend Alcoholics Anonymous."

ANS: C Clients who have chronic cirrhosis may have alienated relatives over the years because of substance abuse. The nurse should assist the client to identify a friend, neighbor, or person in his or her recovery group for support. The nurse should not minimize the client's concerns by brushing off the client's comment. Attending AA may be appropriate, but this response doesn't address the client's concern. Making peace with the client's family may not be possible. This statement is not client-centered.

19. The nurse is taking the vital signs of a client after hemodialysis. Blood pressure is 110/58 mm Hg, pulse 66 beats/min, and temperature is 99.8° F (37.6° C). What is the most appropriate action by the nurse? a. Administer fluid to increase blood pressure. b. Check the white blood cell count. c. Monitor the client's temperature. d. Connect the client to an electrocardiographic (ECG) monitor.

ANS: C During hemodialysis, the dialysate is warmed to increase diffusion and prevent hypothermia. The client's temperature could reflect the temperature of the dialysate. There is no indication to check the white blood cell count or connect the client to an ECG monitor. The other vital signs are within normal limits.

4. A 55-year-old post-menopausal woman is assessed by the nurse with a history of dyspareunia, backache, pelvis pressure, urinary tract infections, and a frequent urinary urgency. Which condition does the nurse suspect? a. Ovarian cyst b. Rectocele c. Cystocele d. Fibroid

ANS: C Dyspareunia, backache, pelvis pressure, urinary tract infections, and urinary urgency are all symptoms of a cystocelea protrusion of the bladder through the vaginal wall. Ovarian cysts are rare after menopause. A rectocele is associated with constipation, hemorrhoids, and fecal impaction. Fibroids are associated with heavy bleeding.

A nurse assesses a client who has cholecystitis. Which clinical manifestation indicates that the condition is chronic rather than acute? a. Temperature of 100.1° F (37.8° C) b. Positive Murphy's sign c. Light-colored stools d. Upper abdominal pain after eating

ANS: C Jaundice, clay-colored stools, and dark urine are more commonly seen with chronic cholecystitis. The other symptoms are seen equally with both chronic and acute cholecystitis.

10. The charge nurse of the medical-surgical unit is making staff assignments. Which staff member should be assigned to a client with chronic kidney disease who is exhibiting a low-grade fever and a pericardial friction rub? a. Registered nurse who just floated from the surgical unit b. Registered nurse who just floated from the dialysis unit c. Registered nurse who was assigned the same client yesterday d. Licensed practical nurse with 5 years' experience on this floor

ANS: C The client is exhibiting symptoms of pericarditis, which can occur with chronic kidney disease. Continuity of care is important to assess subtle differences in clients. Therefore, the registered nurse (RN) who was assigned to this client previously should again give care to this client. The float nurses would not be as knowledgeable about the unit and its clients. The licensed practical nurse may not have the education level of the RN to assess for pericarditis.

After teaching a client who is prescribed pancreatic enzyme replacement therapy, the nurse assesses the client's understanding. Which statement made by the client indicates a need for additional teaching? a. "The capsules can be opened and the powder sprinkled on applesauce if needed." b. "I will wipe my lips carefully after I drink the enzyme preparation." c. "The best time to take the enzymes is immediately after I have a meal or a snack." d. "I will not mix the enzyme powder with food or liquids that contain protein."

ANS: C The enzymes should be taken immediately before eating meals or snacks. If the client cannot swallow the capsules whole, they can be opened up and the powder sprinkled on applesauce, mashed fruit, or rice cereal. The client should wipe his or her lips carefully after drinking the enzyme preparation because the liquid could damage the skin. Protein items will be dissolved by the enzymes if they are mixed together.

13. A client is diagnosed with chronic kidney disease (CKD). What is an ideal goal of treatment set by the nurse in the care plan to reduce the risk of pulmonary edema? a. Maintaining oxygen saturation of 89% b. Minimal crackles and wheezes in lung sounds c. Maintaining a balanced intake and output d. Limited shortness of breath upon exertion

ANS: C With an optimal fluid balance, the client will be more able to eject blood from the left ventricle without increased pressure in the left ventricle and pulmonary vessels. Other ideal goals are oxygen saturations greater than 92%, no auscultated crackles or wheezes, and no demonstrated shortness of breath.

23. A nurse is caring for a client who is scheduled for a dose of cefazolin and vitamins at this time. Hemodialysis for this client is also scheduled in 60 minutes. Which action by the nurse is best? a. Administer cefazolin since the level of the antibiotic must be maintained. b. Hold the vitamins but administer the cefazolin. c. Hold the cefazolin but administer the vitamins. d. Hold all medications since both cefazolin and vitamins are dialyzable.

ANS: D Both the cefazolin and the vitamins should be held until after the hemodialysis is completed because they would otherwise be removed by the dialysis process.

After teaching a client who has a history of cholelithiasis, the nurse assesses the client's understanding. Which menu selection made by the client indicates the client clearly understands the dietary teaching? a. Lasagna, tossed salad with Italian dressing, and low-fat milk b. Grilled cheese sandwich, tomato soup, and coffee with cream c. Cream of potato soup, Caesar salad with chicken, and a diet cola d. Roasted chicken breast, baked potato with chives, and orange juice

ANS: D Clients with cholelithiasis should avoid foods high in fat and cholesterol, such as whole milk, butter, and fried foods. Lasagna, low-fat milk, grilled cheese, cream, and cream of potato soup all have high levels of fat. The meal with the least amount of fat is the chicken breast dinner.

11. The client is emotionally upset about the recent diagnosis of stage IV endometrial cancer. Which action by the nurse is best? a. Let the client alone for a long period of reflection time. b. Ask friends and relatives to limit their visits. c. Tell the client that an emotional response is unacceptable. d. Create an atmosphere of acceptance and discussion.

ANS: D Discussion of a clients concerns about the presence of cancer and the potential for recurrence will provide emotional support and allay fears. Coping behaviors are encouraged with the support of friends and relatives. An emotional response should be accepted.

22. A client is assessed by the nurse after a hemodialysis session. The nurse notes bleeding from the client's nose and around the intravenous catheter. What action by the nurse is the priority? a. Hold pressure over the client's nose for 10 minutes. b. Take the client's pulse, blood pressure, and temperature. c. Assess for a bruit or thrill over the arteriovenous fistula. d. Prepare protamine sulfate for administration.

ANS: D Heparin is used with hemodialysis treatments. The bleeding alerts the nurse that too much anticoagulant is in the client's system and protamine sulfate should be administered. Pressure, taking vital signs, and assessing for a bruit or thrill are not as important as medication administration.

A nurse cares for a client who is hemorrhaging from bleeding esophageal varices and has an esophagogastric tube. Which action should the nurse take first? a. Sedate the client to prevent tube dislodgement. b. Maintain balloon pressure at 15 and 20 mm Hg. c. Irrigate the gastric lumen with normal saline. d. Assess the client for airway patency.

ANS: D Maintaining airway patency is the primary nursing intervention for this client. The nurse suctions oral secretions to prevent aspiration and occlusion of the airway. The client usually is intubated and mechanically ventilated during this treatment. The client should be sedated, balloon pressure should be maintained between 15 and 20 mm Hg, and the lumen can be irrigated with saline or tap water. However, these are not a higher priority than airway patency.

A nurse assesses a client who is prescribed an infusion of vasopressin (Pitressin) for bleeding esophageal varices. Which clinical manifestation should alert the nurse to a serious adverse effect? a. Nausea and vomiting b. Frontal headache c. Vertigo and syncope d. Mid-sternal chest pain

ANS: D Mid-sternal chest pain is indicative of acute angina or myocardial infarction, which can be precipitated by vasopressin. Nausea and vomiting, headache, and vertigo and syncope are not side effects of vasopressin.

5. The nurse is caring for a postoperative client following an anterior colporrhaphy. What action can be delegated to the unlicensed assistive personnel (UAP)? a. Reviewing the hematocrit and hemoglobin results b. Teaching the client to avoid lifting her 4-year-old grandson c. Assessing the level of pain and any drainage d. Drawing a shallow hot bath for comfort measures

ANS: D The UAP is able to provide comfort through a bath. The registered nurse should review any laboratory results, complete any teaching, and assess pain and discharge.

7. A client has just had a central line catheter placed that is specific for hemodialysis. What is the most appropriate action by the nurse? a. Use the catheter for the next laboratory blood draw. b. Monitor the central venous pressure through this line. c. Access the line for the next intravenous medication. d. Place a heparin or heparin/saline dwell after hemodialysis.

ANS: D The central line should have a heparin or heparin/saline dwell after hemodialysis treatment. The central line catheter used for dialysis should not be used for blood sampling, monitoring central venous pressures, or giving drugs or fluids.

1. Which action would the nurse teach to help the client prevent vulvovaginitis? a. Wipe back to front after urination. b. Cleanse the inner labial mucosa with soap and water. c. Use feminine hygiene sprays to avoid odor. d. Wear loose cotton underwear.

ANS: D To prevent vulvovaginitis, the client should wear cotton underwear. The client should wipe front to back after urination, not back to front. The client should cleanse the inner labial mucosa with water only, and avoid using feminine hygiene sprays.

A client hospitalized for worsening kidney injury suddenly becomes restless and agitated. Assessment reveals tachycardia and crackles bilaterally at the bases of the lungs. Which is the nurse's first intervention? Place the client in high Fowler's position Administer an antianxiety agent. Place the client on mechanical ventilation. Set up for continuous renal replacement therapy.

ANS:A Restlessness, anxiety, tachycardia, dyspnea, and crackles at the bases of the lungs are early manifestations of pulmonary edema, which is a complication of kidney failure. Initial treatment of pulmonary edema consists of placing the client in high Fowler's position and administering oxygen. Mechanical ventilation and ultrafiltration may be indicated if symptoms become worse. An antianxiety agent would not be helpful. Morphine, however, has both vasoactive and sedating effects.

A client with acute kidney injury is placed on a fluid restriction. To determine whether outcomes related to fluid balance are being met, the nurse assesses for which finding? Increase in serum pH Decreased serum potassium level Decreased serum creatinine level Absence of lung crackles

Absence of lung crackles The client with chronic kidney disease is expected to achieve and maintain an acceptable fluid balance. Fluid restriction helps with this outcome. Absence of lung crackles can indicate that the client is not fluid overloaded. The other options are not related to fluid balance.

A patient who is receiving fluid resuscitation per the Parkland formula after a serious burn continues to have urine output ranging from 0.2 to 0.25 mL/kg/hour. After the health care provider checks the patient, which order does the nurse question? Administer furosemide (lasix) 40mg IVP BID Continue to monitor urine output hourly Increase IV fluid by 150 mL/hr. Serum blood draw for electrolytes stat

Administer furosemide (lasix) 40mg IVP BID

The emergency nurse is assigned to care for four clients. Which client does the nurse see first? Young adult with a swollen, painful scrotum who has a recent history of mumps infection Older adult with a history of benign prostatic hyperplasia and palpable bladder distention Middle-age adult discharged 2 days ago after a TURP with hematuria Adolescent with an erection for "10 or 11 hours" who is reporting severe pain

Adolescent with an erection for "10 or 11 hours" who is reporting severe pain The nurse first attends to the client who has had an erection for "10 or 11 hours." This client has symptoms of priapism which is considered a urologic emergency because the circulation to the penis may be compromised. With an erect penis, the client may also be unable to void.The client with a swollen, painful scrotum, the client with hematuria, and the client with a history of benign prostatic hyperplasia can be seen after the client with the ongoing erection.

A client scheduled for a hysterosalpingogram is interviewed by the nurse. What interview information is critical for the nurse to report to the primary health care provider before the procedure? Administration of a rectal suppository 4 hours ago Allergy to shellfish Menstrual period that ended 3 days ago Abortion 2 months ago

Allergy to shellfish The contrast medium used during hysterosalpingography is iodine-based, so the primary health care provider will need to know if the client is allergic to shellfish.Obstetric history, menstrual history, and recent medications are communicated to the primary health care provider but do not require any change in the procedure. Two months between an abortion and this procedure is adequate. This test is done just at the completion of menses so that it would not interrupt a pregnancy in the uterus or the fallopian tube.

Which statements about eating habits and diet indicate to the nurse that the client re covering from acute pancreatitis understand. recommendations made in collaboration. the registered dietitian nutritionist?

Although the/ do not contain fat, I w'ill avoid chocolate and caffeine.' If vomiting or diarrhea occur, I will call my primary health care provider: I am planning on joining -Alcoholics. Anonymous and giving up drinking altogether.

The patient is in the emergency room with a suspicion of cholecystitis. What initial diagnostic tool would the nurse expect to be ordered? A magnetic resonance cholangiopancreatography An ultrasonography of the right upper quadrant An endoscopic retrograde cholangiopancreatography A hepatobiliary scan

An ultrasonography of the right upper quadrant

Which signs or symptoms will the nurse assess for in a client who is suspected of having cholecystitis?

Anorexia Jaundice Steatorrhea Eructation Rebound tenderness

77. A patient in hypovolemic shock is receiving sodium nitroprusside to enhance myocardioperfusion. What is an important nursing implication for administering this drug? Assess the patient every 30 minutes for indicators of extravasation because nitroprusside can cause severe tissue ischemia Assess the patient for headache because it is an early symptom of drug excess Assess blood pressure every 15 minutes because of the risk of hypotension Monitor patient's level of alertness, because this medication causes severe confusion and seizures

Assess blood pressure every 15 minutes because of the risk of hypotension

A patient in hypovolemic shock is receiving sodium nitroprusside to enhance myocardioperfusion. What is an important nursing implication for administering this drug? Assess blood pressure every 15 minutes because of the risk of hypotension Monitor patient's level of alertness, because this medication causes severe confusion and seizures Assess the patient for headache because it is an early symptom of drug excess Assess the patient every 30 minutes for indicators of extravasation

Assess blood pressure every 15 minutes because of the risk of hypotension

A nurse cares for a patient with a burn injury who presents with drooling and difficulty swallowing. What action would the nurse take first? Assess the level of consciousness and pupillary reactions Ausculate breath sounds over the trachea and bronchi Ascertaining the time food or liquid was last consumed Place the patient on an O2 sat monitor

Ausculate breath sounds over the trachea and bronchi

A client has undergone transurethral resection of the prostate (TURP). Which intervention does the nurse incorporate in the postoperative plan of care? (Select all that apply.) Select all that apply. Perform intermittent urinary catheterization every 4 to 6 hours. Assist to mobilize as soon as permitted. Encourage urination around the catheter if pressure is felt. Administer antispasmodic medications. Place in a supine position with his knees flexed.

Assist to mobilize as soon as permitted. Administer antispasmodic medications. Antispasmodic drugs can be administered to decrease the bladder spasms that may occur due to catheter use. Assisting the client to a chair as soon as permitted postoperatively will help to decrease the risk of complications from immobility. An indwelling catheter and continuous bladder irrigation are in place for about 24 hours after TURP.The client would not try to void around the catheter. This would cause the bladder muscles to contract and may result in painful spasms. Intermittent urinary catheterization is not necessary and increases the risk for infection. Typically, the catheter is taped to the client's thigh, so he needs to keep his leg straight.

67. An older client with benign prostatic hyperplasia and hypertension is being treated with Doxazosin while in the hospital. Which activity does the nurse delegate to the unlicensed assistive personnel as a priority? Helping the client choose low sodium meal items Frequently re-orienting the client to his surroundings Encouraging the client to use the incentive spirometer hourly Assisting the client whenever he get out of bed

Assisting the client whenever he get out of bed When treating a client in an inpatient setting with alpha blockers such as doxazosin (Cardura) or terazosin (Hytrin), the nurse must provide for the client's safety because this medication can cause orthostatic hypotension or syncope. The nurse should instruct the UAP to help the client whenever he gets out of bed, to prevent injury. Because this medication is being used for BPH and not for hypertension, a low-sodium diet is not necessary. Using the spirometer is always a good intervention, but it use is not related to safety and to this medication. The client, although older, may not be confused and may not need frequent reorientation.

What task does the RN delegate to assistive personnel (AP) working on the medical-surgical unit? Inserting a catheter in a client who has a history of uterine prolapse Assisting with a sitz bath for a client who is postposterior colporrhaphy Giving report to a receiving nurse about a client who is being transferred Providing discharge teaching for a client who is scheduled for brachytherapy

Assisting with a sitz bath for a client who is postposterior colporrhaphy Assisting with a client's sitz bath is within the AP's scope of practice and can safely be delegated.Some specially trained APs do catheterize clients, but a client with a uterine prolapse poses additional problems and needs to be managed by a licensed nurse. Giving report to a receiving nurse about a client who is being transferred is an interaction that must be accomplished be "nurse-to-nurse." Providing discharge teaching for a client who will be having brachytherapy (intracavitary radiation) is a complex nursing action and must be done by an RN.

A client with benign prostatic hyperplasia is being discharged with a prescription for prazosin. Which teaching will the nurse include? (Select all that apply.) Select all that apply. Avoid drugs used to treat erection problems. Take the medication in the afternoon. Keep all appointments for follow-up laboratory testing. Hearing tests will need to be conducted periodically. Be careful when changing positions.

Avoid drugs used to treat erection problems. Keep all appointments for follow-up laboratory testing. Be careful when changing positions. Drugs used to treat erectile dysfunction can worsen side effects, such as hypotension. Alpha-adrenergic blockers may cause orthostatic hypotension and can cause liver damage. The nurse needs to remind the client to be careful when changing positions and to keep all appointments for follow-up laboratory testing.These drugs do not affect hearing. Prazosin should be taken in the evening to decrease the risk of problems related to hypotension.

A client with a recently created vascular access for hemodialysis is being discharged. Which teaching will the nurse include in the discharge instructions? How to practice proper nutrition? Avoiding venipuncture and blood pressure measurements in the affected arm How to assess for a bruit in the affected arm? Modifications to allow for complete rest of the affected arm

Avoiding venipuncture and blood pressure measurements in the affected arm The nurse must teach the client to avoid venipunctures and blood pressure measures in the arm that contains the newly created vascular access device. Compression of vascular access causes decreased blood flow and may cause occlusion. If this occurs, lifesaving dialysis will not be possible.The arm with the access device must be exercised to encourage venous dilation, not rested. The client can palpate for a thrill, but a stethoscope is needed to auscultate the bruit at home. The nurse needs to take every opportunity to discuss nutrition, even as it relates to wound healing, but loss of the venous access device must take priority.

8. Which condition is the client most likely to report to the nurse after uterine prolapse? A. "I have to use the bathroom much more often than I used to." B. "It feels like something is falling out down there." C. "I leak urine whenever I laugh, cough, or sneeze." D. "I've had two urinary tract infections in the last 3 months."

B. "It feels like something is falling out down there."

Which client's previous health history will the nurse most associate with a risk for developing postnecrotic cirrhosis of the liver? A. 28-year-old woman who had gallstones 1 year ago and has recently lost 20 lb (9 kg) on a low-calorie, low-fat diet B. 45-year-old man with hepatitis C infection and chronic use of acetaminophen C. 50-year-old man who has many years of excessive alcohol consumption D. 55-year-old woman who has chronic biliary obstruction

B. 45-year-old man with hepatitis C infection and chronic use of acetaminophen

3. What are the nurse's best actions when a client with uterine leiomyomas reports pelvic pressure, constipation, and urinary retention? Select all that apply. A. Check the lower extremities for fluid retention. B. Assess the abdomen for distention or enlargement. C. Measure fluid intake and urinary output. D. Ask the client if her pants fit tighter now. E. Measure residual urine using a urinary catheter. F. Measure the abdomen as a baseline for comparison.

B. Assess the abdomen for distention or enlargement. D. Ask the client if her pants fit tighter now. F. Measure the abdomen as a baseline for comparison.

15. Which action will the nurse expect to take when a client diagnosed and treated for ovarian cancer has a recurrence? A. Arranging discussions with others who have recurring cancer B. Assessing readiness to explore palliative care and hospice C. Assisting to identify complementary therapies that may be helpful for palliation D. Teaching about radical hysterectomy followed by brachytherapy

B. Assessing readiness to explore palliative care and hospice

Which health and lifestyle habits does the nurse assess when taking a history from a client with a reproductive health problem? Select all that apply. A. Socioeconomic status B. Dietary intake C. Exercise pattern D. Sleep pattern E. Sexual practices F. Occupational status

B. Dietary intake C. Exercise pattern D. Sleep pattern E. Sexual practices

Which common factors will the nurse recognize as contributing to or worsening of hepatic encephalopathy in clients with liver cirrhosis? Select all that apply. A. Anorexia B. Infection C. Opioids D. Diarrhea E. GI bleeding F. High-protein diet G. Diabetes mellitus H. Chronic confusion

B. Infection C. Opioids E. GI bleeding F. High-protein diet Factors that may contribute to HE include high protein diet, infection, hypovolemia, hypokalemia, constipation, GI bleed, (causes large protein load in the intestines), some drugs like hypnotics, opioids, sedatives, analgesics, diuretics, illicit drugs

Which serum electrolyte value in a client with early-stage ascites from chronic liver disease who is taking spironolactone will the nurse report immediately to the primary health care provider? A. Sodium 133 mEq/L (mmol/L) B. Potassium 6.4 mEq/L (mmol/L) C. Chloride 101 mEq/L (mmol/L) D. Calcium 8.9 mg/dL (2.2 mmol/L)

B. Potassium 6.4 mEq/L (mmol/L)

Which change in laboratory values or clinical symptoms in a client with hypovolemic shock indicates to the nurse that current therapy may need to be changed? A. Urine output increases from 5 mL/hr to 6 mL/hr. B. Pulse pressure decreases from 28 mm Hg to 22 mm Hg. C. Serum potassium level increases from 3.6 mEq/L to 3.9 mEq/L. D. Core body temperature increases from 98.2°F (36.8°C) to 98.8°F (37.1°C).

B. Pulse pressure decreases from 28 mm Hg to 22 mm Hg. A compensatory response to shock is vasoconstriction. Initially, the diastolic pressure increases but systolic pressure remains the same. As a result, the difference between the systolic and diastolic pressures (pulse pressure), is smaller or "narrower." When interventions are inadequate and shock worsens, systolic pressure decreases as cardiac output decreases. This causes the pulse pressure to narrow even further, indicating that shock is progressing. Although an increase in urine output usually signals improvement, a change of 1 1 mL/hr is within the margin of measurement error and is meaningless in this situation.

18. Which instruction will the nurse provide to the client who is currently receiving brachytherapy for gynecologic cancer? A. Call for help when getting out of bed. B. Stay in bed during the treatment session. C. Use a separate bathroom when you go home. D. Avoid being near children when you are home.

B. Stay in bed during the treatment session.

The nurse is caring for a client who had a Whipple surgical procedure yesterday. For what serum laboratory test results would the nurse want to monitor frequently and carefully? Blood glucose Blood urea nitrogen Phosphorus Platelet count

Blood glucose During a Whipple procedure, most or all of the pancreas is manipulated, stressed, and possibly removed. Therefore, the client is at risk for hyperglycemia or hypoglycemia and blood glucose would need careful monitoring with a possible need for treatment.

Which advantages of minimally invasive surgery (MIS) laparoscopic cholecystectomy will the nurse reinforce to a client after the surgeon provided information for informed consent?

Bile duct injuries are rare. Complications are uncommon. Postoperative pain is less severe. Depending on the nature of the job some clients can return to work within 1 to 2 weeks

The nurse is reviewing discharge plans with a client who is recovering from a cervical biopsy. Which statements indicate good understanding by the client? (Select all that apply.) a. I can return to work this afternoon. b. There should be no problem lifting my 2-year-old toddler when I get home. c. I cannot douche until the biopsy site is healed. d. I need to wait for about 2 weeks to have intercourse with my husband. e. If I have some bleeding, I can use a regular tampon this evening.

C, D The client should not douche, have intercourse, or use tampons until the biopsy site is healed. The client should rest for 24 hours after the procedure and should not lift heavy objects.

23. Which preventive measure will the nurse include when teaching a group of female clients about prevention of toxic shock syndrome? A. "Use sanitary napkins on heavy flow days." B. "Use superabsorbent tampons only at night." C. "Change the tampon every 3 to 6 hours." D. "Void before and immediately after intercourse."

C. "Change the tampon every 3 to 6 hours."

CHECK THIS 3. Which client does the nurse consider to be at highest risk for neural-induced distributive shock? A. 25-year-old receiving 500 mg of penicillin IV B. 47-year-old with sudden-onset severe chest pain and dyspnea C. 21-year-old who has received 4 mg of morphine IV for acute pain D. 82-year-old who has had severe vomiting and diarrhea for 2 days

C. 21-year-old who has received 4 mg of morphine IV for acute pain Both acute pain and morphine can lead to neural-induced distributive shock. At first, acute pain stimulates the sympathetic nervous system. However, the parasympathetic system over-rides the sympathetic division. This interference together with morphine acting within the central nervous system result in decreased sympathetic tone to blood vessel smooth muscles, causing widespread vasodilation and reduced mean arterial pressure. Option A is incorrect. The type of shock possibly produced by a reaction to penicillin is anaphylactic shock, a type of chemical-induced distributive shock. Option B is incorrect. The type of shock associated with sudden-onset severe chest pain and dyspnea is cardiogenic shock from an acute myocardial infarction. Option D is incorrect. The type of shock associated with prolonged vomiting and diarrhea is hypovolemic shock with true volume depletion as a result of dehydration.

Which actions will the nurse perform when preparing a client for paracentesis? Select all that apply. A. Obtaining informed consent B. Maintaining the client on NPO status C. Asking the client to void before the procedure D. Placing the client in the flat supine position E. Weighing the client before the procedure F. Assessing the respiratory rate and blood pressure

C. Asking the client to void before the procedure E. Weighing the client before the procedure F. Assessing the respiratory rate and blood pressure

What is the nurse's priority action when a client with ascites reports increased abdominal pain and chills? A. Applying oxygen and making the client NPO B. Notifying the primary health care provider immediately C. Assessing for abdominal rigidity and taking the client's temperature D. Applying a heating blanket and raising the head of the bed to a 45-degree angle

C. Assessing for abdominal rigidity and taking the client's temperature

13. Which care action will the nurse assign to the assistive personnel (AP) for a client receiving external radiation therapy for treatment of endometrial cancer? A. Gently washing the markings outlining the treatment site B. Monitoring for skin breakdown in the perineal area C. Assisting the client to ambulate to the bathroom if she feels fatigued D. Assessing the urinary catheter and urine output for color and odor

C. Assisting the client to ambulate to the bathroom if she feels fatigued

12. Which assessment has the highest priority for the nurse to perform to prevent harm when caring for a client in hypovolemic shock who is receiving IV sodium nitroprusside? A. Asking about chest pain B. Determining mental status C. Checking blood pressure every 15 minutes D. Checking extremities for color and perfusion

C. Checking blood pressure every 15 minutes Sodium nitroprusside improves myocardial perfusion by dilating coronary arteries rapidly for a short time. This action also occurs in systemic blood vessels and can result in dangerously low blood pressure. Thus, the nurse assesses blood pressure every 15 minutes while a client is receiving this drug. The drug does not cause chest pain and does not cause peripheral vasoconstriction that could interfere with perfusion of extremities. The drug's effect on mental status is related to blood pressure decreases.

What liver problem does the nurse suspect in a client whose liver is hard with a nodular texture and the hepatic enzymes remain normal? A. Prenecrotic inflammation B. Postnecrotic inflammation C. Compensated cirrhosis D. Decompensated cirrhosis

C. Compensated cirrhosis

When the nurse takes a history from a male client and finds that he had mumps with orchitis, which common potential complication is possible? A. Decreased libido B. Prostate hypertrophy C. Testicular atrophy D. Impotence

C. Testicular atrophy

When caring for a client who is obtunded and admitted with shock of unknown origin, which action will the nurse take first? Obtain IV access and hang prescribed fluid infusions. Assess level of consciousness and pupil reaction to light. Apply the automatic blood pressure cuff. Check the airway and respiratory status.

Check the airway and respiratory status. The nurse's first action when caring for an obtunded client admitted with shock is to check the client's airway and respiratory status. When caring for any client, determining airway and respiratory status is the priority.The airway takes priority over obtaining IV access, applying the blood pressure cuff, and assessing for changes in the client's mental status.

31. During the first 4 hours after a transurethral resection of the prostate, 1200 ml of bladder irrigation solution was infused through the patient's 3-way catheter. The nurse emptied the catheter drainage bag and measured the total drainage as 1000 ml. Based on this information, the nurse should first: Check the catheter patency Increase the IV flow rate Encourage the patient to increase his oral fluid intake Slow the rate of the bladder irrigation

Check the catheter patency You should continuously monitor the inflow and outflow of the irrigant. If outflow is less than inflow, assess the catheter for kinks or clots. If the outflow is blocked and patency cannot be reestablished by manual irrigation, stop the CBI and notify the physician.

Which client will the nurse assess as at risk for acute kidney injury (AKI)? (Select all that apply.) Select all that apply. Client in the intensive care unit on high doses of antibiotics Football player in preseason practice Accident victim recovering from a severe hemorrhage Accountant with poorly controlled diabetes mellitus Client who underwent contrast dye radiology Client recovering from gastrointestinal influenza

Client in the intensive care unit on high doses of antibiotics Football player in preseason practice Accident victim recovering from a severe hemorrhage Client who underwent contrast dye radiology Client recovering from gastrointestinal influenza To prevent AKI, all people must be urged to avoid dehydration by drinking at least 2 to 3 L of fluids daily, especially during strenuous exercise or work associated with diaphoresis, or when recovering from an illness that reduces kidney blood flow, such as influenza. Contrast media may cause acute renal failure, especially in older clients with reduced kidney function. Recent surgery or trauma, transfusions, or other factors that might lead to reduced kidney blood flow may cause AKI. Certain antibiotics may cause nephrotoxicity.Poorly controlled diabetes mellitus is a risk factor for chronic kidney disease.

The RN has just received change-of-shift report. Which client will the nurse assess first? Client with azotemia whose blood urea nitrogen and creatinine are increasing. Client with kidney insufficiency who is scheduled to have an arteriovenous fistula inserted. Client receiving peritoneal dialysis who needs help changing the dialysate bag. Client with chronic kidney failure who was just admitted with shortness of breath.

Client with chronic kidney failure who was just admitted with shortness of breath. After the change-of-shift report, the nurse must first assess the newly admitted client with chronic kidney failure and shortness of breath, the dyspnea of the client with chronic kidney failure may indicate pulmonary edema and must be assessed immediately.The client with kidney insufficiency is stable and assessment can be performed later. The client with azotemia requires assessment and possible interventions but is not at immediate risk for life-threatening problems. The client receiving peritoneal dialysis can be seen last because it is a slow process and the client has no urgent needs.

Which assessment technique will the nurse use to most accurately determine increasing ascites in a client with advanced liver cirrhosis and portal hypertension? A. Interpreting the serum albumin value B. Measuring the client's abdominal girth C. Testing stool for the presence of occult blood D. Weighing the client daily at the same time of the day

D. Weighing the client daily at the same time of the day

Which of the following would indicate to the nurse that the client has developed the most serious complication associated with peritoneal dialysis? Leakage of dialysate through the catheter site Cloudy dialysate drainage Decreased hemoglobin and hematocrit Increased BUN and creatinine

Cloudy dialysate drainage

22. Which complication will the nurse remain alert for in a client who has septic shock? A. Psychosis B. Skin necrosis C. Febrile seizures D. Acute respiratory distress syndrome (ARDS)

D. Acute respiratory distress syndrome (ARDS) The lungs are susceptible to damage during septic shock, and the complication of ARDS may occur. ARDS in septic shock is caused by the continued systemic inflammatory response syndrome (SIRS) increasing the formation of oxygen free radicals, which damage lung cells.

6. Which client does the nurse understand is most suitable to undergo a uterine artery embolization procedure for leiomyomas?A. 18-year-old who is unmarried B. 25-year-old who is engaged to be married C. 35-year-old who is married and wants a child D. 46-year-old who is divorced with four children

D. 46-year-old who is divorced with four children

What finding does the nurse expect when re-viewing a male client's laboratory results and finding a testosterone level of 200 ng/dL? A. Increased muscle mass and weight B. Increase in urinary output C. Testicular pain with nausea D. Changes in sexual performance

D. Changes in sexual performance

26. Which blood product type does the nurse anticipate will be ordered to infuse first in a client who has septic shock with poor clotting and hemorrhage? A. Packed red blood cells (PRBCs) B. Fresh frozen plasma (FFP) C. Clotting factors D. Platelets

D. Platelets In a client with septic shock who has poor clotting and bleeding, it is most likely that disseminated intravascular coagulation (DIC) has occurred and consumed all of the client's available platelets, which are an absolute necessity for appropriate clotting to occur. These are given first.

15. For which change in condition will the nurse teach a client, who is discharged and at continued risk for fluid loss, to check for daily at home? A. Elevated temperature and itchiness B. Loss of taste sensation and appetite C. Numbness of the fingers and toes D. Reduced urine output and light-headedness

D. Reduced urine output and light-headedness Reduced urine output and light-headedness are early indicators of hypovolemia related to fluid loss. The other options are not associated with any problem related to hypovolemia resulting from fluid loss.

14. What primary factor does the nurse understand is the reason for low survival rates in clients with ovarian cancer? A. Ovarian cancer develops in clients with underlying immunosuppression and poor health. B. There are no specific diagnostic tests that can confirm or rule out ovarian cancer. C. Ovarian cancer does not respond well to conventional radiation or chemotherapy treatments. D. Symptoms are mild and vague, so the cancer is often not detected until in its late stage.

D. Symptoms are mild and vague, so the cancer is often not detected until in its late stage.

2. Why are the clinical signs and symptoms of most types of shock the same regardless of what specific events or conditions caused the shock to occur? A. The blood, blood vessels, and heart are directly connected to each other so that when one is affected, all three are affected. B. Because blood loss occurs with all types of shock, the most common first manifestation is hypotension. C. Every type of shock interferes with oxygenation and metabolism of all cells in the same sequence. D. The sympathetic nervous system is triggered by any type of shock and initiates the stress response.

D. The sympathetic nervous system is triggered by any type of shock and initiates the stress response. Most clinical signs and symptoms of shock are similar regardless of what starts the process or which tissues are affected first. These common changes result from physiologic adjustments (compensatory mechanisms) in the attempt to ensure continued oxygenation of vital organs. These adjustment actions are performed by the sympathetic nervous system triggering the stress response and activating the endocrine and cardiovascular systems.

The nurse is caring for a client in the refractory stage shock. Which intervention does the nurse consider? Enrollment in a cardiac transplantation program Admission to rehabilitation hospital for ambulatory retraining Collaboration with home care agency for return to home Discussion with family and provider regarding palliative care

Discussion with family and provider regarding palliative care When caring for a client in the refractory stage of shock, the nurse considers discussing palliative care with the family and provider. In this irreversible phase, therapy is not effective in saving the client's life, even if the cause of shock is corrected and mean arterial pressure temporarily returns to normal. A discussion on palliative care would be considered.Rehabilitation or returning home is unlikely. The client with sustained tissue hypoxia is not a candidate for organ transplantation.

The nurse is teaching a client how to prevent vulvovaginitis. What information does the nurse include? Cleanse the inner labia daily with soap and water. Monthly douching may help reduce symptoms. Do not have unprotected sex with multiple partners. Wear snug-fitting polyester undergarments to prevent chafing.

Do not have unprotected sex with multiple partners. Unprotected sex with multiple partners can lead to vaginal infection.Tight clothing, such as pantyhose or tight jeans, would be avoided because it can cause chafing. Tight clothing can also cause the vaginal area to get hot and sweaty, which can lead to infection. Encourage the client to wear breathable fabrics such as cotton and to avoid irritants or allergens in products such as laundry detergents or bath products. During a bath or shower, the inner labial mucosa would be cleansed with only water, not soap. Soap is an irritant to the sensitive skin in those areas. The use of douches or feminine hygiene sprays is not recommended because they disturb the balance of both pH and bacteria and can aggravate irritation.

25. A client has returned to the nursing unit after a total abdominal hysterectomy. During the initial assessment, the nurse auscultates the client's abdomen and does not hear bowel sounds. Which is the nurse's priority intervention? Document the finding in the chart. Irrigate the nasogastric tube. Measure abdominal girth. Position the client on the right side.

Document the finding in the chart. Absence of bowel sounds for 1 to 2 days after abdominal surgery is an expected finding that should be documented. No intervention specific to this finding is needed.

When assisting with dietary protein needs for a client on peritoneal dialysis, the nurse recommends which food selection? Eggs Ham Eggplant Macaroni

Eggs The nurse recommends eggs as a dietary protein need for a client on peritoneal dialysis. Other suggested protein-containing foods for this client are milk and meat.Although a protein, ham is high in sodium and needs to be avoided. Vegetables and pasta contain mostly carbohydrates. Peritoneal dialysis clients are allowed 1.2 to 1.5 g of protein/kg/day because protein is lost with each exchange.

The nurse is reviewing laboratory results of a client recently admitted with a diagnosis of acute pancreatitis. Which values would the nurse expect to be elevated? (Select all that apply.) Select all that apply. Elastase Amylase Glucose Lipase Trypsin Calcium

Elastase Amylase Glucose Lipase Trypsin The client who has acute pancreatitis experiences elevation of all pancreatic enzymes and glucose. The serum calcium level is usually decreased (rather than elevated) because the release of fatty acids combined with available calcium.

The nurse plans to administer an antibiotic to a client newly admitted with septic shock. What action will the nurse take first? Take the client's vital signs. Ensure that blood cultures were drawn. Insert an intravenous line. Administer the antibiotic.

Ensure that blood cultures were drawn. The nurse's first action when planning to administer an antibiotic to a newly admitted patent in septic shock is to ensure that blood cultures were drawn. Cultures must be taken to identify the organism for more targeted antibiotic treatment before antibiotics are administered. Antibiotics are not administered until after all cultures are taken.An intravenous line will be needed, but the nurse must ensure that blood cultures have been drawn. Monitoring the client's vital signs is important, but the antibiotic must be administered within 1 hour of shock recognition.

Which of the following is most likely to put a client with cirrhosis at risk for hepatic encephalopathy? Gastrointestinal bleeding Urinary tract infection Nausea and vomiting Dehydration

Gastrointestinal bleeding

A client is receiving immune-suppressive therapy after kidney transplantation. Which measure for infection control is important for the nurse to implement? Adherence to therapy Handwashing Monitoring for low-grade fever Strict clean technique

Handwashing Handwashing is the most important infection control measure for the client receiving immune-suppressive therapy to perform.Adherence to therapy and monitoring for low-grade fever are important but are not infection control measures. The nurse must practice aseptic technique for this client, not simply clean technique.

The nurse is caring for a postoperative client at risk for hypovolemic shock. Which assessment indicates an early sign of shock? First-degree heart block Blood pressure 100/48 mm Hg Respiratory rate 12 breaths/min Heart rate 120 beats/min

Heart rate 120 beats/min Tachycardia is an early symptom of shock. Heart and respiratory rates increased from the client's baseline level or a slight increase in diastolic blood pressure may be the only objective manifestation of this early stage of shock. Catecholamine release occurs early in shock as a compensation for fluid loss; blood pressure will be normal (not hypotensive). Early in shock, the client displays rapid, not slow, respirations. Dysrhythmias are a late sign of shock; they are related to lack of oxygen to the heart.Catecholamine release occurs early in shock as a compensation for fluid loss; blood pressure will be normal and not abnormally low. Early in shock, the client displays rapid, not slow, respirations. Dysrhythmias are a late sign of shock and are related to lack of oxygen to the heart.

What is the nurse's priority action when caring for a client with acute cholecystitis who now has severe abdominal pain, diaphoresis, heart rate of 118 beats/min, BP 95/70, respirations 32 breaths/min, and temperature 101 F (38.3°C?

Initiating the Rapid Response Team

The nurse is caring for a client who recently had an external percutaneous transhepatic biliary catheter placed for severe biliary obstruction. What is the nurse's priority intervention when caring for this client? Keeping the biliary drainage bag below the level of the catheter-insertion site Checking the client's blood glucose frequently to monitor for diabetes Managing pain with continuous opioid patient-controlled analgesia (PCA) Capping the catheter if it starts to leak around the insertion site

Keeping the biliary drainage bag below the level of the catheter-insertion site The client who has an external percutaneous transhepatic biliary catheter drains by gravity and therefore needs to have the drainage bag placed lower that the catheter-insertion site. The catheter is not capped if jaundice or leakage around the catheter site occurs. Opioids are not needed while the client has the catheter; however, if it is in place for an extended period of time, it needs to be changed.

How does the nurse caring for a client with septic shock recognize that severe tissue hypoxia is present? Lactate 81 mg/dL (9.0 mmol/L) Partial thromboplastin time 64 seconds Potassium 2.8 mEq/L (2.8 mmol/L) PaCO2 58 mm Hg

Lactate 81 mg/dL (9.0 mmol/L) The client with septic shock and a lactate level of 81 mg/dL (0.9 mmoL/L) indicates that severe tissue hypoxia is present. Poor tissue oxygenation at the cellular level causes anaerobic metabolism, with the by-product of lactic acid.Elevated partial pressure of carbon dioxide occurs with hypoventilation, which may be related to respiratory muscle fatigue, secretions, and causes other than hypoxia. Coagulation times reflect the ability of the blood to clot, not oxygenation at the cellular level. Elevation in potassium appears in septic shock due to acidosis, but this value is decreased and is not consistent with septic shock.

Which clinical symptoms in a postoperative client indicate early sepsis with an excellent recovery rate if treated? Reduced urinary output and increased respiratory rate Low-grade fever and mild hypotension Low oxygen saturation rate and decreased cognition Localized erythema and edema

Low-grade fever and mild hypotension Low-grade fever and mild hypotension in a postoperative client can indicate very early sepsis. With treatment, the probability of recovery is high.Localized erythema and edema indicate local infection. A low oxygen saturation rate and decreased cognition indicate severe sepsis. Reduced urinary output and increased respiratory rate indicate active (not early) sepsis.

The nurse is teaching a class on the management of sepsis. What teaching will the nurse include regarding the Hour-1 sepsis management bundle? (Select all that apply.) Select all that apply. Measure fibrinogen levels. Measure lactate levels. Initiate insulin therapy according to blood glucose levels. Administer broad spectrum antibiotics. Begin rapid administration of crystalloids for hypotension. A bundle is a group of two or more interventions that has been shown to be effective when applied in a sequence.

Measure lactate levels. Administer broad spectrum antibiotics. Begin rapid administration of crystalloids for hypotension. A bundle is a group of two or more interventions that has been shown to be effective when applied in a sequence. A care bundle is a group of two or more interventions that have been shown to be effective when applied in a timely sequence. The following are included in the Hour-1 Sepsis bundle:1. Measure lactate level.2. Obtain blood cultures before administering antibiotics.3. Administer broad-spectrum antibiotics.4. Begin rapid administration of 30 mL/kg crystalloid for hypotension or lactate ≥4 mmol/L.5. Apply vasopressors if hypotensive during or after fluid resuscitation to maintain a mean arterial pressure ≥65 mm Hg.

Which nursing assessment has the highest priority for the nurse to perform on a client admitted in severe pain with acute pancreatitis?

Measuring heart rate, blood pressure, and oxygen saturation

After returning from transurethral resection of the prostate, the client's urine in the continuous bladder irrigation system is a burgundy color. Which client needs does the nurse anticipate will be prescribed after the surgeon sees the client? (Select all that apply.) Select all that apply. Encouragement of oral fluids Emergency surgery Monitoring for anemia Antispasmodic drugs Increased intermittent irrigation

Monitoring for anemia Antispasmodic drugs Although not a common occurrence, bleeding may occur in the postoperative period. Venous bleeding, which is deep red or burgundy in color, is more common than arterial bleeding. The surgeon may apply traction on the catheter for a few hours to control the venous bleeding. Traction on the catheter is uncomfortable and increases the risk for bladder spasms, so analgesics or antispasmodics is usually prescribed. Hemoglobin and hematocrit would be monitored and trended for indications of anemia.Emergency surgery and increased intermittent irrigation would be indicated for an arterial bleed, which would be a bright red color. Encouragement of oral fluids are indicated after the catheter is removed.

A client is being discharged after a total abdominal hysterectomy (TAH). What principle guides the nurse in planning discharge care and teaching? Clients in their childbearing years generally adapt better. Psychological reactions would be evident by discharge. The client can resume normal activities upon discharge. No special home equipment will be necessary for the client.

No special home equipment will be necessary for the client. Usually, no special home equipment is needed for the client who has undergone a TAH.Generally, clients adjust better to the surgery if they have completed their childbearing years. Psychological reactions can occur months to years after surgery, particularly if sexual functioning and libido are diminished. The client who has undergone a TAH would be taught about the expected physical changes, including activity restrictions. A 2- to 6-week convalescent period is usually required.

The nurse is caring for a client who states that her mother had "gallbladder problems" and wonders if she is at risk for this disorder. What major risk factor places women most at risk for gallbladder disease? Obesity Birth control pills Infertility Advanced age

Obesity Obese women who are middle age and have had multiple children are at the highest risk for gallbladder disease, although it can occur in anyone.

A nurse is assessing the patency of a client's left arm arteriovenous fistula prior to initiating hemodialysis. Which finding indicates that the fistula is patent? Absence of a bruit on auscultation of the fistula Palpation of a thrill over the fistula Presence of a radial pulse in the left wrist Capillary refill less than 3 seconds in the nail beds of the fingers on the left hand

Palpation of a thrill over the fistula

Which complication in a client with acute necrotizing pancreatitis who develops a temperature spike to 104°F (40°C) will the nurse suspect?

Pancreatic abscess

The emergency room nurse is caring for an elderly female client who appears to be in the oliguric stage of acute renal failure due to dehydration. Which of the following findings supports this diagnosis?[ Select all that apply]. Magnesium = 1.3 mEq/L ABGs; pH = 7.43 Potassium - 6.0 mEq/L Crackles in bilateral lower lung fields Creatinine = 4.2 mg/dl

Potassium - 6.0 mEq/L Crackles in bilateral lower lung fields Creatinine = 4.2 mg/dl

What will the nurse include in teaching to assist the client with BPH to address incontinence? Perform routine self-catheterization and bladder retraining. Practice Kegel exercises to improve muscle control. Request a prescription for sildenafil. Use analgesic medications as needed to maintain comfort.

Practice Kegel exercises to improve muscle control. Practicing Kegel exercises may help to regain urinary continence.Sildenafil is used to help restore erectile function, not for incontinence. Analgesic medications do not help with urinary continence. Self-catheterization and bladder retraining are used for neurogenic bladder.

A client has been placed on enzyme replacement for treatment of chronic pancreatitis. In teaching the client about this therapy, the nurse advises the client not to mix enzyme preparations with foods containing which element? High fat High fiber Carbohydrates Protein

Protein The nurse tells the client not to mix enzyme preparations with foods containing protein because the enzymes will dissolve the food into a watery substance. Pancreatic-enzyme replacement therapy (PERT) is the standard of care to prevent malnutrition, malabsorption, and excessive weight loss. Pancrelipase is usually prescribed in capsule or tablet form and contains varying amounts of amylase, lipase, and protease.No evidence suggests that enzyme preparations should not be mixed with carbohydrates, food with highfat content, and food with high-fiber content.

The nurse is obtaining a personal health history on a 21-year-old male. How does the nurse approach questions about his sexual practices? Ask if the client wants to have his partner present for the health history. Defer questions about his sexual practices to the health care provider. Skip questions about sexual practices as he is unlikely to be sexually active. Respect the client's choice to answer or not answer questions about sexual practices.

Respect the client's choice to answer or not answer questions about sexual practices. Respecting the client's choice to answer or not answer questions about sexual practices is an important part of the process of taking the sexual history of any client.Deferring questions about sexual practices to the primary health care provider or skipping questions is inappropriate, as important health information may be missed. All adult clients should be offered the opportunity to discuss sexual practices as these relate to patient-centered care.

The nurse is caring for a client with chronic kidney disease who has developed uremia. Which assessment finding does the nurse correlate with this problem? Decreased breath sounds Respiratory rate of 40 breaths/min Heart rate of 50 beats/min Foul-smelling urine

Respiratory rate of 40 breaths/min A client with uremia will also have metabolic acidosis. With severe metabolic acidosis, the client will develop hyperventilation, or Kussmaul respirations, as the body attempts to compensate for the falling pH. The other manifestations would not be associated with acidosis.

The nurse is teaching dietary modification to a client with acute kidney injury (AKI). What dietary teaching will the nurse include? (Select all that apply.) Select all that apply. Liberal sodium Low fat Restricted fluids Restricted protein Low potassium

Restricted fluids Restricted protein Low potassium A client with acute kidney injury needs to modify the diet to include restricted protein, restricted fluids, and low potassium. Breakdown of protein leads to azotemia and increased blood urea nitrogen. For the client who does not require dialysis, 0.6 g/kg of body weight or 40 g/day of protein is usually prescribed. For clients who do require dialysis, the protein level needed will range from 1 to 1.5 g/kg. Fluid is restricted during the oliguric stage. The daily amount of fluid permitted is calculated to be equal to the urine volume plus 500 mL. Potassium intoxication may occur, so dietary potassium is also restricted. Dietary potassium is restricted to 60 to 70 mEq/kg (70 mmol/kg).Sodium is restricted during AKI because oliguria causes fluid retention. Dietary sodium recommendations range from 60 to 90 mEq/kg (60 to 90 mmol/kg). Fats may be used for needed calories when proteins are restricted.

A client has been admitted to the hospital with a diagnosis of acute pancreatitis. The client is complaining of severe mid-epigastric pain. His abdomen is distended and firm. Bowel sounds are hypoactive. VS are BP = 98/56, HR=130, RR = 18, T = 100.6. Which one of the following nursing diagnoses has the highest priority at this time? Ineffective thermoregulation Risk for deficient fluid volume Risk for infection Chronic pain R/T infectious process

Risk for deficient fluid volume

When providing care for a client receiving peritoneal dialysis, the nurse notices that the effluent is cloudy. Which intervention is most important for the nurse to carry out? Irrigate the peritoneal catheter with saline Send a specimen for culture and sensitivity. Document the finding in the client's chart. Change the dialysate solution and catheter tubing.

Send a specimen for culture and sensitivity. Cloudy or opaque effluent is the earliest sign of peritonitis. The health care provider should be notified, and a sample of the outflow should be sent for culture and sensitivity. Irrigating the catheter or changing the solution and tubing will not help reveal the cause of the problem so that appropriate treatment can be started. Documentation is important but is not the priority.

A client is scheduled for a total hysterectomy with a laparoscopic vaginal approach after a diagnosis of microinvasive cervical cancer. What psychological and/or social changes does the nurse expect this client to experience? Sexual counseling may be needed, especially if the client has doubts about her ability to feel like a woman and engage in sexual activities. The client will be actively involved in her own care in the immediate postoperative period. Because the surgery does not affect a visible site, altered body image issues are not as common. The client would demonstrate reality testing and would experience a grief reaction immediately after her surgery.

Sexual counseling may be needed, especially if the client has doubts about her ability to feel like a woman and engage in sexual activities. Sexual function may be (or feel) different after a hysterectomy. Couples may need counseling about intercourse or alternative sexual activities. The nurse assesses the need for sexual counseling by listening for cues about altered perceptions of body image and anxiety in either of the sexual partners' responses.For many women, hysterectomy can mean the loss of their femininity, so altered body image issues must be expected with the client. Hysterectomy is major surgery, so the client will be recovering for days to a week or longer. Active involvement in her self-care will be delayed until she has moved past the initial surgical procedure recovery period. Reality testing is a later step in the grief and acceptance processes experienced by women who have had hysterectomies.

The nurse will review the side effects of the medication, Levofloxacin prior to administering the medication. The information contains the Black Box Warning for the drug which it that it may cause: Tendonitis, tendon rupture Severe leukocytosis Hyperkalemia Pulmonary edema

Tendonitis, tendon rupture

A client with pelvic pain is admitted to the same-day surgery unit for a laparoscopic procedure. Which nursing action will the RN delegate to assistive personnel (AP)? Taking admission blood pressure and heart rate Educating about analgesic use for referred pain Teaching about postoperative activity restrictions Inserting a retention catheter using sterile technique

Taking admission blood pressure and heart rate Although most of the admission assessment and history will be completed by the RN, the admission vital signs can be delegated to a AP.Client education and teaching is a higher-level skill and must be done by the RN. Catheter insertion is also a higher-level skill and would be done by the RN.

The nurse is caring for a client with acute kidney injury and a temporary subclavian hemodialysis catheter. Which assessment finding requires nursing action? Mild discomfort at the insertion site Temperature 100.8° F (38.2° °C) Anorexia 1+ ankle edema

Temperature 100.8° F (38.2° °C) In this client situation, the nurse reports an assessment finding of a temperature of 100.8° F (38.2° C) to the HCP. Infection is a major complication of temporary catheters. All symptoms of infection, including fever, must be reported to the provider because the catheter may have to be removed.Mild discomfort at the insertion site is expected with a subclavian hemodialysis catheter. During acute injury, oliguria with resulting fluid retention and 1+ ankle edema is expected. Rising blood urea nitrogen may result in anorexia, nausea, and vomiting.

A 68-year-old client has recently undergone a prostate biopsy. Which assessment finding will the nurse report to the health care provider? Rust-colored semen Slight rectal bleeding Mild pain and soreness at the site Temperature of 101.6° F (38.7° C)

Temperature of 101.6° F (38.7° C) Rarely, sepsis can develop after a prostate biopsy. However, clients need to be told to contact their primary health care provider immediately if they experience fever, pain when urinating, or penile discharge.Expected findings after a prostate biopsy may include slight soreness, light rectal bleeding, and blood in the urine or stools for a few days. Semen may be red or rust-colored for several weeks.

The nurse is reviewing a laboratory report that indicates a decrease in a client's estradiol level. How does the nurse interpret this information? The client is experiencing a normal pregnancy. The client may have a malignant tumor. The client may be in menopause. The client may be pregnant with twins.

The client may be in menopause. Decreased levels of estradiol in a client may indicate menopause, hypopituitarism, anorexia nervosa, or a possible pregnancy concern. This laboratory finding does not indicate a normal pregnancy, pregnancy with twins, or the presence of a malignant tumor.

The nurse in the dialysis center is initiating the morning dialysis run. Which client should the nurse assess first? The client who did not take antihypertensive medication this morning The client whose labs reflect: Hemoglobin = 9.8 g/dl/ Hematocrit = 30% The client who is complaining of being exhausted and is sleeping The client who does not have a palpable thrill or auscultated bruit

The client who does not have a palpable thrill or auscultated bruit. Rationale: This client's dialysis access is compromised and he or she should be assessed first.

The nurse is teaching a client with gallbladder disease about diet modification. Which meal would the nurse suggest to the client? Sausage and scrambled eggs Steak and french fries Turkey sandwich on wheat bread Fried chicken and mashed potatoes

Turkey sandwich on wheat bread Turkey is an appropriate low-fat selection for this client. High fiber, from the wheat bread, also helps reduce the risk. Typically, diets high in fat, high in calories, low in fiber, and high in refined white carbohydrates place clients at higher risk for developing gallstones.Steak, french fries, fried chicken and mashed potatoes, and sausage are too fatty. Eggs are too high in cholesterol for a client with gallbladder disease.

While assisting a client during peritoneal dialysis, the nurse observes the drainage stop after 200 mL of peritoneal effluent drains into the bag. What action will the nurse implement? Document the effluent as output. Instruct the client to cough. Reposition the catheter. Turn the client to the opposite side.

Turn the client to the opposite side. The nurse's first action in this situation is to turn the client to the opposite side. With peritoneal dialysis, usually 1 to 2 L of dialysate is infused by gravity into the peritoneal space. The fluid dwells in the peritoneal cavity for a specified time, then drains by gravity into a drainage bag. The peritoneal effluent or outflow generally is a continuous stream after the clamp is completely open. Potential causes of flow difficulty include constipation, kinked or clamped connection tubing, the client's position, fibrin clot formation, and catheter displacement. If inflow or outflow drainage is inadequate, reposition the client to stimulate inflow or outflow. Turning the client to the other side or ensuring that he or she is in good body alignment may help.Instructing the client to cough will not promote dialysate drainage. Increased abdominal pressure from coughing contributes to leakage at the catheter site. The nurse needs to measure and record the total amount of outflow after each exchange. However, the nurse needs to reposition the client first to assist with complete dialysate drainage. An x-ray is needed to identify peritoneal dialysis catheter placement. Only the health care provider repositions a displaced catheter.

The nurse is preparing to instruct a client with chronic pancreatitis who is to begin taking pancrelipase. Which instruction does the nurse include when teaching the client about this medication? Wipe your lips after taking pancrelipase. Take pancrelipase before meals. Administer pancrelipase before taking an antacid. Chew tablets before swallowing.

Wipe your lips after taking pancrelipase. The nurse will instruct the client to wipe the lips after taking pancrelipase. Pancrelipase is a pancreatic enzyme used for enzyme replacement for clients with chronic pancreatitis. To avoid skin irritation and breakdown from residual enzymes, the lips should be wiped.Pancrelipase should be administered after, and not before, antacids or histamine2 blockers are taken. It should not be chewed to minimize oral irritation and allow the drug to be released more slowly. It should be taken with meals and snacks, and not before, and followed with a glass of water.

The family of a client who has hepatic encephalopathy asks why the client is restricted to moderate amounts of dietary protein and has to take lactulose. What is an appropriate response by the nurse? a) "These interventions help to reduce the ammonia level." b) "These interventions help to prevent heart failure." c) "These interventions help the client's jaundice improve." d) "These interventions help to prevent nausea and vomiting."

a) "These interventions help to reduce the ammonia level." The client's high ammonia level has caused encephalopathy which can become so severe that it causes death. These interventions help to reduce ammonia in the body so that this condition does not worsen.

When assessing a client with hepatitis B, the nurse anticipates which assessment findings? (Select all that apply.) a) Right upper quadrant tenderness b) Itching c) Recent influenza infection d) Brown stool e) Tea-colored urine

a) Right upper quadrant tenderness b) Itching e) Tea-colored urine Assessment findings the nurse expects to find in a client with hepatitis B include brown, tea-, or cola-colored urine; right upper quadrant pain due to inflammation of the liver; and itching, irritating skin caused by deposits of bilirubin on the skin secondary to high bilirubin levels and jaundice.Hepatitis B virus, not the influenza virus, causes hepatitis B, which is spread by blood and body fluids. The stool in hepatitis may be tan or clay-colored, not typically brown.

5. A client is in shock and the nurse prepares to administer insulin for a blood glucose reading of 208 mg/dL (11.6 mmol/L). The spouse asks why the client needs insulin as the client is not a diabetic. What response by the nurse is best? a. "High glucose is common in shock and needs to be treated." b. "Some of the medications we are giving are to raise blood sugar." c. "The IV solution has lots of glucose, which raises blood sugar." d. "The stress of this illness has made your spouse a diabetic."

a. "High glucose is common in shock and needs to be treated." High glucose readings are common in shock, and best outcomes are the result of treating them and maintaining glucose readings in the range of 140 to 180 mg/dL (7.7 to 10 mmol/L. Medications and IV solutions may raise blood glucose levels, but this is not the most accurate answer. The stress of the illness has not "made" the client diabetic.

Which questions will the nurse ask a client to determine the presence of signs and symptoms of benign prostatic hyperplasia (BPH)? SATA a. "have you noticed a sensation of incomplete bladder emptying?" b. "have you recently experienced a testicular or bladder infection?" c. "have you noticed dribbling or leaking after you finish urination?" d. "how many times do you have to get up during the night to urinate?" e. "have you noticed blood at the start or at the end of urination?" f. "have you noticed increased force or size of your urine stream?"

a. "have you noticed a sensation of incomplete bladder emptying?" c. "have you noticed dribbling or leaking after you finish urination?" d. "how many times do you have to get up during the night to urinate?" e. "have you noticed blood at the start or at the end of urination?"

5. The nurse is caring for a client with suspected septic shock. What does the nurse prepare to do within 1 hour of the client being identified as possibly having sepsis? (Select all that apply.) a. Administer antibiotics. b. Draw serum lactate levels. c. Infuse vasopressors. d. Measure central venous pressure. e. Obtain blood cultures. f. Administer rapid bolus of IV crystalloids.

a. Administer antibiotics. b. Draw serum lactate levels. c. Infuse vasopressors. e. Obtain blood cultures. f. Administer rapid bolus of IV crystalloids. Within the first hour of suspecting severe sepsis, the nurse would draw (or facilitate) serum lactate levels, obtain blood cultures (or other cultures), administer antibiotics (after the cultures have been obtained), bGegRiAn DraEpiSdLadAmBi.niCstOraMtion of 30 mL/kg crystalloid for hypotension or lactate 4 mmol/L. and administer vasopressors if hypotensive during or after fluid resuscitation to maintain a mean arterial pressure 65 mm Hg. Initiating hemodynamic monitoring would be done after these "bundle" measures have been accomplished.

9. A client is receiving norepinephrine for shock. What assessment finding best indicates a therapeutic effect from this drug? a. Alert and oriented, answering questions b. Client denies chest pain or chest pressure c. IV site without redness or swelling d. Urine output of 30 mL/hr for 2 hours

a. Alert and oriented, answering questions Normal cognitive function is a good indicator that the client is receiving the benefits of norepinephrine. The brain is very sensitive to changes in oxygenation and perfusion. Norepinephrine can cause chest pain as an adverse reaction, so the absence of chest pain is good but does not indicate therapeutic effect. The IV site is normal. The urine output is normal, but only minimally so.

3. The nurse caring frequently for older adults in the hospital is aware of risk factors that place them at a higher risk for shock. For what factors would the nurse assess? (Select all that apply.) a. Altered mobility/immobility b. Decreased thirst response c. Diminished immune response d. Malnutrition e. Overhydration f. Use of diuretics

a. Altered mobility/immobility b. Decreased thirst response c. Diminished immune response d. Malnutrition f. Use of diuretics Immobility, decreased thirst response, diminished immune response, malnutrition, and use of diuretics can place the older adult at higher risk of developing shock. Overhydration is not a common risk factor for shock.

1. The nurse studying shock understands that the common signs and symptoms of this condition are directly related to which problems? (Select all that apply.) a. Anaerobic metabolism b. Hyperglycemia c. Hypotension d. Impaired renal perfusion e. Increased systemic perfusion

a. Anaerobic metabolism c. Hypotension The common signs and symptoms of shock, no matter the cause, are directly related to the effects of anaerobic metabolism and hypotension. Hyperglycemia, impaired renal function, and increased perfusion are not the cause of common signs and symptoms of shock.

4. A client is in the early stages of shock and is restless. What comfort measures does the nurse delegate to the assistive personnel (AP)? (Select all that apply.) a. Bringing the client warm blankets b. Giving the client hot tea to drink c. Massaging the client's painful legs d. Reorienting the client as needed e. Sitting with the client for reassurance

a. Bringing the client warm blankets b. Giving the client hot tea to drink d. Reorienting the client as needed e. Sitting with the client for reassurance The AP can bring the client warm blankets, reorient the client as needed to decrease anxiety, and sit with the client for reassurance. If the nurse assesses the client is safely able to swallow, small amounts of fluids would be allowed. Massaging the legs is not recommended as this can dislodge any clots present, which may lead to pulmonary embolism.

3. The nurse gets the hand-off report on four clients. Which client would the nurse assess first? a. Client with a blood pressure change of 128/74 to 110/88 mm Hg b. Client with oxygen saturation unchanged at 94% c. Client with a pulse change of 100 to 88 beats/min d. Client with urine output of 40 mL/hr for the last 2 hours

a. Client with a blood pressure change of 128/74 to 110/88 mm Hg This client has a falling systolic blood pressure, rising diastolic blood pressure, and narrowing pulse pressure, all of which may be indications of worsening perfusion status and possible shock. The nurse would assess this client first. The client with the unchanged oxygen saturation is stable at this point. Although the client with a change in pulse has a slower rate, it is not an indicator of shock since the pulse is still within the normal range; it may indicate that the client's pain or anxiety has been relieved, or he or she is sleeping or relaxing. A urine output of 40 mL/hr is above the normal range, which is 30 mL/hr.

13. The nurse is teaching an uncircumcised 65-year-old client about self-management of a urinary catheter in preparation for discharge to his home. What statement indicates a lack of understanding by the client? a. I only have to wash the outside of the catheter once a week. b. I should take extra time to clean the catheter site by pushing the foreskin back. c. The drainage bag needs to be changed at least once a week and as needed. d. I should pour a solution of vinegar and water through the tubing and bag.

a. I only have to wash the outside of the catheter once a week. The first few inches of the catheter must be washed daily starting at the penis and washing outward with soap and water. The other options are correct for self-management of a urinary catheter in the home setting.

13. A nurse receives hand-off report from the emergency department on a new admission suspected of having septic shock. The client's qSOFA score is 3. What action by the nurse is best? a. Plan to calculate a full SOFA score on arrival. b. Contact respiratory therapy about ventilator setup. c. Arrange protective precautions to be implemented. d. Call the hospital chaplain to support the family.

a. Plan to calculate a full SOFA score on arrival. The qSOFA score is an abbreviated Sequential Organ Failure Assessment (or "quick"). A score of 3 is high and requires the nurse to assess the client further for organ impairment. The client may or may not need a ventilator, but that in not specified in the score. The client does not need protective precautions. The client's family may well need support, but the nurse would assess their needs and wishes prior to calling the chaplain.

7. A client is diagnosed with metastatic prostate cancer. The client asks the nurse the purpose of his treatment with the luteinizing hormonereleasing hormone (LH-RH) agonist leuprolide (Lupron) and the bisphosphonate pamidronate (Aredia). Which statement by the nurse is most appropriate? a. The treatment reduces testosterone and prevents bone fractures. b. The medications prevent erectile dysfunction and increase libido. c. There is less gynecomastia and osteoporosis with this drug regimen. d. These medications both inhibit tumor progression by blocking androgens.

a. The treatment reduces testosterone and prevents bone fractures. Lupron, an LH-RH agonist, stimulates the pituitary gland to release luteinizing hormone (LH) to the point that the gland is depleted of LH and testosterone production is lessened. This may decrease the prostate cancer since it is hormone dependent. Lupron can cause osteoporosis, which results in the need for Aredia to prevent bone loss. Erectile dysfunction, decreased libido, and gynecomastia are side effects of the LH-RH medications. Antiandrogen drugs inhibit tumor progression by blocking androgens at the site of the prostate.

After a transrectal ultrasound with biopsy for prostate cancer, what instructions will the nurse provide the client? a. report fever, chills, bloody urine, and any difficulty voiding b. the biopsy will diagnose if you have prostate cancer c. drink plenty of fluids during the first 24 hours d. expect to see bright red bleeding at first e. report any pink color in the urine to the health care provider f. avoid strenuous physical activity

a. report fever, chills, bloody urine, and any difficulty voiding b. the biopsy will diagnose if you have prostate cancer c. drink plenty of fluids during the first 24 hours f. avoid strenuous physical activity

What does the nurse teach an older client with prostate cancer who is scheduled to have a digital rectal examination (DRE) and a prostate-specific antigen (PSA) test? a. the PSA laboratory test is drawn before the DRE b. the DRE is completed 2 weeks before the PSA c. the PSA is reviewed first because DRE may not be necessary d. both tests can be completed at the client's convenience

a. the PSA laboratory test is drawn before the DRE

1. The nurse is administering finasteride (Proscar) and doxazosin (Cardura) to a 67-year-old client with benign prostatic hyperplasia. What precautions are related to the side effects of these medications? (Select all that apply.) a.Assessing for blood pressure changes when lying, sitting, and arising from the bed b.Immediately reporting any change in the alanine aminotransferase laboratory test c.Teaching the client about the possibility of increased libido with these medications d.Taking the clients pulse rate for a minute in anticipation of bradycardia e.Asking the client to report any weakness, light-headedness, or dizziness

a.Assessing for blood pressure changes when lying, sitting, and arising from the bed b.Immediately reporting any change in the alanine aminotransferase laboratory test e.Asking the client to report any weakness, light-headedness, or dizziness Both the 5-alpha-reductase inhibitor (5-ARI) and the alpha1-selective blocking agents can cause orthostatic (postural) hypotension and liver dysfunction. The 5-ARI agent (Proscar) can cause a decreased libido rather than an increased sexual drive. The alpha-blocking drug (Cardura) can cause tachycardia rather than bradycardia.

A week after kidney transplantation, the client develops a temperature of 101 degrees F, the BP is elevated, and the flank area is tender. The x-ray shows that the transplanted kidney is enlarged. Based on these assessment findings, the nurse would suspect which of the following complications? Acute kidney infection Renal artery stenosis Hyperacute rejection Acute rejection

acute rejection Acute rejection most often occurs in the first 2 weeks after transplantation. Clinical manifestations include fever, malaise, elevated white blood cell count, acute hypertension, graft tenderness, and manifestations of deteriorating renal function. Chronic rejection occurs gradually over a period of months to years. Although kidney infection or obstruction can occur, the symptoms presented in the question do not relate specifically to these disorders.

20. The nurse is caring for a patient with signs and symptoms of prerenal azotemia. A fluid challenge is performed to promote kidney perfusion by doing what? Instilling warm sterile normal saline into the bladder Administering drugs to suppress aldosterone release administering normal saline 500-1000 mL infused over one hour Having the patient drink several large glasses of water.

administering normal saline 500-1000 mL infused over one hour

A 52-year old diagnosed with cirrhosis presents to the ED, vomiting dark brown emesis, and is very anxious and restless. The ED nurse realizes that the first priority should be to: assess for signs of hypovolemia test the emesis and stool for blood insert an NG tube and initiate irrigation with iced normal saline administer Zofran for vomiting

assess for signs of hypovolemia

The nurse is teaching a client and family about home care following a transjugular intrahepatic portal-systemic shunt (TIPS) procedure. Which client finding would the nurse teach the family to report to the primary health care provider immediately? a) Decreased ascitic fluid b) Changes in consciousness or behavior c) Fatigue and weakness d) Decreased pulse rate

b) Changes in consciousness or behavior Although serious complications of the TIPS are not common, the client needs to be monitored for hepatic encephalopathy. This complication is manifested by changes in consciousness, mental status, and/or behavior. A decreased pulse rate and ascitic fluid are expected and clients with cirrhosis are usually fatigued and weak.

The nurse is caring for a client who just had a paracentesis. Which client finding indicates that the procedure was effective? a) Increased blood pressure b) Decreased weight c) Increased pulse d) Decreased pain

b) Decreased weight A paracentesis is performed to remove ascitic fluid from the abdomen. Therefore, the client should weigh less after the procedure than before. Blood pressure should decrease due to less fluid volume and the pulse rate may not be affected. The client may report less abdominal discomfort or ease in breathing, but pain is not a common problem for cirrhotic clients.

When caring for a client with Laennec cirrhosis, which of these findings does the nurse expect to find on assessment? (Select all that apply.) a) Elevated magnesium b) Swollen abdomen c) Prolonged partial thromboplastin time d) Elevated amylase level e) Currant jelly stool f) Icterus of skin

b) Swollen abdomen c) Prolonged partial thromboplastin time f) Icterus of skin Clients with Laennec cirrhosis have damaged clotting factors, so prolonged coagulation times and bleeding may result. Icterus, or jaundice, results from cirrhosis. The client with cirrhosis may develop ascites, or fluid in the abdominal cavity.Elevated magnesium is not related to cirrhosis. Amylase is typically elevated in pancreatitis. Currant jelly stool is consistent with intussusception, a type of bowel obstruction. The client with cirrhosis may develop hypocalcemia and/or hypokalemia. It is also consistent with elevations of aspartate aminotransferase (AST), alanine aminotransferase (ALT), and alkaline phosphatase.

2. A nurse is caring for a client after surgery. The client's respiratory rate has increased from 12 to 18 breaths/min and the pulse rate increased from 86 to 98 beats/min since the client was last assessed 4 hours ago. What action by the nurse is best? a. Ask if the client needs pain medication. b. Assess using the MEWS score. c. Document the findings in the client's chart. d. Increase the rate of the client's IV infusion.

b. Assess using the MEWS score. Signs of the earliest stage of shock are subtle and may manifest in slight increases in heart rate, respiratory rate, or blood pressure. Even though these readings are not out of the normal range, the nurse would conduct a thorough assessment of the patient, focusing on indicators of perfusion. The MEWS score (Modified Early Warning Score) was developed to identify clients at risk for deterioration. The client may need pain medication, but this is not the priority at this time. Documentation would be done thoroughly but would be done after the assessment. The nurse would not increase the rate of the IV infusion without an order.

A nurse and unlicensed assistive personnel (UAP) are helping a client during a hysterosalpingogram. Which action by the nurse is best delegated to the UAP? a. Witnessing of the consent form b. Assisting the client into a lithotomy position c. Asking about allergies to iodine or shellfish d. Assessing for pelvic or shoulder pain after the study

b. Assisting the client into a lithotomy position The UAP would be able to position the client for the procedure. Only the nurse has the ability to witness the consent form and assess allergies and pain within the nursing scope of practice.

7. A nurse works at a community center for older adults. What self-management measure can the nurse teach the clients to prevent shock? a. Do not get dehydrated in warm weather. b. Drink fluids on a regular schedule. c. Seek attention for any lacerations. d. Take medications as prescribed.

b. Drink fluids on a regular schedule. Preventing dehydration in older adults is important because the age-related decrease in the thirst mechanism makes them prone to dehydration. Having older adults drink fluids on a regular schedule will help keep them hydrated without the influence of thirst (or lack of thirst). Telling clients not to get dehydrated is important, but not the best answer because it doesn't give them the tools to prevent it from occurring. Older adults should seek attention for lacerations, but this is not as important an issue as staying hydrated. Taking medications as prescribed may or may not be related to hydration.

A client is concerned about her irregular menstrual periods since she has increased her daily workouts at the gym to 2 hours each day. What is the nurses best response? a. Do you want to talk about the need for that much exercise? b. Exercise is healthy but can decrease body fat and cause irregular periods. c. Bingeing and purging can cause electrolyte problems in your body. d. Anorexic behavior can result in decreased estrogen levels.

b. Exercise is healthy but can decrease body fat and cause irregular periods. There needs to be a certain level of body fat and weight to maintain regular menstrual cycles. The client has only indicated that she has increased her workouts. There is no indication that she has anorexic or bingeing and purging behaviors.

12. A nurse is caring for several clients at risk for shock. Which laboratory value requires the nurse to communicate with the primary health care provider? a. Creatinine: 0.9 mg/dL (68.6 mcmol/L) b. Lactate: 5.4 mg/dL (6 mmol/L) c. Sodium: 150 mEq/L (150 mmol/L) d. White blood cell count: 11,000/mm3 (11 109/L)

b. Lactate: 5.4 mg/dL (6 mmol/L) A lactate level of 5.4 mg/dL (6 mmol/L) is high and is indicative of possible shock. A creatinine level of 0.9 mg/dL (68.6 mcmol/L) is normal. A sodium level of 150 mEq/L (150 mmol/L) is slightly high but does not need to be communicated. A white blood cell count of 11,000/mm3 (11 109/L) is slightly high but is not as critical as the lactate level.

4. A nurse is caring for a client after surgery who is restless and apprehensive. The assistive personnel (AP) reports the vital signs and the nurse sees that they are only slightly different from previous readings. What action does the nurse delegate next to the AP? a. Assess the client for pain or discomfort. b. Measure urine output from the catheter. c. Reposition the client to the side. d. Stay with the client and reassure him or her.

b. Measure urine output from the catheter. Urine output changes are a sensitive early indicator of shock. The nurse would delegate emptying the urinary catheter and measuring output to the AP as a baseline for hourly urine output measurements. The AP cannot assess for pain. Repositioning may or may not be effective for decreasing restlessness, but does not take priority over physical assessments. Reassurance is a therapeutic nursing action, but the nurse needs to do more in this situation.

12. A 70-year-old client returned from a transurethral resection of the prostate 8 hours ago with a continuous bladder irrigation. The nurse reviews his laboratory results as follows: Sodium 128 mEq/L Hemoglobin 14 g/dL Hematocrit 42% Red blood cell count 4.5 What action by the nurse is the most appropriate? a. Consider starting a blood transfusion. b. Slow down the bladder irrigation if the urine is pink. c. Report the findings to the surgeon immediately. d. Take the vital signs every 15 minutes.

b. Slow down the bladder irrigation if the urine is pink. The serum sodium is decreased due to large-volume bladder irrigation (normal is 136 to 145 mEq/L). By slowing the irrigation, there will be less fluid overload and sodium dilution. The hemoglobin and hematocrit values are a low normal, with a slight decrease in the red blood cell count. Therefore, a blood transfusion or frequent vital signs should not be necessary. Immediate report to the surgeon is not necessary.

5. A client has returned from a transurethral resection of the prostate with a continuous bladder irrigation. Which action by the nurse is a priority if bright red urinary drainage and clots are noted 5 hours after the surgery? a. Review the hemoglobin and hematocrit as ordered. b. Take vital signs and notify the surgeon immediately. c. Release the traction on the three-way catheter. d. Remind the client not to pull on the catheter.

b. Take vital signs and notify the surgeon immediately. Bright red urinary drainage with clots may indicate arterial bleeding. Vital signs should be taken and the surgeon notified. The traction on the three-way catheter should not be released since it places pressure at the surgical site to avoid bleeding. The nurses review of hemoglobin and hematocrit and reminding the client not to pull on the catheter are good choices, but not the priority at this time.

Which behavioral modification instructions will the nurse teach a client with benign prostatic hyperplasia (BPH)? SATA a. take diuretics to increase urine output b limit alcohol intake c. avoid caffeine containing beverages d. do not consume large amounts of fluid in a short time e. avoid sexual intercourse f. avoid taking antihistamine drugs

b. limit alcohol intake c. avoid caffeine containing beverages d. do not consume large amounts of fluid in a short time f. avoid taking antihistamine drugs

What action will the advanced practice nurse take after performing a digital rectal examination on a client with benign prostatic hyperplasia (BPH)? a. instruct the client to remain in a supine position with knees bent b. massage the prostate to obtain a fluid sample for possible prostatitis c. use a sterile cotton-tipped applicator for a sample from the penis for possible infection d. administer pain medication to relieve the discomfort from the examination

b. massage the prostate to obtain a fluid sample for possible prostatitis

Which diagnostic procedure does the nurse expect the health care provider to order to test a client with an enlarged bladder, for bladder outlet obstruction? a. bladder ultrasound scan b. urodynamic pressure-flow study c. computed tomography scan d. transrectal ultrasound

b. urodynamic pressure-flow study

9. When a client is in the diuretic phase of acute kidney injury (AKI), what priority action will the nurse take? A. Assessing for hypertension and fluid overload B. Monitoring for hypovolemia and electrolyte loss C. Adjusting the dosage of diuretic medications D. Balancing diuretic therapy with intake and output

✅ B For the client in the diuretic phase of AKI, the nurse plans care that focuses on fluid and electrolyte replacement and monitoring. Onset of polyuria can signal the start of recovery from AKI.

What teaching does the home health nurse give the family of a client with hepatitis C to prevent the spread of the infection? a) Drink only bottled water and avoid ice. b) Avoid sharing the bathroom with the client. c) Members of the household must not share toothbrushes. d) The client must not consume alcohol.

c) Members of the household must not share toothbrushes. The nurse teaches the family of a client with hepatitis C that toothbrushes, razors, towels, and any other items may spread blood and body fluids and must not be shared.The client should not consume alcohol, but abstention will not prevent spread of the virus. The client may share a bathroom if he or she is continent. To prevent hepatitis A when traveling to foreign countries, bottled water should be consumed and ice made from tap water needs to be avoided.

When providing community education, the nurse emphasizes that which group needs to receive immunization for hepatitis B? a) Clients who work with shellfish. b) Clients with elevations of aspartate aminotransferase and alanine aminotransferase. c) Men who engage in sex with men. d) Clients traveling to a third-world country.

c) Men who engage in sex with men. Men who prefer sex with men are at increased risk for hepatitis B, which is spread by the exchange of blood and body fluids during sexual activity.Consuming raw or undercooked shellfish may cause hepatitis A, not hepatitis B. Travel to third-world countries exposes the traveler to contaminated water and risk for hepatitis A. Hepatitis B is not of concern, unless the client is exposed to blood and body fluids during travel. Clients who have liver disease should receive the vaccine, but men who have sex with men are at higher risk for contracting hepatitis B.

14. What is the nurse's priority action when the health care provider orders IV fluids at a rate of 1 mL/kg/hr for 12 hours prior to a CT scan with contrast media for a client who weighs 152 lbs? A. Set the IV pump to deliver fluid at 69 mL/hr. B. Set the IV pump to deliver fluid at 152 mL/hr. C. Call the health care provider for clarification of the order. D. Ask the radiologist for clarification of the order.

✅A The nurse calculates the client's weight in kilograms (152 lb divided by 2.2 = 69 kg). Then the nurse sets the IV pump to run at 69 mL/hr.

The nurse is assessing the reproductive history of a 68-year-old postmenopausal woman. Which finding is cause for immediate action by the nurse? a. Vaginal dryness b. Need for a Papanicolaou test if none for 3 years c. Bleeding from the vagina d. Leakage of urine

c. Bleeding from the vagina Vaginal bleeding is not normal for the postmenopausal woman. Vaginal dryness and leakage of urine are common findings in adults of this age range. Pap tests may not be needed for women over 65 who have had regular cervical cancer testing with normal results.

11. A 25-year-old client has recently been diagnosed with testicular cancer and is scheduled for radiation therapy. Which intervention by the nurse is best? a. Ask the client about his support system of friends and relatives. b. Encourage the client to verbalize his fears about sexual performance. c. Explore with the client the possibility of sperm collection. d. Provide privacy to allow time for reflection about the treatment.

c. Explore with the client the possibility of sperm collection. Sperm collection is a viable option for a client diagnosed with testicular cancer and should be completed before radiation therapy, chemotherapy, or radical lymph node dissection. The other options would promote psychosocial support but are not the priority intervention.

9. A 55-year-old male client is admitted to the emergency department with symptoms of a myocardial infarction. Which question by the nurse is the most appropriate before administering nitroglycerin? a. On a scale from 0 to 10, what is the rating of your chest pain? b. Are you allergic to any food or medications? c. Have you taken any drugs like Viagra recently? d. Are you light-headed or dizzy right now?

c. Have you taken any drugs like Viagra recently? Phosphodiesterase-5 inhibitors such as sildenafil (Viagra) relax smooth muscles to increase blood flow to the penis for treatment of erectile dysfunction. In combination with nitroglycerin, there can be extreme hypotension with reduction of blood flow to vital organs. The other questions are appropriate but not the highest priority before administering nitroglycerin.

6. A nurse caring for a client notes the following assessments: white blood cell count 3800/mm3 (3.8 109/L), blood glucose level 198 mg/dL (11 mmol/L), and temperature 96.2° F (35.6° C). What action by the nurse takes priority? a. Document the findings in the client's chart. b. Give the client warmed blankets for comfort. c. Notify the primary health care provider immediately. d. Prepare to administer insulin per sliding scale.

c. Notify the primary health care provider immediately. This client has several indicators of sepsis with systemic inflammatory response. The nurse would notify the primary health care provider immediately. Documentation needs to be thorough but does not take priority. The client may appreciate warm blankets, but comfort measures do not take priority. The client may need insulin if blood glucose is being regulated tightly.

3. A 55-year-old African-American client is having a visit with his health care provider. What test should the nurse discuss with the client as an option to screen for prostate cancer, even though screening is not routinely recommended? a. Complete blood count b. Culture and sensitivity c. Prostate-specific antigen d. Cystoscopy

c. Prostate-specific antigen The prostate-specific antigen test should be discussed as an option for prostate cancer screening. A complete blood count and culture and sensitivity laboratory test will be ordered if infection is suspected. A cystoscopy would be performed to assess the effect of a bladder neck obstruction.

10. A nurse is caring for a client who has a diagnosis of multiple organ dysfunction syndrome (MODS) who will be receiving sodium nitroprusside via IV infusion. What action by the nurse causes the charge nurse to intervene? a. Assessing the IV site before giving the drug b. Obtaining a programmable ("smart") IV pump c. Removing the IV bag from the brown plastic cover d. Taking and recording a baseline set of vital signs

c. Removing the IV bag from the brown plastic cover Nitroprusside degrades in the presence of light, so it must be protected by leaving it in the original brown plastic bag when infusing. The other actions are correct

15. A client with MODS has been started on dobutamine. What assessment finding requires the nurse to communicate with the primary health care provider immediately? a. Blood pressure of 98/68 mm Hg b. Pedal pulses 1+/4+ bilaterally c. Report of chest heaviness d. Urine output of 32 mL/hr

c. Report of chest heaviness Chest heaviness or pain indicates myocardial ischemia, a possible adverse effect of dobutamine. While taking dobutamine, the oxygen requirements of the heart are increased due to increased myocardial workload, and may cause ischemia. Without knowing the client's previous blood pressure or pedal pulses, there is not enough information to determine if these are an improvement or not. A urine output of 32 mL/hr is acceptable.

When preparing a client to undergo paracentesis, which action is necessary to reduce potential injury as a result of the procedure? a) Assist the provider to insert a trocar catheter into the abdomen. b) Position the client with the head of the bed flat. c) Encourage the client to take deep breaths and cough. d) Ask the client to void prior to the procedure.

d) Ask the client to void prior to the procedure. To avoid injury to the bladder during a paracentesis, the client would be asked to void prior to the procedure. Taking deep breaths and coughing does not prevent complications or injury as a result of paracentesis. Clients would be positioned with the head of the bed elevated. The trocar catheter is used to drain the ascetic fluid and does not reduce the risk of damage to the bladder.

It is essential that the nurse monitor the client returning from hepatic artery embolization for hepatic cancer for which potential complication? a) Right shoulder pain b) Bone marrow suppression c) Polyuria d) Bleeding

d) Bleeding A potential complication of hepatic artery embolization for hepatic cancer is bleeding. Prompt detection of hemorrhage is the priority.Discomfort such as right shoulder pain may be present, but the priority is to assess for hemorrhage. The nurse must assess for signs of shock, not polyuria. Embolization does not suppress the bone marrow. If chemotherapy or immune modulators is used, the nurse then assesses for bone marrow suppression.

The nurse is caring for a client who had a liver transplant last week. For which complication will the nurse teach the client and family to monitor? a) Acute kidney injury b) Hypertension c) Pulmonary edema d) Infection

d) Infection The client is at the most risk for rejection of the transplant which can be the result of an infection if not identified and managed effectively. Therefore, the nurse would teach the client and family to report cough, fever, skin redness, and other signs of infection.

Which client does the nurse recognize has the highest risk for development of prostate cancer? a. 45-year-old Asian American with a history of benign prostatic hyperplasia b 55-year-old Hispanic American who practices poor dietary intake c. 65-year-old Caucasian American with two cousins who developed prostate cancer d. 75-year-old African American whose father and brother developed prostate cancer

d. 75-year-old African American whose father and brother developed prostate cancer

11. A nurse on the general medical-surgical unit is caring for a client in shock and assesses the following: Respiratory rate: 10 breaths/min Pulse: 136 beats/min Blood pressure: 92/78 mm Hg Level of consciousness: responds to voice Temperature: 101.5° F (38.5° C) Urine output for the last 2 hours: 40 mL/hr. What action by the nurse is best? a. Transfer the client to the Intensive Care Unit. b. Continue monitoring every 30 minutes. c. Notify the unit charge nurse immediately. d. Call the Rapid Response Team.

d. Call the Rapid Response Team. This client has a MEWS score of 7 (RR: 0, P: 3, SBP: 1, LOC: 1, Temperature: 1, UO: 1). Scores above 5 are associated with a high risk of death and ICU admission. The most important action for the nurse is to notify the Rapid Response Team so that timely interventions can be initiated. The client most likely will be transferred to the ICU, but an order is required. Monitoring the client every 30 minutes is appropriate, but the nurse needs to obtain care for the client. The charge nurse is a valuable resource, but the best action is to notify the Rapid Response Team.

A client is scheduled for a laparoscopy to remove endometriosis tissue. Which response by the client alerts the nurse of the need for further teaching? a. The surgeon told me that carbon dioxide would be infused into my pelvic cavity. b. There will be one or more small incisions in order to visualize all of the organs. c. There will be some shoulder pain after the procedure that may last 48 hours. d. I can return to jogging my 3-mile routine in a few days.

d. I can return to jogging my 3-mile routine in a few days. No strenuous activity should occur for 7 days after the procedure. Carbon dioxide is infused into the pelvic cavity to visualize the organs. There are only one or more small incisions with this procedure. The referred shoulder pain that will occur should only last 48 hours.

The mother of an 18-year-old girl asks the nurse which screening her daughter should receive now based on evidence-based recommendations. Which suggestion by the nurse is best? a. Papanicolaou test b. Human papilloma virus (HPV) test c. Mammogram d. No screenings at this time

d. No screenings at this time Since the daughter is only 18, it is not recommended that she receive any of these screenings. Pap screenings are recommended to start at age 21. The HPV test can be done with the Pap test for women older than 30 or who had an abnormal Pap test result. A mammogram is recommended for women age 40 or older since cancers are more able to be distinguished from normal glandular tissue at that age.

A 67-year-old male client had some serum tests performed during his annual examination. The nurse reviews his results, as follows: testosterone: 680 ng/dL; prostate-specific antigen: 10 ng/mL; prolactin: 5 ng/mL. What action by the nurse is best? a. Assess for possible galactorrhea with breast discharge. b. Note the possibility of a testicular tumor. c. Communicate to the provider that results were normal. d. Prepare the client for further diagnostic testing.

d. Prepare the client for further diagnostic testing. The prostate-specific antigen is increased from the normal of 0 to 2.5, which could indicate benign prostatic hyperplasia or prostate cancer. Further testing would have to be done. The values of testosterone and prolactin are within normal range. If the prolactin were increased, there would be a possibility of galactorrhea. An increase in testosterone could indicate a possible testicular tumor.

2. A client is diagnosed with benign prostatic hyperplasia and seems sad and irritable. After assessing the clients behavior, which statement by the nurse would be the most appropriate? a.The urine incontinence should not prevent you from socializing. b.You seem depressed and should seek more pleasant things to do. c.It is common for men at your age to have changes in mood. d.Nocturia could cause interruption of your sleep and cause changes in mood.

d.Nocturia could cause interruption of your sleep and cause changes in mood. Frequent visits to the bathroom during the night could cause sleep interruptions and affect the clients mood and mental status. Incontinence could cause the client to feel embarrassment and cause him to limit his activities outside the home. The social isolation could lead to clinical depression and should be treated professionally. The nurse should not give advice before exploring the clients response to his change in behavior. The statement about age has no validity.

27. Which drug will the nurse avoid administering to a client with chronic kidney disease (CKD) to prevent harm? A. Opioids B. Antibiotics C. Oral antihyperglycemics D. Magnesium antacids

✅D The nurse questions a prescription for magnesium-containing antacids for clients with CKD because they cannot excrete magnesium and need to avoid any additional intake and build- up of magnesium. To avoid hypermagnesemia, the nurse teaches clients with kidney disease to avoid antacids containing magnesium.

6. Which assessment questions are most appropriate for the nurse to ask a client at risk for acute kidney injury (AKI)? Select all that apply. A. "Have you noticed any changes in your urine's appearance, frequency, or volume?" B. "Have you experienced any leakage of urine when coughing or laughing?" C. "Do you weigh yourself and have you noticed any unexpected weight loss?" D. "Do you have a history of diabetes, hypertension, or peripheral vascular disease?" E. "Do you use any nonsteroidal anti-inflammatory drugs regularly?" F. "Have you had and recent surgeries, traumas, or transfusions?"

✅ A, D, E, F The nurse asks about any noted changes in urine, as well as any exposure to nephrotoxic substances or drugs. Other important information from the client's medical history includes surgeries, trauma, transfusions, and chronic conditions such as diabetes, hypertension, and peripheral vascular disease. For additional essential topic, see the History section on AKI in your text.

42. Why will the nurse immediately notify the nephrology health care provider if a client develops hypotension and diuresis postoperatively after a kidney transplant? A. These problems place the client at risk for hypervolemia and dehydration. B. Dehydration with hypotension reduces perfusion and oxygen to the new kidney. C. These assessment findings are indicators of a possible serious acute infection. D. Increased work by the kidney for diuresis results in excessive buildup of cellular toxins that damage the new kidney's tubules.

✅ B If hypotension or excessive diuresis (e.g., unanticipated urine output 500 to 1000 mL greater than intake over 12 to 24 hours or other goal for intake and output) is present, respond by notifying the nephrology health care provider because hypotension reduces perfusion and oxygen to the new kidney, threatening the kidney's survival.

1. Which criteria does the nurse understand are included in the current definition of acute kidney injury (AKI)? Select all that apply. A. Signs and symptoms of fluid overload such as peripheral edema and crackles in the lungs B. Urine volume of less than 0.5 mL/kg/hr for 6 hours C. Presence of polyuria, nocturia, and very dilute pale yellow urine D. Increase in serum creatinine by 0.3 mg/dL (26.2 μmol/L) or more within 48 hours E. Hypotension and tachycardia with progressively decreased amounts of urine F. Increase in serum creatinine to 1.5 times or more from baseline in the previous 7 days

✅ B, D, F The most current definition of AKI is an increase in serum creatinine by 0.3 mg/dL (26.2 μmol/L) or more within 48 hours; or an increase in serum creatinine to 1.5 times or more from baseline, which is known or presumed to have occurred in the previous 7 days; or a urine volume of less than 0.5 mL/kg/hr for 6 hours.

5. Which condition will the nurse recognize increases the risk for a client with benign prostatic hyperplasia (BPH) to develop? A. Perfusion reduction (prerenal failure) B. Intrinsic or intrarenal failure C. Urine flow obstruction (postrenal failure) D. End-stage kidney disease

✅ C BPH (enlarged prostate gland) increases the client's risk for urine flow obstruction leading to postrenal failure because the prostate gland surrounds and puts pressure on the urethra.

18. For which condition does the nurse suspect a client with chronic kidney disease (CKD) is attempting to compensate for when respirations increase in rate and depth? 1. Hypoxia B. Alkalosis C. Acidosis D. Hypoxemia

✅ C As CKD worsens and acid retention increases, increased respiratory action is needed to keep the blood pH normal. The respiratory system adjusts or compensates for the increased blood hydrogen ion levels (acidosis or decreased pH) by increasing the rate and depth of breathing to excrete carbon dioxide through the lungs. This breathing pa

19. Which instruction will the nurse give an assistive personnel (AP) to prevent harm when providing care to a client who has osteodystrophy? A. Assist the client with feeding for all meals. B. Gently wash the client's skin with a mild soap and rinse well. C. Assist the client with ambulation to the toilet every 2 hours. D. Use a lift sheet when moving or lifting the client.

✅ D Clients with osteodystrophy have thin, fragile bones that are at risk for fractures with even slight trauma. When lifting or moving a client with fragile bones, the AP is instructed to use a lift sheet rather than pulling the client.

33. How does the nurse best interpret a condition when a client is undergoing hemodialysis (HD) and develops symptoms including headache, nausea, vomiting, and fatigue? A. Mild dialysis disequilibrium syndrome B. Adverse reaction to the dialysate solution C. Transient symptoms in a client new to hemodialysis D. Expected manifestations of end-stage kidney disease

✅. A Dialysis disequilibrium syndrome may develop during HD or after HD has been completed. It is characterized by mental status changes and can include seizures or coma, although this severity of disequilibrium syndrome is rare with today's HD practice. A mild form of disequilibrium syndrome includes symptoms of nausea, vomiting, headaches, fatigue, and restlessness. It is thought to be the result of a rapid reduction in electrolytes and other particles.

29. For which client conditions does the nurse expect the possibility of emergent hemodialysis (HD)? Select all that apply. A. Severe uncontrollable hypertension B. Pericarditis C. Symptomatic hyperkalemia with ECG changes D. Myocardial infarction E. Pulmonary edema F. Some drug overdoses

✅. A, B, C, E, F Some indications for emergent dialysis include: pulmonary edema; severe uncontrollable hypertension; symptomatic hyperkalemia with ECG changes; other severe electrolyte or acid-base disturbances; some drug overdoses; and pericarditis.

30. What are the criteria used for selection of clients for hemodialysis (HD)? Select all that apply. A. Client values and preferences B. Client's family member or partner who is willing to learn about HD C. Irreversible kidney failure when other therapies are unacceptable or ineffective D. No disorders that would seriously complicate HD E. Expected ability to continue or resume roles at home, work, or school F. Insurance plan will cover costs of procedures

✅. A, C, D, E Selection criteria for HD include: irreversible kidney failure when other therapies are unacceptable or ineffective; no disorders that would seriously complicate HD; client values and preferences; and expected ability to continue or resume roles at home, work, or school.

32. What is the nurse's best response when a client asks how often and for how long he or she will have to go for hemodialysis (HD)? A. "It varies and you will need to discuss this with your nephrology health care provider for specific instructions." B. "Most clients require about 12 hours per week, which is usually divided into three 4-hour treatments." C. "If you follow the diet and fluid therapies you will spend less time in dialysis, about 8 hours each week." D. "Many clients prefer to have home treatment dialysis that occurs every night while sleeping."

✅. B The best answer the nurse can provide this client is the most common treatment. Most clients receive three 4-hour treatments over the course of a week. The nurse provides additional information for some clients with ongoing urine production, who may need only two 5- to 6-hour treatments a week. If a client gains large amounts of fluid, a longer HD treatment time may be needed to remove the fluid without hypotension or other severe side effects.

17. When the nurse reviews the laboratory results and finds that a client with chronic kidney disease (CKD) has a serum potassium level of 8 mEq/L (mmol/L), which assessment will be completed before notifying the health care provider? A. Cardiac rhythm B. Respiratory rate and depth C. Tremors of the hands D. Change in urine appearance

✅A Normal potassium level is within 3.5 to 5 mEq/L (mmol/L). With CKD, high potassium (K+) levels can develop quickly, reaching 7 to 8 mEq/L (mmol/L) or greater. Life-threatening changes in cardiac rate and rhythm result from K+ elevation because of abnormal depolarization and repolarization.

43. What does the nurse expect the nephrology health care provider to prescribe when a post kidney transplant client develops oliguria, elevated temperature of 100° F (37.8° C), increased blood pressure, and signs of fluid retention 9 days after the surgery? A. Immediate removal of the transplanted kidney B. Increased doses of immunosuppressive drugs C. Immediate return to either hemodialysis or peritoneal dialysis D. Antibiotic therapy until infection symptoms are resolved

✅. B These symptoms within the time frame of a week or more indicate that the client may be having an acute rejection. The treatment for acute rejection is increased dosages of immunosuppressive drugs. Immunosuppressive drugs protect the transplanted organ. These drugs include corticosteroids, inhibitors of T-cell proliferation and activity (azathioprine, mycophenolic acid, cyclosporine, and tacrolimus), mTOR inhibitors (to disrupt stimulatory T-cell signals), and monoclonal antibodies.

31. Which gastrointestinal changes does the nurse expect to find when assessing a client with uremia? A. Increased salivation B. Halitosis C. Stomatitis D. Anorexia E. Nausea and vomiting F. Hiccups

✅. B, C, D, E, F Uremia affects the entire GI system. The flora of the mouth change with uremia. The mouth contains the enzyme urease, which breaks down urea into ammonia. The ammonia generated remains and then causes halitosis (uremic fetor) and stomatitis (mouth inflammation). Anorexia, nausea, vomiting, and hiccups are common in clients with uremia. For more information about uremic changes in the body, see chart 63.2 Key Features of Uremia.

37. Which actions will the nurse take to check the peritoneal dialysis system of a client when the dialysate outflow is slow? Select all that apply. A. Ensuring that the drainage bag is elevated above the client's abdomen B. Inspecting the tubing to ensure there is no kinking or twisting C. Making sure that clamps are open and unclamped D. Repositioning the client to the other side and ensuring good body alignment E. Instructing the client to stand up at the bedside and cough F. Placing the client in a supine low-Fowler position

✅. B, C, D, F When PD outflow drainage is slow, actions that can help improve flow include: ensuring that the drainage bag is lower than the client's abdomen to enhance gravity drainage; inspecting the connection tubing and PD system for kinking or twisting; and ensuring that clamps are open. If outflow drainage is still inadequate, reposition the client to stimulate outflow. Turning the client to the other side or ensuring that he or she is in good body alignment may help. Having the client in a supine low- Fowler position reduces abdominal pressure. Increased abdominal pressure from si

36. Which priority teaching will the nurse provide to the client receiving peritoneal dialysis (PD) when the effluent becomes cloudy? A. The change means that more waste products are being removed from the blood. B. The presence of cloudiness is an early sign of an infection called peritonitis and is very serious. C. Effluent cloudiness is the result of eating foods that contain too much protein and electrolytes. D. The effluent is expected to be cloudy because it has spent time (dwelled) in the abdomen, in close contact with the intestines.

✅. B. The nurse teaches the client to recognize indications of peritonitis (e.g., cloudy dialysate outflow [effluent], fever, abdominal tenderness, abdominal pain, general malaise, nausea, and vomiting). Cloudy or opaque effluent is the earliest indication of peritonitis. The client is taught to examine all effluent for color and clarity to detect peritonitis early and to report indications of peritonitis immediately to the nephrology health care provider.

38. What is the best method for the nurse to monitor the weight of a client who is receiving peritoneal dialysis (PD)? A. Calculating the client's dry weight by comparing daily weights to baseline weights B. Determining dry weight by comparing the client's weight to a standard weight chart C. Checking the weight after a drain and before the next fill to monitor the dry weight D. Weighing the client daily and subtracting dialysate volume to determine dry weight

✅. C The client's actual weight is his or her "dry weight". For a client receiving PD, dry weight is checked after a drain and before the next fill. The client is always weighed on the same scale, with the same amount of clothes.

39. What is the nurse's best action when a client receiving PD has slightly less outflow than inflow? A. Placing the client on an oral fluid intake restriction B. Notifying the nephrology health care provider C. Recording the difference as intake on the flow sheet D. Instructing the client to stand and walk then measuring the next outflow

✅. C When outflow is less than inflow, the difference is retained by the client during dialysis and it is counted as fluid intake.

41. For how many hours will the nurse instruct the assistive personnel (AP) to check the hourly urine output of a postoperative client who had a kidney transplant? A. 8 hours B. 12 hours C. 24 hours D. 48 hours

✅. D A postoperative client who had a kidney transplant has a urinary catheter in place for accurate measurements of urine output and decompression of the bladder. Decompression prevents stretch on sutures and ureter a

35. What does the nurse expect when comparing a client's posthemodialysis weight and blood pressure with predialysis data? A. Blood pressure is increased and weight is decreased B. Blood pressure and weight are slightly increased C. Blood pressure and weight are the same D. Blood pressure and weight are decreased

✅. D Posthemodialysis, the nurse obtains vital signs and weight for comparison with predialysis measurements. After dialysis, the nurse expects blood pressure and weight to be reduced as a result of fluid removal.

34. What instructions will the nurse give to the assistive personnel (AP) regarding care of a client with an arteriovenous fistula? A. Assess for bleeding at the needle insertion sites every 2 hours. B. Monitor the client's distal pulses and capillary refill for circulation. C. Palpate the dialysis site for thrills and auscultate for a bruit every 4 hours. D. Avoid taking blood pressure readings on the client's arm with the arteriovenous fistula.

✅. D The AP's scope of practice includes taking and recording vital signs. For a hemodialysis client, checking blood pressure includes not taking blood pressure readings using the extremity in which the vascular access is placed. Assessment, monitoring, palpation, and auscultation are more advanced skills performed by the professional registered nurse. For more information on care of a client's arteriovenous fistula, see Best Practice for Patient Safety & Quality Care Caring for the Patient With an Arteriovenous Fistula or Arteriovenous Graft in your text.

25. Which nutritional supplements does the nurse expect the health care provider will prescribe for a client with chronic kidney disease? Select all that apply. A. Water-soluble vitamins B. Calcium C. Iron D. Magnesium E. Vitamin D F. Phosphorus

✅A, B, C, E The nurse expects the health care provider to prescribe daily vitamin and mineral supplements for most clients with CKD. Low-protein diets are also low in vitamins, and water-soluble vitamins are removed from the blood during dialysis. Anemia also is a problem in clients with CKD because of the limited iron content of low-protein diets and decreased kidney production of erythropoietin. Thus, supplemental iron is needed. Calcium and vitamin D supplements may be needed, depending on the client's serum calcium levels and bone status.

10. Which are the goals of nutritional support for a client with acute kidney injury (AKI) when the nurse collaborates with the registered dietitian nutritionist (RDN)? Select all that apply. A. Maintaining or improving nutritional status B. Creating a program for weight loss C. Preserving lean body mass D. Restoring or maintaining fluid balance E. Preserving kidney function F. Preventing end-state kidney disease

✅A, C, D, E Nutrition support goals in AKI are to provide sufficient nutrients to maintain or improve nutrition status, preserve lean body mass, restore or maintain fluid balance, and preserve kidney function.

40. Which client conditions will the nurse recognize as absolute contraindications to receiving a kidney transplant? Select all that apply. A. Breast cancer and metastasis to the lungs B. Type 2 diabetes controlled with diet and exercise C. Urinary tract infection D. Active treatment for peptic ulcer disease E. Chemical dependency F. Living related donor

✅A, C, E Absolute contraindications to kidney transplant include active cancer, current infection, active psychiatric illness, active substance abuse, and nonadherence with dialysis or medical regimen.

21. What urinalysis findings does the nurse expect when a client is in the early stage of chronic kidney disease? Select all that apply. A. Proteinuria B. Increased specific gravity C. Red blood cells (RBCs) D. Increased urine osmolarity E. White blood cells (WBCs) F. Glucosuria

✅A, C, E, F In the early stages of CKD, the nurse expects the urinalysis may show protein, glucose, red blood cells (RBCs) and white blood cells (WBCs), and decreased or fixed specific gravity. Urine osmolarity is usually decreased. As CKD progresses, urine output decreases dramatically, and osmolarity then increases.

16. Which electrolyte imbalance does the nurse expect when a client is in the early phase of chronic kidney disease (CKD)? A. Hyperkalemia B. Hyponatremia C. Hypercalcemia D. Hypokalemia

✅B Early in CKD, the nurse expects the client's laboratory values to reveal hyponatremia (sodium depletion) because there are fewer healthy kidney nephrons to reabsorb sodium.

26. What is the nurse's first action when a client with chronic kidney disease (CKD) develops restlessness, anxiousness, shortness of breath, a rapid heart rate, frothy sputum, and crackles in the bases of the lungs? A. Facilitating transfer to the intensive care unit for aggressive treatment B. Placing the client's head of bed in the high-Fowler position C. Monitoring vital signs and assessing the lungs every 15 minutes D. Administering an IV loop diuretic such as furosemide

✅B The nurse recognizes this client's symptoms as indicators of pulmonary edema. First, the client is placed in a high-Fowler position and given oxygen to improve gas exchange. Then health care provider or Rapid Response Team is notified for treatment and management of pulmonary edema.

23. The nurse collaborates with the registered dietician nutritionist (RDN) to teach a client about which recommendations for management of chronic kidney disease? Select all that apply. A. Reducing calories B. Controlling protein intake C. Limiting fluid intake D. Restricting potassium E. Increasing sodium F. Restricting phosphorus

✅B, C, D, F The nurse collaborates with the RDN to teach the client about diet changes that are needed as a result of CKD. Common changes include control of protein intake; fluid intake limitation; restriction of potassium, sodium, and phosphorus intake; taking vitamin and mineral supplements; and consuming enough calories to meet metabolic need.

15. Which outcome statement indicates to the nurse that the goal of giving a client IV therapy after a diagnostic imaging test with contrast media has been met? A. Lung sounds are clear and there are no signs or symptoms of fluid overload. B. The client has no signs or symptoms of contrast-induced immune response. C. Urine output is 150 mL/hr for 6 hours after the use of the contrast agent. D. Urine output is 0.5mL/kg/hr for 6 hours and the client remains euvolemic.

✅C A common desired outcome for clients undergoing a procedure with contrast medium is a urine output of 150 mL/hr for the first 6 hours after administration of the contrast agent.

13. Which type of medication does the nurse expect the health care provider to prescribe for a client with acute kidney injury to improve blood flow to the kidneys? A. Loop diuretics B. Phosphate binders C. Calcium channel blockers D. Erythropoietin-stimulating agents

✅C Calcium channel blockers can improve the GFR and blood flow within the kidney. They also help to control blood pressure.

28. Which client will the nurse consider most likely to be a candidate for continuous kidney replacement therapy (CKRT) using venovenous hemofiltration? A. 65-year-old with fluid volume overload B. 55-year-old who needs long-term management C. 45-year-old who is critically ill and unstable D. 35-year-old with a peritoneal infection

✅C Clients who need continuous kidney replacement therapy (CKRT) are hospitalized and are too unstable to tolerate the changes in blood pressure that occur with intermi

12. For a client diagnosed with acute kidney injury (AKI), the nurse considers questions an order for which diagnostic test? A. Ultrasonography B. Kidney-ureter-bladder x-ray (KUB) C. Computed tomography with contrast D. Kidney biopsy

✅C The client's diagnosis is AKI. To complete the CT with contrast, the client will be injected with a contrast dye, which is nephrotoxic, therefore the nurse considers questioning that diagnostic test.

24. To avoid harm and prevent osteodystrophy, which intracollaborative action does the nurse implement? A. Encouraging high-quality protein foods B. Administering iron supplements twice a day C. Encouraging extra milk with meals and snacks D. Administering phosphate binders with each meal

✅D Phosphorus restriction for control of phosphorus levels is started early in CKD to avoid renal osteodystrophy. The nurse administers phosphate binders at mealtime to increase their effectiveness in slowing or preventing the absorption of dietary phosphorus.


Related study sets

CH21 - trust funds and trust money

View Set

chapter 18 homework final review

View Set

Fundamentals of Nursing CH 19: Safety

View Set

Unit 4 Study Guide: Social Psychology Branscrombe

View Set

Reciprocal Trigonometric Functions

View Set

BUS101 - Module 6 - Week 1 - Management

View Set